You are on page 1of 223

MACMILLAN COLLEGE WORK OUT SERIES

Heat and
Thermodynamics
Titles in this Series

Dynamics Heat and Thermodynamics


Electric Circuits Mathematics for Economists
Electromagnetic Fields Molecular Genetics
Electronics Operational Research
Elements of Banking Organic Chemistry
Engineering Materials Physical Chemistry
Engineering Thermodynamics Structural Mechanics
Fluid Mechanics Waves and Optics
MACMILLAN COLLEGE WORK OUT SERIES

Heat
and
Thermodynamics
Michael Sprackling

M
150th YEAR

MACMILLAN
© Michael Sprackling 1993
All rights reserved. No reproduction, copy or transmission of
this publication may be made without written permission.
No paragraph of this publication may be reproduced, copied or
transmitted save with written permission or in accordance with
the provisions of the Copyright, Designs and Patents Act 1988,
or under the terms of any licence permitting limited copying
issued by the Copyright Licensing Agency, 90 Tottenham Court
Road, London W1P 9HE.

Any person who does any unauthorised act in relation to this


publication may be liable to criminal prosecution and civil
claims for damages.
First published 1993 by
THE MACMILLAN PRESS LTD
Houndmills, Basingstoke, Hampshire RG21 2XS
and London
Companies and representatives
throughout the world

ISBN 978-0-333-56513-1 ISBN 978-1-349-12690-3 (eBook)


DOI 10.1007/978-1-349-12690-3
A catalogue record for this book is available
from the British Library
Contents

Preface viii

1 Basic Concepts in Thermodynamics


1.1 Introduction 1
1.2 Some Definitions 1
1.3 The First Stages in Problem-Solving With Closed Systems 3
1.4 Worked Example 4

2 Work
2.1 Definition of Work 5
2.2 Problem-Solving 6
2.3 Electrical Work 6
2.4 Worked Examples on Electrical Work 6
2.5 Work to Change the Volume of a Closed Hydrostatic System 8
2.6 Worked Examples on Pressure-Volume Work 8
2.7 Work to Change the Length of an Elastic Rod 12
2.8 Worked Examples on Work and Elastic Deformation 12
2.9 Exercises 14

3 Gases and the Ideal Gas Scale of Temperature


3.1 Boyle's Law 15
3.2 Ideal Gases 16
3.3 Problem-Solving 16
3.4 Worked Examples on Boyle's Law 16
3.5 The Ideal Gas Scale of Temperature and the Ideal Gas Equation 20
3.6 Worked Examples on the Ideal Gas Equation 21
3. 7 Exercises 25

4 The First Law of Thermodynamics


4.1 Internal Energy 26
4.2 The First Law of Thermodynamics 27
4.3 Problem-Solving 27
4.4 Worked Examples on the First Law of Thermodynamics 27
4.5 Reversible Processes 29
4.6 Worked Examples on Reversible Processes 30
4. 7 Heat Capacities 31
4.8 Worked Examples on Heat Capacities 33
4.9 Exercises 42

5 The Second Law of Thermodynamics


5.1 The Clausius Statement of the Second Law 43
5.2 Heat Engines 43
5.3 Worked Examples on Heat Engines 44
5.4 Kelvin's Statement of The Second Law 48

v
5.5 Carnot's Theorem and its Corollary 48
5.6 Universal Temperatures 49
5.7 Thermodynamic Temperature 49
5.8 Worked Examples on Heat Engines and Thermodynamic Temperature 50
5.9 Celsius Temperature 53
5.10 Problem-Solving 53
5.11 Worked Examples on Heat Engines and the Second Law of Thermodynamics 53
5 .12 Exercises 60

6 Entropy
6.1 The Inequality of Clausius 61
6.2 Entropy 61
6.3 Worked Examples on Entropy 62
6.4 Entropy and Work 66
6.5 Worked Examples on Entropy and Work 67
6.6 The Entropy Form of the First Law 68
6.7 Worked Examples on the Entropy Form of the First Law 69
6.8 Entropy and Irreversible Processes 71
6.9 Worked Examples on the Law of the Increase of Entropy 71
6.10 Maxwell's Relations 75
6.11 Some Useful Mathematical Relations 75
6.12 Problem-Solving 75
6.13 Worked Examples on Entropy 76
6.14 Exercises 81

7 Thermodynamic Potential Functions


7.1 The Helmholtz Function (Helmholtz Free Energy) 82
7.2 Worked Examples on the Helmholtz Function 83
7.3 The Gibbs Function (Gibbs Free Energy) 86
7.4 Worked Examples on the Gibbs Function 87
7.5 Useful Work and Availability 90
7.6 Worked Examples on Availability 91
7. 7 Exercises 100

8 Some Simple Thermodynamic Systems


8.1 Closed Hydrostatic Systems 101
8.2 Worked Examples on Closed Hydrostatic Systems 102
8.3 Ideal Gases 111
8.4 Worked Examples on Ideal Gases 112
8.5 Perfectly Elastic Solids 119
8.6 Worked Examples on Perfectly Elastic Solids 120
8.7 Voltaic Cells 126
8.8 Worked Example on Voltaic Cells 127
8.9 Exercises 128

9 Heat Capacities and Equations of State


9.1 Introduction 130
9.2 Heat Capacities 130
9.3 Worked Examples on Heat Capacities 131
9.4 Equations of State 139
9.5 Worked Examples on Equations of State 141
9.6 Exercises 145

10 Phase Changes
10.1 The Enthalpy of a Phase Change (Latent Heat) 147
10.2 Worked Examples on the Enthalpy of a Phase Change 148
10.3 The Clapeyron-Clausius Equation 153
vi
10.4 Worked Examples on the Clapeyron-Clausius Equation 154
10.5 The Equation of Clausius 157
10.6 Worked Example on the Equation of Clausius 158
10.7 Exercises 159

11 The Third Law of Thermodynamics


11.1 The Third Law of Thermodynamics 161
11.2 Worked Examples on the Third Law 161
11.3 Planck's and Simon's Versions of the Third Law 164
11.4 Worked Example on Absolute Entropy 165
11.5 Allotropic Transformations 165
11.6 Worked Example on Allotropic Transformations 166
11.7 Exercises 167

12 Irreversible Processes
12.1 Irreversible Processes and Thermodynamics 168
12.2 The Joule Process 168
12.3 Worked Examples on the Joule Process 169
12.4 The Joule-Thomson Process 173
12.5 Worked Examples on the Joule-Thomson Process 174
12.6 Exercises 182

13 A Simple Kinetic Theory of Gases


13.1 A Simple Kinetic Theory of Gases 183
13.2 Worked Examples on a Kinetic Theory of Gases 185
13.3 Exercises 190

14 Heat Transfer
14.1 Introduction 191
14.2 Heat Conduction 191
14.3 A General Approach to Solving Heat Conduction Problems 192
14.4 Worked Examples on Heat Conduction 193
14.5 Heat Convection 197
14.6 Worked Examples on Heat Convection 198
14.7 Thermal Radiation 202
14.8 Worked Examples on Thermal Radiation 204
14.9 Exercises 208

Recommended Reading 210


Index 211

vii
Preface

Thermodynamics is one of the major subjects of classical phenomenological physics, a


subject of great power and beauty. Nevertheless, it is, for many students, a difficult
subject and one that they do not understand on a first (and often, only) reading. To them
the subject seems to be a collection of subtle concepts, linked by countless equations with
no underlying framework.
Despite its forbidding nature, thermodynamics is a subject with a wide range of
applications and is a vital component in the education of physical scientists. However,
the lack of understanding so often encountered leaves the student with a gap between
wanting to able to use the theory and being able to do so. One way of promoting
understanding and bridging this gap is through problem-based learning. The aim of this
approach is to make students 'doers of the word, not hearers only' by providing detailed
solutions to a carefully selected range of problems, showing how principles and concepts
may be applied to particular situations, and then offering the student situations that differ
slightly but which can be tackled by an extension of the approaches that have been used
in the solved problems.
Learning how to use the theory of thermodynamics through problem-solving is the
approach followed in this book, which is aimed at undergraduates in the physical sciences
and in engineering taking a first course in thermodynamics or thermal physics. The book
starts with a summary of the important basic concepts in thermodynamics and establishes
the basic vocabulary and outlook. The core topics in classical thermodynamics are then
examined in a series of chapters that include a brief introduction to each topic, where
important results are stated and, sometimes, derived, followed by a number of examples
worked out in detail.
One of the powerful features of classical thermodynamics is that it provides rela-
tionships between the properties of a system. Consequently, only a small number of
properties need to be 'explained' at the atomic level. In the simplest treatments this is
done under the heading of kinetic theory and this book contains a chapter on a simple
kinetic theory of gases. Also important is the rate of energy transfer between a system
and its surroundings, so the final chapter deals with the very practical topic of heat
transfer.
The book assumes a knowledge of elementary calculus, including partial differentia-
tion, and SI units are used throughout.

Michael Sprackling
January 1993

viii
Topic Guide

1 Basic Concepts in
1.1 Introduction
1.2 Some Definitions
1.3 The First Stages in
Problem-Solving wit11
Closed Systems
1.4 Worked Example
Thermodynamics

1.1 Introduction The essence of the thermodynamic approach to the description of the behaviour of
matter is to take a particular region of space and its contents and to consider the transfer
of energy across the boundary separating this region from its environment. To put this
simple programme into practice it is necessary to examine the kind of description that
can be simply given of an assembly of matter and to determine the laws that govern the
transfer of energy across the boundary of that assembly of matter. This chapter contains
a number of definitions and statements that introduce the basic terminology of thermo-
dynamics and indicate the nature of the description that is given.

1.2 Some SYSTEM


Definitions A system is the region of space under consideration and the matter that it contains. When
a system is constrained to be of constant mass it is called a closed system; when it is able
to vary its mass it is an open system; and when it is unable to interact with any other part
of the universe it is an isolated system.
BOUNDARY
The boundary of a system is the interface between the system and the rest of the
universe. The boundary is always conceptual, though it may coincide with a real, physical
surface.
SURROUNDINGS
The remainder of the universe outside the system is termed the surroundings of the
system. In practice, the term is restricted to those parts of the environment that have a
direct effect upon the behaviour of the system.
THERMODYNAMIC COORDINATE or PROPERTY
The behaviour of a system is described in terms of large-scale properties of the system,
such as pressure and volume. These quantities describe the system as a whole and are
often called thermodynamic variables or thermodynamic properties. The name thermo-
dynamic coordinate (or, simply, coordinate) is preferred here.
Coordinates, such as volume, that are proportional to the mass of the system are called
extensive coordinates. Coordinates that have an essentially local nature, such as pressure,
are called intensive coordinates.
The coordinates necessary to describe a system must be determined by experiment.
Coordinates whose values are directly measurable are known as primitive coordinates.
If it is possible to vary the magnitude of any coordinate by a finite amount while the
value of another coordinate remains constant, the two coordinates are said to be
independent. The number of independent coordinates needed to describe a given system
is called the number of degrees of freedom of the system.
STATE
The condition of a system, identified by a particular set of values of the coordinates, is
called the state of the system. For this reason thermodynamic coordinates are sometimes
called state variables or state functions.

1
EQUILIBRIUM STATE
For the coordinates to be descriptive of the system as a whole they must be well defined,
and the intensive coordinates must be uniform throughout the system. The system will
then be in a state in which the values of the coordinates are independent of time. The
system is then said to be in thermodynamic equilibrium, and the state is described as an
equilibrium state.

PROCESS
When a system in an initial equilibrium state is made to change to a different equilibrium
state it is said to undergo a process.
If the process is to be described by the thermodynamic coordinates of the system, the
system must, at all stages of the process, be infinitesimally close to a state of thermodyna-
mic equilibrium; that is, the process must consist essentially of a sequence of equilibrium
states. Such a process is called a quasistatic process.

PATH
The path of a process is the complete sequence of equilibrium states, specified by the
values of the coordinates, through which a system passes during a quasistatic process.
When a process is not quasistatic the coordinates are not descriptive of the process and
the path is indeterminate.

INTERACTION
The term interaction is used to describe the transfer of energy across the boundary of a
system.
The interaction is described as a work interaction when the energy transfer is the result
of an unbalanced force existing between the system and its surroundings.
The term thermal interaction is used to describe the energy transfer that takes place in
the absence of unbalanced forces.
An open system allows the possibility of mass transfer across the boundary. When this
occurs it will be described as a chemical interaction, even when no chemical reaction as
such takes place.
A boundary that allows a thermal interaction to occur is a diathermic boundary. A
boundary that does not allow a thermal interaction to occur (a theoretical extrapolation
of what is found by experiment) is an adiabatic boundary. Diathermic and adiabatic
boundaries that are not ideally rigid allow a work interaction to occur.
(Work can still be done on a system with ideally rigid boundaries if the work is done
electrically; see Chapter 2.)

CONDITIONS OF EQUILIBRIUM
The condition of thermodynamic equilibrium is a very stringent condition and has three
aspects, each of which must be satisfied:

(a) The system must be in thermal equilibrium; that is, there must be no thermal
interaction.
(b) The system must be in mechanical equilibrium; that is, there must be no unbalanced
forces between the system and its surroundings, or between different parts of the
system.
(c) The system must be in chemical equilibrium; that is, there must be no net mass
transfer across the boundary and no change in the macroscopic structure of the
system.

TEMPERATURE
The condition of thermal equilibrium is found to depend on the relative hotness of the
system and its surroundings. It is a common experience that when thermal contact
between two systems A and B causes A to become colder and B to become hotter,
wherever the contact is made, there is no process that can cause A to become hotter and
B to become cooler that does not involve a work interaction. This generalisation is one

2
statement of the second law of thermodynamics, and it follows from it that if system A is
hotter than system B and system B is hotter than system C, then system A must be hotter
than system C. When thermal equilibrium is reached, so that system A is in thermal
equilibrium with system B and system B is in thermal equilibrium with system C, then
system A will be in thermal equilibrium with system C. This result is often known as the
zeroth law of thermodynamics: when two (closed) systems are separately in thermal
equilibrium with a third system they are in thermal equilibrium with each other.
Using the zeroth law it is possible to trace out a path for any chosen system in which
each state of the system is in thermal equilibrium with one chosen state of a standard
system. Such a path is known as an isotherm.
The common property of the systems that ensures that they are in thermal equilibrium
is called temperature. Therefore, the temperature of a system is that property which
determines whether or not the system is in thermal equilibrium with other systems.
The temperature of all systems in mutual thermal equilibrium may be represented by a
single number. To number the isotherms of the standard system (which is known as a
thermometer) requires the adoption of a set of rules for assigning the numbers. This
procedure is termed establishing a scale of temperature. The condition for two systems to
be in thermal equilibrium is then that they have the same temperature.

EMPIRICAL TEMPERATURE
This is the name given to the result of the arbitrary labelling of the isotherms of the
system chosen as the standard for the numerical specification of temperature. The only
necessary condition that the labelling process must satisfy is that each isotherm should
have a unique numerical value. In practice, this means that a monotonic function is
chosen to relate empirical temperature to the magnitude of the chosen quantity that
varies with change in temperature.

EQUATION OF STATE
An equation of state is the relation between the independent coordinates needed to
specify equilibrium states of a system and the empirical temperature. For a system
consisting of a fixed mass of gas, the equilibrium states are specified by the values of the
pressure p, the volume V and the empirical temperature 8.
Therefore, the equation of state may be written in the form

f (p,V,8) = 0

where f is a function that must be determined experimentally using the chosen empirical
temperature scale.
Another important simple system that will be considered frequently in this book is the
elastic rod. The equilibrium states of such a system are described by the coordinates
length L, load F and empirical temperature 8.
The equation of state may be written

<t>(F,L,8) = 0

where <t> is a function that must be determined experimentally.

1.3 The First The discussion in this chapter has indicated what the first steps should be when
Stages In Problem- approaching a problem involving the thermodynamic behaviour of a closed system.
These steps are:
Solving With Closed
Systems 1. Draw a diagram of the system and its surroundings, defining clearly the boundary of
the system and specifying the contents.
2. As an aid to understanding the behaviour of the system draw an appropriate coordi-
nate diagram. Label the various parts of the process and indicate the nature of each
part.

3
1.4 Worked
Example Let the system be a fixed mass of gas contained in a cylinder by means of a well-fitting, frictionless
piston. The equilibrium states of this system are characterised by the pairs of values of the pressure
p and the volume V. When this system undergoes a quasistatic isothermal expansion from an
Cylinder
equilibrium state ito an equilibrium state f, steps 1 and 2 above would appear as in Figs. 1.1 and
1.2, respectively. In Fig. 1.1 the boundary of the system is shown by the dashed line.

Figure 1.1 A system


consisting of a fixed mass of
gas contained in a cylinder by
a well-fitting, frictionless
piston.

~. II

Figure 1.2 The coordinate


diagram for a fixed mass of
gas undergoing a quasistatic
isothermal process from an
equilibrium state i to an
equilibrium state f

4
Topic Guide
2.1
2.2
Definition of Work
Problem-Solving
2 Work
2.3 Electrical Work
2.4 Worked Examples on
Electrical Work
2.5 Work to Change the Volume
of a Closed Hydrostatic
System
2.6 Worked Examples on
Pressure-Volume Work
2. 7 Work to Change the Length
of an Elastic Rod
2.8 Worked Examples on Work
and Elastic Deformation
2.9 Exercises

2.1 Definition of Thermodynamics is concerned with the interactions that take place across the boundary
Work between a system and its surroundings and the consequent changes in the state of the
system. These interactions are best specified in terms of the energy transfer that occurs
across the boundary. One type of interaction is the work interaction, and this will be
considered in this chapter.
Work in the Newtonian sense is done when a force moves its point of application in its
own direction. When a force F , acting at a point with position vector r , moves its point of
application by dr it is said to do work of amount F·dr.
In particular, when a body of mass m is raised at a constant velocity through a height h
in the gravitational field , the work done on the body is mgh, where g is the acceleration
of free fall , and the energy of the body increases by mgh .
The work interaction between a system and its surroundings may, therefore, in
principle, be measured by the displacement, relative to some reference level, of a weight
acting on the system through an arrangement of ideal pulleys and strings.
However, care must be taken to allow for possible non-work (that is, thermal)
interactions, so that a pure work interaction is defined in the following way.
The interaction between a system and its surroundings (which could be another
system) is a pure work interaction if what happens in both system and surroundings could
be repeated while the sole effect external to each is the change in height of a weight
above a reference level.
A sign convention is needed when considering a work interaction between a system
and its surroundings. Work done on a system is counted positive, as it represents a
transfer of energy to the system. When using any sign convention, the direction of energy
transfer in an interaction should be shown in a diagram with the arrow in the conven-
tional positive sense. If the energy transfer is in the opposite sense and is known, the
arrow should still be in the conventional positive sense but the value should be given a
negative sign. For example, a general work interaction W between a system A and its
surroundings B is always shown as in Fig. 2.1(a) . However, an interaction in which the
system A did 200 J of work on the surroundings B would be shown as in Fig. 2.1(b) and
would be described by saying that the work done on the system A is -200 J.

Surroundings B Surroundings B

Figure 2.1 The sign


convention for the work
interaction.

5
2.2 Problem- The first stages in solving problems relating to closed systems are now:
Solving
1. The system and its surroundings must be clearly identified and, where it is helpful, a
diagram of the system and its surroundings should be drawn.
2. As an aid to understanding the behaviour of the system, an appropriate coordinate
diagram may be drawn. Label the various parts of the process and indicate the nature
of each part.
3. If a work interaction is identified it should be marked on the diagram of the system,
with an arrow indicating the conventional positive direction of energy transfer,
together with the magnitude and sign of the energy transfer, if that is known.

At this stage three examples of a work interaction will be examined in detail. Some
others will be considered later in this book.

2.3 Electrical Work In a region of space where there is an electrostatic field the work W that must be done by
an external force to move a charge Q (understood not to affect the existing field) between
two points A and B in the field is given by

V= W!Q

where Vis defined as the potential difference between the points A and B. The point B is
at the higher potential when W is positive.
A convenient method of doing work on a system is to incorporate a resistor as part of
the system and pass a current through the resistor. Work must be done in making the
current flow through the resistor and, therefore, work is done on the system. When a
current I flows under a potential difference V through an ohmic resistor of resistance R
for a timet, the work W done on the system is given by

W = IVt = JZRt = V 2t/R (2.1)

2.4 Worked
Examples on
Electrical Work 2.1 Show that the work needed to place a charge Q quasistatically on an initially uncharged isolated
sphere of radius R in free space is given by Q 2 /8rtE 0 R, where E0 is the permittivity of free space.

Solution Let the charge on the sphere at any stage of the charging process be q. By symmetry, this charge
will be distributed uniformly over the surface of the sphere and, to charges outside the sphere, this
charge will behave as if it were concentrated at the centre of the sphere, provided that the
interacting charge does not significantly disturb the symmetry of the charge distribution on the
sphere. Let an infinitesimal charge dq be brought from infinity to the surface of the sphere. When
this charge dq is at a distance r from the centre of the sphere it experiences a repulsive (positive)
force of magnitude q dq/4n:E 0 r 2 • To maintain this charge dq in equilibrium at this position an
external force F equal to- q dq r/4n:E 0 r 2 must be applied, where r is a unit vector in the direction of
the positive force. The work done on the charge dq when the force F moves its point of application
a distance dr is F·dr. This is a well-defined infinitesimal quantity, provided that the charge dq is
always infinitesimally close to a state of (mechanical) equilibrium, that is, provided that the process
is quasistatic. Then, the work w that must be done on the charge dq to move it a distance dr is given
by

and the work that must be done on the charge dq to bring it from infinity to the surface of the
sphere is given by

w = -q dq/4n:E 0 ( (llr 2 ) dr = (q dq/4n:E 0 ) [1/r]~


= q dq/4n:EJ?

6
r
Therefore, the work W needed to place quasistatically a charge Q on the surface of a sphere of
radius R is given by

W = (1/4rtEoR) q dq = Q2 /8rtEoR
0

2.2 A cell having an e.m.f. of 1.5 V and an internal resistance of 2.0 n is connected in parallel with a
cell having an e.m.f. of 2.0 V and an internal resistance of 2.5 n. Calculate the power dissipated in
a resistor of resistance 3 n connected across the combination of cells.

The circuit diagram is as in Fig. 2.2. Kirchhoff's laws may be applied to this circuit. The first law
states that, at any junction, the algebraic sum of the currents is zero.
Therefore, if the currents are as shown in Fig. 2.2,

1 = 1, + 12 The second law states that, around any circuit,

Figure 2.2 Circuit diagram where E; is an e.m.f. and R; is a resistance. Both sums are algebraic; signs are allocated to E and I
defining the senses of the by choosing positive senses around each mesh, as shown in Fig. 2.2.
circuit meshes. From mesh 1:

1.5 (V) - 2.0 (V) = 11 X 2.0 (!1) - 12 X 2.5 (!1)

From mesh 2:

2.0 (V) = 12 X 2.5 (!1) + (11 + 12 ) X 3.0 (!1)

Therefore,

-0.5 = 2.0 Jl- 2.5 12 (2.2)

and

2.0 = 2.5 12 + 3.0 Jl + 3.0 12


= 3.0 /1 + 5.5 12 (2.3)

Multiply Equation (2.2) by 3 and Equation (2.3) by 2 to get

-1.5 = 6.0 J, - 7.5 12 (2.4)

and

4.0 = 6.0 Jl + 11.0 12 (2.5)

Subtract Equation (2.4) from Equation (2.5). Then

5.5 = 18.5 12

or

12 = 5.5/18.5 = 0.297 A

Then, from Equation (2.2)

- 0.5 = 2.0 11 - 2.5 X 0.297

giving

11 = 0.121 A

7
Therefore,

I = II + I2 = 0.418 A

and

WR = I 2R = (0.418f X 3.0
= 0.52 w

2.5 Work to Change A closed hydrostatic system is a system of constant mass that exerts a uniform hydrostatic
the Volume of a pressure on its surroundings. Examples of closed hydrostatic systems are pure liquids,
solids and gases, and also mixtures of different substances that may be in the same or in
Closed Hydrostatic different phases. The equilibrium states of a closed hydrostatic system are specified by
System the values of the pressure p, volume V and empirical temperature e, only two of these
coordinates being independent.
Consider a closed hydrostatic system contained in a cylinder by means of a well-fitting
frictionless piston, as in Fig. 2.3. Let the area of cross-section of the cylinder be A and
the distance from the closed end of the cylinder to the piston be x . In the absence of
friction the external pressure must equal the pressure p and the force acting over the
piston is pA. When the system is compressed by the piston moving an infinitesimal
0 X distance -dx, the work done on the system is -pAdx, provided that p remains sensibly
constant during this infinitesimal process. Since the change in volume dV is given by
Figure 2.3 A fixed mass of
gas contained in a cylinder by dV = Adx, an alternative expression for the work done on the system is -pdV.
means of a well-fitting piston. When the system undergoes a finite change in volume from Vi to Vt the work W done
on the system is given by

W=- fVf

Vj
pdV (2.6)

Equation (2.6) shows that p must be known as a function of V during the process; that is,
the work interaction depends on the path, and this implies that pis well-defined during
the process. Pressure gradients in the system will be vanishingly small only if the piston
moves sufficiently slowly. Therefore, the process must be quasistatic.
Work is done on a closed hydrostatic system when it undergoes a change in volume.
Such work is often called displacement work or pressure-volume work. From an en-
gineering viewpoint, displacement work is usually considered to be useless work, since,
when done by the system, its only effect external to the system is to push back the
surroundings. However, in addition to the coupling to the uniform surroundings, a
system can also be coupled to a second system, which will be called the body, and which
is usually treated as thermally isolated. The system can then do work on the body in
addition to the displacement work. Work done on the body is termed useful work.

2.6 Worked
Examples on
Pressure-Volume
Work 2.3 A mass of gas at a pressure pis contained in a rigid copper sphere of volume V that is connected by
a short tube, fitted with a stopcock, to a similar tube that is evacuated. Take the gas as the system
and calculate the work done on the system as it expands into the second sphere when the stopcock
is opened.

Solution The system is the gas, but the boundary of the system must be the inner surface of the first sphere
before the stopcock is opened and the inner surface of the two spheres and the connecting tube
after the stopcock has been opened. Any interaction between the system and its surroundings must
take place across the boundary of the system; that is, across the inner surface of the two rigid
copper spheres. As the spheres are rigid, there is no change in the volume offered to the gas by the

8
two spheres as the expansion of the gas takes place. Consequently, there is no displacement (or
pressure-volume) work. As this is the only source of a work interaction, there is no work done on
the gas during its expansion.

2.4 A mass of gas contained in a cylinder by a well-fitting, frictionless piston undergoes a change of
state from an equilibrium state ito an equilibrium state f by two quasistatic processes (a) and (b).
In process (a) the constraints on the system are such that the pressure p of the gas and the
volume V occupied by the gas are related by the equation

p (Pa) x V (m3 ) = 2000 (J)

For the second process the relation between p and Vis

p (Pa) = 2.25 X 105 - 2.5 X 106 V (m 3 )

Calculate the work done on the gas in the two processes when Vi = 0.01 m 3 , Vf = 0.08 m3 and
Pi = 2.0 x 105 Pa.

Solution The system is the constant mass of gas contained in the cylinder. Using either of the path
equations, the value of Pf is 25 000 Pa. For example, using the first equation:

gives

Pf = 25 000 Pa
The work w done on the system in any infinitesimal part of a quasistatic process in which the
volume increases by dV, while the pressure p remains effectively constant, is given by

w = -pdV

-rf
In a finite quasistatic process the work W done on the system is given by

W= pdV
Vi

and, for this integral to be evaluated, p must be known as a function of V. For process (a) the
relation between p and V is

p (Pa) x V (m 3 ) = 2000 ( J)

Therefore,

W =- f Vi
Vf
(2000/V) dV

= -2000 ln (Vf/Vi)
= - 2000 ln [0.08 (m 3 )/0.0l (m3 )]
= -2000ln8 = -4158.91
= -4200 J
The sign of W is negative, indicating that the work is done on the surroundings by the system.
For process (b), the relation between p and Vis

p (Pa) = 2.25 X 105 - 2.5 X 106 V (m 3 )

so that

W= - f Vf

Vi
(2.25 X 105 - 2.5 X 106 V) dV

9
= - [2.25 X 105 V - (2.5 X 106 V 2/2)]g:g~
= -7800 J

Again, W is negative, showing that the work is done by the system on the surroundings.

2.5 An evacuated cylinder, with rigid diathermic walls and having a volume of 0.87 m3 , is fitted with a
stopcock and placed in an atmosphere where the pressure is 1.0 x 105 Pa. When the stopcock is
opened slightly, gas leaks slowly into the cylinder. Calculate the work that must be done by the
atmosphere in making the pressure inside the cylinder equal to the atmospheric pressure.

Solution The system is the gas that enters the cylinder when the stopcock is slightly opened and which brings
the pressure in the cylinder up to p 0 , the pressure of the atmosphere. When this gas is part of the
atmosphere it is convenient to imagine it contained in a notional cylinder, fitted with a well-fitting,
frictionless piston, and shown dotted in Fig. 2.4.

Figure 2.4 The system and


its boundary immediately after
the opening of the stopcock.

Before the stopcock is opened the boundary of the system coincides with the inner surface of this
notional cylinder and piston. Immediately after the slight opening of the stopcock the boundary
enlarges to include the volume of the notional cylinder and that of the real cylinder into which the
gas is made to flow. The boundary of the system is shown by the dashed line in Fig. 2.4. When the
stopcock is slightly opened, the gas constituting the system is pushed slowly into the cylinder by the
atmosphere, simulated by the movement of the notional piston, which moves under the constant
pressure p 0 of the atmosphere. The process takes place at constant temperature because the piston
moves slowly and the walls of the cylinder are diathermic. Therefore, the volume V 0 of gas initially
in the notional cylinder must be equal to the volume of the real cylinder. The work w done on the
system by the surroundings, that is, the atmosphere, during an infinitesimal part of the process is
given by

and, in the finite process, the work W done on the system is given by

W = -pofvf dV
Vj

where Vi is the initial volume occupied by the system immediately after the opening of the stopcock
and Vf is the final volume occupied by the gas when the pressure in the cylinder has reached p 0 •
From the above discussion it follows that

Therefore,

or

Inserting the numerical values into this equation gives

W = 1.0 X lOS (Pa) x 0.87 (m3 )


= 87 kJ
The sign of W is positive, as the work is done by the surroundings on the system.

10
2.6 A fixed mass of gas is contained in a cylinder by means of a well-fitting, frictionless piston. The
cylinder also has a small resistor in it. Starting from an equilibrium state A the gas first has its
pressure increased from p A to PB at constant volume VA as the result of a current I being passed
through the resistor of resistance R for a time t. The current is then switched off and the gas is
allowed to expand in such a way that its pressure p and volume V are related by the equation

pV = constant

until its volume reaches the value Vc and its pressure Pc is equal to PA·
Calculate (a) the total amount of work done on the gas and (b) the pressure-volume (or
displacement) work done on the gas.

Solution The system is the mass of gas and the small resistor contained in the cylinder and piston
arrangement, as shown in Fig. 2.5. The boundary of the system is shown by the dashed line. The
two stages of the process are shown on the pressure-volume diagram of Fig. 2.6.
Cylinder

System
Figure 2.5 A mass of gas
and a small resistor contained
in a cylinder having a well-
fitting, frictionless piston. Boundary

Figure 2.6 The two stages of


the process shown on the
pressure-volume diagram. v
In the process AB the volume of the system remains constant and, therefore, no pressure-
volume work is done on the system. However, electrical work We is done on the system, given by

In the process BC there is no electrical work done on the system as I is zero, but there is a
volume change so that pressure-volume work WP is done on the system. This work is given by

wp =- f vc

VB
pdV

where p is the pressure exerted by the gas when it occupies a volume V .


In the process BC the relationship between p and V is

p V = constant = K

say. Then,

Now

so that

(2.7)

Equation (2.7) gives the pressure-volume work done on the system. Since VB is less than Vc,
ln(Vc/VB) is positive so that WP is negative, indicating that the work is done on the surroundings by
the system.
The total work done on the system is We + WP, given by

11
(2.8)

If it is assumed that a negligible amount of the energy transfer across the boundary of the system
goes to changing the energy of the resistor, Equation (2.7) gives the pressure-volume work done
on the gas and Equation (2.8) the total amount of work done on the gas.

2. 7 Under a constant pressure the melting of ice to water takes place at a constant temperature. Under
a pressure of 1.01 x 105 Pa and at room temperature the density of ice is 916.23 kg m- 3 while,
under the same conditions, the density of water is 999.84 kg m- 3 • Calculate the work done by the
atmosphere exerting a pressure of 1.01 x 105 Pa on a system consisting of 20.0 kg of ice as it melts
to water at room temperature.

Solution Since
density = mass/volume

the volume Vi of 20.0 kg of ice at room temperature and under a pressure of 1.01 x 105 Pais equal
to 20.0 (kg)/916.23 (kg m - 3) while the volume Vt of 20.0 kg of water at the same temperature and
pressure is 20.0 (kg)/999.84 (kg m- 3 ).
The atmosphere exerts a constant pressure p 0 so that the work W done on the system in the
process of melting is given by

Therefore,

W = -1.01 x 105 (Pa) [(20.0/998.84) (m 3 ) - (20.0/916.23) (m3 )]


= -1.01 x 105 (2.0003 x 10-z - 2.1828 x 10- 2 )

= -1.01 X 105 (- 0.1825 X 10- 2 )


= 184.3 J

2. 7 Work to Change The equilibrium states of an elastic rod are specified by the values of the length L, the
load F and the (empirical) temperature 8. Then, when the length of the elastic rod is
the Length of an increased by the infinitesimal amount dL, while the load remains sensibly constant, the
Elastic Rod work w done on the rod is given by

w = FdL

In a finite process, in which the length of the rod changes from Li to Lt the work W done
on the system is given by

w = fLf F dL (2.9)
Li

Equation (2.9) confirms that the work done on the system depends on the path. For the
elastic rod the relation between F and L must be known. F and L are only defined for
equilibrium states of the rod and, therefore, the process must be quasistatic. Further, F
and L must be uniquely related and, therefore, the material must be perfectly elastic,
though not necessarily Hookean, showing no elastic hysteresis.

2.8 Worked
Examples on Work
and Elastic
Deformation 2.s A cylinder of a certain ideal elastic material has a length L when it is under a load F at a
temperature 8. At constant temperature F and L are related through the equation

12
FIA = E (L - L 0 )1L0

where L 0 is the unstretched length of the cylinder, A is its area of cross-section and E is a function
of temperature only. If the value of Eat the temperature considered is 9.1 x 1011 Pa, determine
the amount of work that must be done on a rod of this material of unstretched length 10.0 em and
diameter 1.00 em to extend it quasistatically and isothermally by 0.010 em.

Solution The system is the rod of ideal elastic material and the state is specified by the values of L,F and e.
At constant temperature F and L are linearly related, as shown in Fig. 2.7. The process is the
quasistatic extension of the rod. When the rod has a length L under a tension F, the extension of
the rod by an infinitesimal amount dL, the load remaining essentially constant, requires the

'I~~~~.
performance on the rod of an amount of work w, given by

L0 dL L
w = FdL

= [AE(L - L 0 )1L0 ] dL
Figure 2. 7 The relationship
between L and F for a linear w is represented by the area of the element shown shaded in Fig. 2.7.
elastic material at constant In a finite quasistatic process, in which L changes from Li to Lt, the work done on the rod is W,
temperature.
given by

W = (AEIL 0 ) fLf (L -
Lj
L 0 ) dL = (AEIL 0 ) [L 2 /2- L 0 L]ff

= (AEIL 0 ) (Li/2 - LtLo - Lf/2 + LoLi)


= (AE/2L 0 ) [Li - Lf - 2Lo(Lt- Li)]

When Li = L 0 the expression for W simplifies to


W = (AE/2L 0 )(Lj - 2LoLt + L~)
= (AE/2L 0 )(Lt - L 0) 2

Using the values given

W = -21 (rc (0.1) 2 ( m2 ) /4 ] x 9.1 X 1011 (Pa) X (0.0001) 2 (mf


0.1 (m)
= 3.57 J
2.9 A cylinder of a certain ideally elastic material has a length L when it is under a load F at a
temperature e. The equation relating F, Lande is
F = Ke(L - L 0 ) 112
where L 0 is the unstretched length of the cylinder and K is a constant.
Compare the work that must be done on the cylinder to extend it under quasistatic, isothermal
conditions from a length L 0 to a length 1.5 L 0 with that needed to extend it under the same
conditions from a length 1.5 L 0 to a length 3.0 L 0 •

Solution The system is the cylinder of ideally elastic material. If the cylinder has a length L under a load F,
the work w done on the system when the length is increased by an amount dL, with F remaining
effectively constant, is given by

w = FdL
so that, in a finite quasistatic process, the work W done on the cylinder is given by

W= fLf FdL
Lj

where Li is the initial length of the cylinder and Lt is its final length. For a quasistatic isothermal

rt
extension of the cylinder the expression for W becomes

W = Ke (L - L 0 ) 112 dL
Lj

= (2Ke/3) [(L - L 0 ) 312]ft


1

13
In process (a), L; = L 0 and Lf = 1.5 L 0 , so that

W(a) = (2K8/3)[(1.5 L 0 - L 0 ) 3n - (L 0 - L 0 ) 3n]


= (2K8/3)(0.5 L 0 ) 3n
= (2K6/3)L~ X 0.3536

In process (b), L; = 1.5 L 0 and Lf = 3.0 L 0 , giving

W(b) = (2K8/3)[(3L0 - L 0 ) 312 - (1.5L 0 - L 0 ) 3n]


= (2K6/3)L~12 X 2.4744

Therefore,

W(a)/W(b) = 2.4744/0.3536 = 7

2.9 Exercises
2.1 Show that the work W needed to charge a capacitor of result of a current being passed through the resistor. This is
capacitance C to a state where the potential difference between equilibrium state B.
the plates is V is given by The gas is then allowed to expand quasistatically in such a way
that its pressure p and volume V are related by the equation

p Vl. 4 = constant
2.2 The current I through a certain conducting solid sample in
the form of a cylindrical rod when the potential difference be- until the pressure reaches a value of 1.0 X 105 Pa. This is
tween the ends of the rod is V is given by equilibrium state C. Finally, the gas is compressed at constant
pressure until its volume has returned to a value of 2.0 x 10- 3
/(A) = 0.3(V(V)) 3 m3 • Calculate the pressure-volume work done on the gas in this
cyclic process.
The rod is connected in series with a resistor and a cell with an [+ 128 J in the process CA; -180 J in the process BC]
e.m.f. of 6.0 V and negligible internal resistance. Calculate the
value of the series resistance when the power dissipated in the 2.6 A certain gas obeys the equation of state
rod is twice that dissipated in the resistor. [0.10 0]
p(Vm- b)= K6
2.3 A certain closed hydrostatic system has the equation of
state where V m is the molar volume under a pressure p when the
empirical temperature is 8; b and K are constants. If b has the
pV~ = K8 value 30 x 10- 6 m 3 mol- 1 , calculate the work done on a system
consisting of two moles of molecules of this gas when it changes
where K and q are constants and e is the temperature measured from being in an equilibrium state with f equal to 1.01 x 105 Pa
on some empirical scale and V m is the molar volume. Show that and molar volume equal to 24.7 x 10- m3 to one in which the
the work W done on n moles of the system in a quasistatic, pressure is 0.50 x 105 Pa, by means of a quasistatic isothermal
isothermal expansion in which the volume occupied by the sys- process. [ -1800 J]
tem changes from V; to V£ is given by
2. 7 A certain ideal elastic material is such that a uniform wire
W = - (pfV£- p;V;)/(1 - q) made of it extends by 1.00 mm when the applied load is in-
creased by 200 N at constant temperature. If the unstretched
length of the wire is 2.000 m, calculate the work that must be
2.4 A cylinder fitted with a stopcock and having rigid, diather- done on the wire to extend it quasistatically from a length of
mic walls, contains gas at a high pressure p. When the stopcock 2.001 m to a length of 2.002 m. [0.3 J]
is opened slightly, gas leaks into the atmosphere, where the
pressure is p0 • Calculate the work done on the gas in the cylinder 2.8 When a certain polymeric material is in the form of a rod
as it leaks out to leave the pressure in the cylinder equal to Po· the length L of the rod under a tensile load F is given by
[If V 0 is the total volume occupied by the gas at atmospheric
pressure and Vc is the volume of the cylinder, W = -p 0 (Vo - F = K6[(LIL 0 ) - (LJLf]
Vc)]
where K is a constant, e is an empirical temperature and L 0 is the
2.5 A fixed mass of a certain gas is contained in a cylinder by unstretched length at temperature e. If the material behaves as
means of a well-fitting, frictionless piston. The cylinder has an ideal elastic material, derive an expression for the work that
adiabatic walls and also contains a small resistor connected to a must be done on the rod to change its length from 1.5 L 0 to 2.0
power supply outside the cylinder. Starting from an equilibrium L 0 in a quasistatic isothermal extension. [(17/24)K8L 0]
state A, in which the gas pressure is 1.0 x 105 Pa and the volume
occupied by the gas is 2.0 X 10- 3 m 3 , the pressure is increased to
2.0 x 105 Pa while the volume of the gas remains constant, as a

14
Topic Guide

3 Gases and the Ideal Gas


3.1 Boyle's Law
3.2 Ideal Gases
3.3 Problem-Solving
3.4 Worked Examples on
Boyle's Law
3.5 The Ideal Gas Scale of
Scale of Temperature
Temperature and the Ideal
Gas Equation
3.6 Worked Examples on the
Ideal Gas Equation
3.7 Exercises

3.1 Boyle's Law At sufficiently low pressures the behaviour of all gases is closely described by the
statement 'the volume V of a given mass of gas is inversely proportional to its pressure p,
provided that the temperature remains constant'. This statement is known as Boyle's law
and may be written

p V = constant (temperature and mass constant) (3.1)

Precise experiments show that at other than very low pressures Equation (3.1) needs
correction. At all pressures the behaviour of an arbitrary mass of any gas is represented
by the equation

pV =A' + B'p + C'p 2 + ... (3.2)

where A', B', C', ... depend on the mass, nature and temperature of the gas and A' >>
B' >> C' .... For a fixed mass of a given gas, A', B', C', ... are, therefore, functions
of temperature only.
Equation (3.2) shows that Boyle's law is more and more closely obeyed as the pressure
of the gas is reduced, so that an alternative statement of Boyle's law is
lim pV = F(8) (3.3)
p~o

where F is a function of the empirical temperature 8 only, for a fixed mass of a chosen
gas.
pV
The behaviour represented by Equation (3.2) is shown in Fig. 3.1. At high tempera-
tures, the product p V always increases asp increases, but at lower temperatures p V first
falls and then rises, passing through a minimum known as the Boyle point for the
j.....,---------11· particular isotherm. The locus of Boyle points is shown by the dashed line in Fig. 3.1. For
one isotherm the minimum occurs when p is zero and this isotherm shows an extended
range of pressure over which the product p Vis practically constant. The temperature of
this isotherm is known as the Boyle temperature eB, and is given by the condition

when (cJ(pV)/cJp] 8 =0 at p=O

Since the coefficient B' in Equation (3.2) is much larger than any other coefficient
p
except A', an alternative way of writing the condition for the Boyle temperature is
Figure 3.1 Isotherms for a
real gas. when B' = 0

When the system considered consists of 1 mole of molecules of gas, Equation (3.2)
may be written

p Vm = A + Bp + Cp 2 + ... (3.4)

15
where V m is the molar volume.
One mole is the amount of gas which contains as many molecules as there are atoms in
0.012 kg of 12C. Therefore, 1 mol of any gas contains the same number of molecules, a
number known as Avogadro's constant.
Experiment gives the remarkable result that, when 1 mol of molecules is taken as the
system, not only does p V m tend to the value A asp tends to zero, but A has the same
value for all gases at a given temperature. Therefore, coefficient A in Equation (3.4) is
independent of the nature of the gas and is a function of temperature only. The most
comprehensive statement of Boyle's law is, therefore,

lim pVm = /(8) (3.5)


p~o

where f(8) is a universal function of temperature.

3.2 Ideal Gases The behaviour of a real gas in the limit of vanishingly small pressures is described by
Equation (3.5). A model gas may be imagined that shows this behaviour at all tempera-
tures and pressures. Such a gas is termed an ideal gas and the behaviour of such a gas is
described by the equation

pVm = f(S) (3.6)

where f(8) is a universal function of temperature.

3.3 Problem- This chapter is examining the behaviour of a particular class of system, but the approach
Solving to problem-solving is essentially that of Chapter 2:

1. The system must be clearly identified and, where it is helpful, a diagram should be
drawn.
2. An appropriate coordinate diagram may be drawn. Note that Boyle's law applies only
to a system consisting of a constant mass of gas and that the shape of an isotherm for a
gas that obeys the law is a rectangular hyperbola in the graph of pressure against
volume.
3. If a work interaction is identified it should be marked on the diagram with an arrow
indicating the conventional positive direction of energy transfer.

3.4 Worked
Examples on
Boyle's Law 3.1 A certain mass of a gas in equilibrium occupies a volume of0.68 m3 at a certain temperature, when
it exerts a pressure of 3.0 X 105 Pa. The gas then undergoes a quasistatic isothermal expansion until
its volume is 1.26 m3 • Calculate the new equilibrium gas pressure and the work done on the gas in
the process.

Solution The system is a fixed mass of gas and the process is quasistatic and isothermal. Therefore, if it is
assumed that the gas is an ideal gas, the process is described by Boyle's law, so that

where Pi and Pt are the initial and final pressures, respectively, and Vi and Vt are, respectively, the
initial and final volumes occupied by the gas. The process is shown on the indicator diagram of Fig.
3.2 and the given values are Pi = 3.0 X 105 Pa; Vi = 0.68 m3 ; Vt = 1.26 m3 •
Therefore, applying Boyle's law,

16
p

P; --------,i

Figure 3.2 The coordinate


diagram for the expansion. v

so that

Pt = 3.0 X l<f X 0.68/1.26


= 1.6 X l<f Pa

If, at any arbitrarily chosen stage of the quasistatic expansion, the gas pressure is p, when the
volume occupied by the gas is V the work w done on the gas when the volume increases by the
infinitesimal amount dV with the pressure remaining sensibly constant is given by

w = -pdV

and is shown by the shaded area in Fig. 3.2


In the complete process the work W done on the gas is given by

W =- f pdV

To integrate this equation, p must be known as a function of V. Boyle's law may be written

p = KIV

where K is a constant for a fixed mass of gas at a constant temperature.


In particular,

Therefore,

J
Vf
W = -K dVIV = -K ln(Vt!VJ
Vj

and, substituting for K,

Inserting the given values gives

W = -3.0 x 105 (Pa) x 0.68 (m3 ) x ln[l.26 (m 3 )/0.68 (m 3 )]


= -1.3 X 105 J

The minus sign shows that the work is done by the gas on the surroundings.

3.2 A mercury barometer has a little air in the space above the mercury. The height of the mercury
column is 759 mm and the length of the tube above the column is 79.0 mm. When the tube is
lowered a little the height of the mercury column becomes 756 mm and the length of the space
above the mercury 49.0 mm. Calculate the atmospheric pressure in mm of mercury.

17
Solution Assume that the barometer tube is of uniform cross-section and that air under the pressures
involved can be treated as an ideal gas.
The system is the fixed mass of air trapped above the mercury. In all equilibrium states the
system has the same temperature, so that Boyle's law may be applied.
It is convenient to work in units of mm of mercury (mm Hg) for pressure and mm 3 for volume.
Let the atmospheric pressure be p 0 mm Hg and the area of cross-section of the barometer tube be
Amm2 •
In the first configuration (see Fig. 3.3(a)) the volume occupied by the trapped air is 79.0 x A
mm3 and the atmospheric pressure is supporting the pressure exerted by the trapped air plus the
pressure due to a column of mercury of height 759 mm. Therefore, the pressure exerted by the
(a) (b) trapped air in this situation is (p 0 - 759) mm Hg.
In the second configuration (see Fig. 3.3(b)) the volume occupied by the trapped air is 49.0 x
A mm3 and the pressure exerted by it is (p 0 - 756) mm Hg.
Applying Boyle's law gives
Figure 3.3 The two
equilibrium configurations of
the barometer tube. (p 0 - 759) (mm Hg) x (79.0 x A) (mm 3 ) (p 0 - 756) (mm Hg) X (49.0 X A) (mm 3)

Therefore,

Po X 79.0 - 759 X 79.0 = Po X 49.0 - 756 X 49.0

or,

Po= 764 mm Hg

3.3 Show that, if the Earth's atmosphere is treated as an ideal gas and the temperature of the
atmosphere is considered to be independent of altitude, the atmospheric pressure p must vary with
height h above the Earth's surface according to the equation

p =Po exp(-cgh)

where cis a constant, g is the acceleration of free fall (assumed independent of height above the
Earth's surface) and p 0 is the atmospheric pressure at the Earth's surface.

Solution Assume that the only variation of atmospheric pressure is in the direction normal to the Earth's
surface and place the origin of coordinates 0 on the Earth's surface, the direction vertically
upwards being taken as the positive x-direction.
Take as the system the air contained in a prism with vertical sides and area of cross-section A.
Consider the behaviour of that part of the system lying between (notional) horizontal planes at
heights x and x + dx above the Earth's surface, as in Fig. 3.4. At a height x above the Earth's
surface let the atmospheric pressure be p and the density of the atmosphere be p, while at a height
x + dx let the pressure be p + dp.
------------------,------,------------------- h

:p+dp:
, I ,

Figure 3.4 The variation of : p+ :: lx


::::::::::::::::::V/.0'&'~:::::::::::::::::::~ + dx
AreaA~
pressure with height in the
atmosphere. : : 0

Consider now the equilibrium of that part of the system lying between the horizontal surfaces at
heights x and x + dx. The resultant force acting on the element because of the variation in the
pressure is Adp, while the gravitational force on the element is pgAdx. In equilibrium there is no
net force acting on the element and, therefore,

dp + pg dx = 0

or,

dp -pg dx

18
In any fixed volume of gas the density is proportional to the pressure, provided that the
temperature is constant. With this restriction

p = cp

where c is a constant and

dp = -cpg dx

Therefore, integrating from the surface of the Earth, where the atmospheric pressure is p 0 , to a

r
height h, where the pressure is p, gives

Po
dp/p = -cg rdx o
Then,

ln(p/p 0) - cgh

or,

plp 0 exp(-cgh)

3.4 The equation of state of a certain gas is

pVm =A+ Bp

where A and Bare functions of temperature only. A sample of this gas, consisting of n moles of
molecules, is contained in a cylinder by means of a well-fitting, frictionless piston. In equilibrium
the gas occupies a volume Vi and exerts a pressure Pi· Calculate the new equilibrium pressure
following a quasistatic isothermal expansion of the gas to a volume Vt> equal to 2Vi, and also the
work done on the gas during the process.

Solution The system is the fixed mass of gas contained in the cylinder. In an isothermal process the
temperature of the gas remains constant, so that A and B may be treated as constants.
If n moles of the gas occupy a volume V, the molar volume Vm is given by

Vm = V/n
Let Pt be the final equilibrium pressure exerted by the gas. Then, since A may be treated as a
constant in the process considered,

A = PiViln- Bpi
= Pt Vtln - Bpt

But Vt = 2Vi, so that

or,

Pt = Pi [(Viln) - B]/[(2VJn) - B]

The work w done on the system in any infinitesimal change of volume dV is given by

w = -pdV

Therefore, in a finite quasistatic process in which the volume of the system changes from Vi to Vt,
the work W done on the system is given by

W =- f Vf
pdV
vi

19
Since the amount of work performed depends en the path of the process, p must be known as a
function of V. Now, from the equation of state,

p = AI(Vm- B)= AI[(VIn)- B]

so that

J
Vf
W = - AI[(VIn) - B] dV
Vj

= -A ln{[(2Vi/n)- B]I[(VJn)- B]}

W is negative, indicating that the work is done by the gas on the surroundings.

3.5 The Ideal The independence off(8) in Equations (3.5) and (3.6) ofthe nature ofthe gas considered
Gas Scale of makes the product p V m for an ideal gas a suitable parameter for the definition of a
temperature scale that does not depend on the properties of a particular substance. The
Temperature function chosen for the measurement of temperature defines a temperature known as the
and the Ideal ideal gas temperature T. The defining equation is
Gas Equation
(3.7)

where Tis the ideal gas temperature corresponding to a molar volume V m and a pressure
p, and R is a constant that has the same value for all ideal gases. The unit of temperature
is fixed by the value of 1/R. To specify R it is necessary to specify the value of Tat one
fixed temperature only. The standard agreed internationally is to assign a value of 273.16
kelvins, symbol K, to the temperature of the triple point of pure water, that is, the
temperature at which ice, water and water vapour coexist in equilibrium (see Section
9.2). The value of R is given by

R = (pVm) 3 (J mol- 1)/273.16 (K)

where (p V m) 3 is the value of p V mat the triple point of water. R has the same value for all
ideal gases. The result is

R = 8.315 J K- 1 mol- 1

R is known as the molar gas constant and Equation (3.7) is usually written in the form

pVm = RT (3.8)

an equation known as the ideal gas equation. For real gases the equation equivalent to
Equation (3.8) is

lim pVm = RT
p->0

The importance of Equation (3. 7) is that it defines a temperature T that is identical to a


temperature based on the behaviour of an ideal heat engine and known as thermodyna-
mic temperature (see Chapter 5). In this book the symbol Twill be used for temperature
defined in both ways.
Note that for a system consisting of n moles of ideal gas, occupying a volume V at a
pressure p and a temperature T, the ideal gas equation takes the form

pV = nRT (3.9)

since V m equals Vln.

20
3.6 Worked
Examples on
the Ideal
Gas Equation 3.5 A spherical bulb A of volume 1.0 x 10-3 m3 is connected to a bulb B of volume 0.50 x 10-3 m 3 by a
short tube and the whole is full of air at a pressure of 1.01 X 105 Pa and at a temperature of 273 K.
The temperature of the larger bulb is then raised to 373 K while that of the smaller bulb is
unchanged. Calculate:

(a) the new equilibrium pressure in the connected bulbs, and


(b) the fraction of the original mass of air in the larger bulb that is transferred to the smaller one as
a result of the change of temperature.

Solution The system is the total amount of air in both bulbs, as shown in Fig. 3.5, and is, therefore, a system
of constant mass.
Assume that air behaves as an ideal gas. A straightforward approach to problems of this nature
is to apply the ideal gas equation to each bulb separately and then apply the condition that the total
amount of gas (that is, the total number of moles of molecules) remains constant.
(a) In the initial equilibrium state let the number of moles of air in the larger bulb be n 1 and that
in the smaller bulb be n 2 • Applying the equation

Figure 3.5 The connected pV = nRT


spherical bulbs.
to each bulb separately gives

1.01 X 105 (Pa) X 1.0 X 10-3 (m 3 ) = n 1R X 273 (K)

for the larger bulb and

1.01 X 105 (Pa) X 0.5 X 10-3 (m3 ) = n2 R X 273 (K)

for the smaller bulb.


In the final equilibrium state let the pressure be Pt, the number of moles of air in the larger bulb
be n 3 and in the smaller bulb be n4 • Then, applying the ideal gas equation to the larger bulb gives

while for the smaller bulb

assuming that there is no change in the volume of the larger bulb.


Conservation of the total amount of air gives

so that

(1.01 X 105 X 1.0 X 10- 3 )/273 R + (1.01 X 105 X 0.5 X 10- 3)/273 R
= (Pt X 1.0 X 10- 3 )/373 R + (Pt X 0.5 X 10- 3 )/273 R

or,

Pt = 1.23 X 105 Pa

(b) Since the mass of gas is proportional to the number of moles of gas, the fraction f of the
original mass of air in the larger bulb that is transferred to the smaller one is given by

21
From the ideal gas equation

n = pVIRT

and, therefore,

f {[1.01 X 105 (Pa) X 1.0 X 10- 3 (m3 ))/273 (K) X R


- [1.23 X 10 (Pa)
5 X 1.0 X 10- (m )/373 (K)
3 3 X R)}
+ [1.01 X 105 (Pa) X 1.0 X 10- 3 (m3 )/273 (K) X R)
= 0.11

3.6 A long, uniform tube, open at one end, is surrounded by gas at a pressure p 0 • The temperature of
the tube varies uniformly from 100 K at the closed end to 200 K at the open end. When
equilibrium is reached the tube is closed and the temperature allowed to become uniform along the
tube at a value of 100 K. Calculate the new equilibrium pressure in the tube.

Solution The system is the gas in the tube when the end initially open is closed.
Let the end of the tube initially at a temperature of 100 K be taken as the origin of coordinates
and let the axis of the tube be parallel to the x-axis. At a distance x along the tube from the origin
let the temperature of the gas be T. Then, assuming that the gas is an ideal gas, the number of
moles of gas molecules in the tube lying between x and x + dx is dn' is given by

Po dV = dn'RT

where dV is the element of volume, equal to Adx where A is the area of cross-section of the tube,
R is the molar gas constant and p 0 is the (uniform) pressure in the tube. Then,

Po Adx = RTdn'

If L is the length of the tube, the temperature distribution along the tube is given by

(T- 100)/x = (200 - 100)/L


or,

TL - 100 L 100 x

that is,

T = 100(x + L)IL
so that

p 0 Adx 100 R (x + L) dn'IL

Separating the variables gives


dxl(x + L) = 100 R dn'lp 0 AL
and, if the total number of moles of molecules of gas in the tube is n, integrating gives

r 0
dx/(x + L) = 100 Rl(p0 AL) r
0
dn'

Then,

ln(2L/L) = 100 R nlp0 AL

This gives the number of moles of molecules in the gas:

n = (p 0 AL/100 R) ln 2

22
When the tube is at a uniform temperature of 100 K application of the ideal gas equation gives

pAL = [ (p 0 AL/100 R) In 2] R 100 (K)

or,

p =Po In 2

3. 7 A capillary tube of length L, closed at both ends, contains ideal gas. When the temperature of the
gas is uniform and equal to T0 the equilibrium pressure of the gas is p 0 • If the temperature
distribution is changed to

T = T0 exp( -ex)
where x is the distance measured from the end maintained at the temperature T0 and c is a
constant, derive an expression for the new equilibrium pressure p.

Solution The system is the mass of ideal gas contained in the uniform tube of length L and area of
cross-section A. When the temperature of the gas is uniform, having the value T0 , applying the
ideal gas gives

Po V = nRT0

where n is the number of moles of gas molecules and V is the volume occupied by the gas. V is
given by

V=AL

When the temperature is changed to a distribution given by

T = T0 exp(-cx)

the new equilibrium pressure p is achieved by a redistribution of the gas within the tube. Consider
an element of tube of length dx at a distance x from the end of the tube at a temperature T0 • The
number of moles of molecules contained in this element is dn', given by

pAdx = dn'RT
Substituting for T gives

pAdx dn'RT0 exp(-cx)

and, separating the variables,

exp(cx) dx (RT0 /Ap) dn'

r
This equation may now be integrated:

0
exp(cx) dx (RT0 /Ap) r0
dn'

giving

(exp(cL)- 1]/c = (RT0 1Ap)n

so that the new equilibrium pressure p is given by

p = (RT0 nc/A) + (exp(cL) - 1]

But,

23
so that

P = p 0 Lc![exp(cL) - 1]

3.8 Two vessels A and B containing an ideal gas are connected by a capillary tube of negligible volume.
Initially the gas in both vessels is in equilibrium at a temperature Ti. The temperature of vessel A is
then raised to a value Th while that of vessel B remains unchanged. When equilibrium is
re-established the pressure in the two vessels is found to be twice the original equilibrium pressure.
If the volume of vessel A is four times that of vessel B, calculate the value of Tr!Ti. Assume that
there is no thermal interaction between the two vessels when they are at different temperatures.

Solution The system is the gas contained in the vessels A and B. Initially, let vessel A contain n 1 moles of
molecules and vessel B contain n2 moles of molecules. When equilibrium is achieved following the
change to the temperature of vessel A, let vessel A contain n3 moles of molecules and vessel B
contain n 4 moles of molecules.
Apply the ideal gas equation to the two vessels when the gas is in equilibrium before the change
in the temperature of vessel A. Then,

and

where Pi is the equilibrium pressure, VA and Va are the volumes of vessels A and B, respectively,
and Ti is the initial, common, temperature.
After changing the temperature of vessel A to the value Tr, let the common equilibrium pressure
be Pt· Assuming that there is no change in the volume of either vessel and no thermal interaction
by the flow of gas molecules from one vessel to the other, applying the ideal gas equation gives

and

From the conservation of the amount of gas

so that

or

Now

and

which gives

Therefore,

(4 +1- 2)/Ti 8/Tr


or

24
3. 7 Exercises
3.1 A barometer tube is 86.40 em in length and there is a 3.4 A certain mass of an ideal gas occupies a volume of 0.53 m 3
small quantity of air trapped above the mercury. The barometer at atmospheric pressure. To compress the gas in a quasistatic
registers a pressure of 76.20 em of mercury when the true atmos- isothermal process so that its volume is halved requires the
pheric pressure is 76.35 em of mercury. Calculate the true expenditure of 3.9 X 104 J of work. Calculate the value of the
atmospheric pressure when the instrument registers 71.37 em of atmospheric pressure. f1.06 X 105 Pa]
mercury, the temperature remaining constant.
[71.47 cmofmercury] 3.5 A motor tyre pump has a uniform cylindrical barrel of
length 12.0 em and is used to pump air into a tyre in which the
3.2 Two copper vessels are connected by a short copper tube pressure is 1.5 x 105 Pa. If the piston traverses the complete
fitted with a stopcock. One vessel has a volume of 1.0 x 10- 3 m3 length of the barrel in each stroke, calculate the distance it
and contains an ideal gas at a pressure of 2.0 x lOS Pa. The other travels before air enters the tyre when the piston is travelling
vessel has a volume of 3.0 x 10- 3 m3 and contains the same gas, slowly. Take atmospheric pressure to be 1.0 x lOS Pa.
but at a pressure of 4.0 x 105 Pa. When the stopcock is opened [4.0 em]
there is a movement of gas to produce a new equilibrium state.
Calculate the new equilibrium pressure if there is no change in 3.6 A bulb of volume 2.0 x 10- 4 m3 is connected by a short
the temperature of the gas. [3.5 X lOS Pa] tube to a bulb of volume 1.0 X 10- 4 m3 • When both bulbs are at a
temperature of 273 K the gas that they contain is at a pressure of
3.3 A capillary tube of uniform bore has one end closed. The 1.10 X 105 Pa. The temperature of the larger bulb is then
tube contains a thread of mercury 13.0 em long that encloses dry changed, while that of the smaller remains at 273 K. If the new
air between it and the closed end of the tube, the other end being equilibrium pressure ofthe gas in the two bulbs is 1.34 X 105 Pa,
open to the atmosphere. When the tube is placed with its axis calculate the new temperature of the larger bulb. [373 K]
vertical and the open end downwards the length of the air
column is 21.5 em, but when it is placed vertically with the open 3. 7 Calculate the volume occupied by one mole of molecules of
end upwards the length of the air column is 15.2 em. Calculate an ideal gas at a temperature of 273 K and a pressure of 1.01 x
the atmospheric pressure in centimetres of mercury. 105 Pa. (22.4 X 10- 3 m3 ]
[75.73 em mercury]

25
Topic Guide

4 The First Law of


4.1 Internal Energy
4.2 The First Law of
Thermodynamics
4.3 Problem-Solving
4.4 Worked Examples
on the First Law of
Thermodynamics
Thermodynamics
4.5 Reversible Processes
4.6 Worked Examples on
Reversible Processes
4.7 Heat Capacities
4.8 Worked Examples on Heat
Capacities
4.9 Exercises

4.1 Internal Energy The results of experiments in which work is done on a system having an adiabatic
boundary, for example, Joule's 'paddle wheel experiment', can be summarised in the
following statement:

When the state of a closed system is changed from an initial equilibrium state to a final
equilibrium state solely by the performance of work, the amount of work needed
depends only on the initial and final equilibrium states of the system and not on the
means by which the work is performed nor on the intermediate states through which
the system passes.

It follows, by analogy with potential energy, that the system possesses a property, the
change in which is determined only by the initial and final equilibrium states of the
system and the measure of which is the work needed to produce the change when
the boundary of the system of the system is adiabatic. This property of the system is the
internal energy, symbol U, and the defining equation is

.Ll U = Ut - Ui = W( adiabatic) (4.1)

Here .LlU is the change in U from Ui to Ut when work W is performed on the system
under adiabatic conditions. When work is done on a system and does not change the bulk
kinetic energy or potential energy it becomes internal energy of the system. The per-
formance of work on a system is one way of changing the internal energy of that system.
For a particular system U depends only on the state of the system, apart from an
arbitrary constant. (Equation (4.1) only defines changes in the internal energy, which
need to be specified relative to some reference state of the system.) Therefore, the
internal energy of a system is a thermodynamic coordinate or thermodynamic property
or state function of the system.
Consequently, the internal energy may be expressed as a function of the independent
coordinates specifying the equilibrium states of the system. For example, the equilibrium
states of a gas may be specified by the values of the pressure p and the volume V.
Therefore, the internal energy of a fixed mass of gas may be written

U = U(p,V) (4.2)

Further, since there can be no change in any coordinate in a cyclic process, that is, in a
process that begins and ends in the same state, the cyclic integral of the internal energy
must be zero:

~ dU = 0 (4.3)

26
4.2 The First Law When a closed system has a boundary that is not adiabatic the state of the system can be
of Thermodynamics changed without the performance of work, that is, it can be changed through a thermal
interaction. In the general situation, where there is both a work interaction and a thermal
interaction between a system and its surroundings, Equation (4.1) becomes

Ut - U; = !lU = Q + W (4.4)

where Q, called heat, is the energy transfer that occurs in addition to that from the
/ --- ...... performance of work on the system. Q results from the difference in temperature
W / System
-·-'~~
"- Q
between the system and its surroundings. A sign convention is implied by Equation (4.4),
~-;--
namely, work done on a system and heat transferred to a system are counted positive.
---
'- L1U I
......
Figure 4.1 shows how energy transfers to a system in the form of heat and work should be
/

Figure 4.1 The sign represented on a diagram. The arrows are drawn in the conventional positive directions.
convention for a work A useful terminology is to use the expression 'heat absorbed by a system' to describe any
interaction W and a thermal heat transfer between a system and its surroundings. If the actual direction of transfer is
interaction Q. known, and needs to be specified, a heat transfer to the system is described by saying that
the system has extracted a certain quantity of heat from its surroundings; when the
transfer is from the system, the system is said to have rejected a certain quantity of heat
to the surroundings.
Equation (4.4) is known as the first law of thermodynamics. It embodies three separate
ideas:

(a) the concept of internal energy;


(b) it extends the law of conservation of energy to situations in which mechanical energy
is not conserved;
(c) it shows that the performance of work on a system and the transfer of heat to a
system are equivalent ways of changing the internal energy of a system and that,
when the transfer of energy has ceased, the terms heat and work are redundant.

4.3 Problem- The steps to be followed in solving problems relating to closed systems are now as
Solving follows.

1. The system and its surroundings must be clearly identified and, where it is helpful, a
diagram of the system and its surroundings should be drawn.
2. As an aid to understanding the behaviour of the system an appropriate coordinate
diagram may be drawn. Label the various parts of the process on the diagram and
indicate the nature of each part.
3. If work and/or thermal interactions are identified they should be marked on the
diagram of the system, with arrows indicating the conventional positive directions of
the energy transfer, together with the magnitudes and signs if they are known.
4. Write down the first law of thermodynamics for each identifiable part of the complete
process. Development of this part may require a use of the heat capacity of the system
(see Section 4.7) and knowledge of the equation of state. When the process is not
reversible (see Section 4.5) care must be taken to ensure that an appropriate descrip-
tion of the process is given. This will become clear in the worked examples.

4.4 Worked
Examples on the
First Law of
Thermodynamics 4.1 In an adiabatic process between two equilibrium states a certain system does work of amount 400 J
on its surroundings. Calculate the corresponding change in the internal energy of the system.

Solution The first law of thermodynamics is

27
Ur - Vi = !!..U = Q + W

where the change in internal energy !!..U results from the system absorbing a quantity of heat Q and
having work of amount W done on it. The process considered is adiabatic so that there is no
thermal interaction between the system and its surroundings, that is, Q is zero. Therefore, the first
law for the process considered becomes

!!..U= W

The work is done by the system on the surroundings, that is, the work done on the system is
counted negative, using the sign convention valid for the given statement of the first law.
Then,

w = -40 J
so that

!!..U= -401

The internal energy of the system decreases by 40 J as a result of the process taking place.

4.2 In a common laboratory experiment two bodies at different temperatures are brought into thermal
contact within a vessel having adiabatic walls. When the two bodies in thermal contact have
reached an equilibrium state, show that the heat absorbed by one of the bodies is equal in
magnitude and opposite in sign to that absorbed by the other. Neglect the work done on the bodies
by the surrounding atmosphere.

Solution Let the vessel with adiabatic walls constitute the boundary of the system, which therefore
comprises the two bodies that will be brought into thermal contact. Since the boundary is adiabatic
there is no thermal interaction between the system and its surroundings. Further, the work
interaction may be neglected. Therefore, it follows from the first law of thermodynamics that the
internal energy U of the system is not changed by the bodies being brought into thermal contact,
that is,

!!..U= 0

Now !!..U may be written

where l!..U1 is the change in the internal energy of body 1 and l!..U2 that of body 2. The first law of
thermodynamics may also be applied to each body separately. Then

and

where Q1 is the heat absorbed by body 1 and W1 is the work done on body 1 during the process.
Both W1 and W2 may be neglected so that, using the result that the total change in internal energy
of the system is zero,

or,

4.3 A system proceeds from an equilibrium state i to an equilibrium state f by a path A and returns to
equilibrium state i by a different path B. Along path A the system extracts heat of amount 40 J
from the surroundings and does 20 J of work on the surroundings. Along path B the system has
10 J of work performed on it by the surroundings. Calculate the amount of heat absorbed by the
system along path B.

28
Solution Paths A and B, taken together, constitute a closed cycle of operations, starting and ending in state
i. Therefore, since the internal energy U of the system is a state function (thermodynamic
coordinate), the total change in the internal energy AU of the system along paths A and B must be
zero. But AU is made up of the change in internal energy AUA that is produced along path A plus
the change in internal energy AUa that occurs along path B. Therefore,

Now,

where QA is the heat absorbed by the system along path A and WA is the work done on the system
along path A. Similarly, for path B

Therefore,

Using the sign convention for the first law

QA=+40J; WA = -20J; Wa = +10 J


Substitution gives

0 = +40 (J) + (-20) (J) + QB + 10 (J)

or

QB = -30 J

Along path B the system rejects 30 J of heat to the surroundings.

4.4 When a fluid undergoes an expansion in which there is neither a work interaction nor a thermal
interaction it is said to undergo a free expansion. Determine the change in internal energy of a
system that results from a free expansion.

Solution For any system undergoing a process from an initial equilibrium state to a final equilibrium state,
the change in internal energy AU is given by the first law of thermodynamics:

AU= Q +W
where Q is the heat absorbed by the system and W is the work done on the system during the
process. In a free expansion both Q and Ware zero and, therefore,

AU= 0 (free expansion)

4.5 Reversible In a general infinitesimal process, Equation (4.4) must be written


Processes
dU = q +w (4.5)

where q and w are simply infinitesimal quantities of heat and work, respectively. The
reason for this is that U is a function of the state of the system and 11U depends only on
the initial and final states of the system and not only on the nature of the process. In
contrast, Q and W simply represent ways of changing the value of U. They refer to
processes rather than to equilibrium states and their relative values for a given change in
the state of a system depend on the nature of the process, that is, on the path. An

29
infinitesimal that is not the differential of an actual function is called an inexact differen-
tial. Both q and ware inexact differentials. However, if certain conditions are fulfilled, it
is possible for q and w to behave as well-behaved (or exact) differentials dQ and dW,
respectively. Equation (4.5) may then be written

dU = dQ + dW (4.6)

These conditions require more information than merely a specification of the initial and
final states of the system. They are:

(a) The process must be quasistatic, that is, a succession of equilibrium states, so that, at
all stages of the process, the intensive coordinates are uniform throughout the
system and the extensive coordinates are well-defined.
(b) There must be no dissipative processes, such as friction or elastic hysteresis.

Processes that satisfy both these conditions are said to be thermodynamically reversi-
ble or, simply, reversible. For such a proc'!ss, the coordinates descriptive of the be-
haviour of both the system and the surroundings are identical. Further, for such a
process, an infinitesimal change in the values of the coordinates describing the surround-
ings is sufficient to start the system retracing its path exactly. For a thermal interaction to
give rise to a reversible process the temperature difference between the system and its
surroundings must always be less than infinitesimal. To achieve such a process, at least in
theory, it is customary to employ a series of heat reservoirs. A heat reservoir is a system
of such large mass that, in any thermal interaction with another system, it suffers no
appreciable change in temperature and so is able to impose its own constant temperature
on the system. Processes that do not satisfy these two conditions are irreversible.

4.6 Worked
Examples on
Reversible
Processes 4.5 It is required to change the temperature of a system from an initial equilibrium value Ti to a final
equilibrium value Tt by means of a reversible process. Describe how this may be achieved by the
use of a series of heat reservoirs.

Solution A series of heat reservoirs is arranged in which the first reservoir has a temperature Ti. Each
successive reservoir has a temperature that is infinitesimally different from that of its neighbours, in
a monotonic sequence, with the final reservoir having a temperature Tt, as in Fig. 4.2.
Initially the system is placed in contact with the reservoir at a temperature Ti. When thermal
equilibrium has been established the system is placed in contact with the reservoir at a temperature
Ti + d T. While the system is in this position it absorbs a quantity of heat dQ reversibly and reaches
an equilibrium temperature Ti + dT. The system is then placed successively in thermal contact with
adjacent reservoirs and allowed to come into thermal contact with each before being moved to the
next. In this way the system changes its temperature from Ti to Tt reversibly.
Figure 4.2 A series of heat
reservoirs to produce a
reversible heat transfer
between a system and its
surroundings over a finite
range of temperatures of the
system.

4.6 Joule found experimentally that, when a dilute real gas undergoes a free expansion, there is no
change in the temperature of the gas. On the assumption, verified by more precise experiments,
that this result applies exactly to ideal gases and is only approximately true for dilute real gases,
examine how the internal energy of an ideal gas depends on pressure when the temperature is held
constant.

30
Solution The system is a fixed mass of an ideal gas and the process is a free expansion. Therefore, there is no
work interaction and no thermal interaction, so that the internal energy has the same value in both
the initial and final equilibrium states. The actual free expansion is a highly irreversible process:
finite pressure gradients exist in the gas while the expansion is taking place and there is a
transformation of some of the internal energy of the gas into bulk kinetic energy and this kinetic
energy is then dissipated through viscosity into internal energy again. However, at the start and
end of the process the gas is in an equilibrium state in which the coordinates are well-defined
quantities and relationships may be looked for between the coordinate values in these states.
Let the system undergo an infinitesimal free expansion. The change in U may be written as a
function of any two other coordinates and it is convenient to choose temperature and volume.
Then

dU = (amav)T dV + (amaT)v dT
Now, in any infinitesimal free expansion dU is zero and Joule's experiment suggests that, for an
ideal gas, dT is also zero. However, for there to be an expansion at all, dV must be non-zero.
Therefore,

(4.7)

that is, U is independent of V for an ideal gas at constant temperature. This result is known as
Joule's law.
It may be noted that, in general,

while, for ideal gases,

but

Therefore, for ideal gases,

(4.8)

Since U can only be a function of any two of p, V and T, Equations (4.7) and (4.8) indicate that
Joule's law may be written more generally in the form

U = <I>(T)

where <I> is a function of temperature only.

4.1 Heal Capacities Let any closed hydrostatic system absorb a quantity of heat Q without undergoing a
phase change. If the temperature of the system changes from Ti to Tt then (C), defined
by the equation

(4.9)

is called the mean heat capacity of the system over the temperature range Ti to Tt. (C)
has units J K~ 1 • The heat capacity Cat a temperature Tis the limiting value of (C) as Ll T
tends to zero:

C = lim Q/ilT (4.10)


AT--+ 0

The heat capacity per unit mass is known as the specific heat capacity c. For a closed
hydrostatic system of mass m the specific heat capacity is given by

c = C/m = (lim) lim Q/ilT (4.11)


tl.T--+0

31
The unit of specific heat capacity is J kg- 1 K- 1 • A molar heat capacity em may be defined
by the equation

em= e/n (4.12)

where n is the number of moles of the specified entity, often the number of moles of
molecules.
e
For a particular system the value of depends on the nature of the process, since,
when a system absorbs a quantity of heat, some of the energy may be used to perform
work on the surroundings. The effect can be seen by writing the first law in the form

Q = !l.U- W (4.13)

When the process is reversible the path is specified and W becomes a well-defined
quantity. A given value of Q then produces a definite change in the internal energy of the
system and this gives a definite change in temperature of the system. Each path that the
system may follow has its own heat capacity. For closed hydrostatic systems two so-called
principal heat capacities are found to be most useful. These are the heat capacity at
constant volume ev, defined by the equation

ev = lim Qvl!l.T = dQvldT (4.14)


dT-o

and the heat capacity at constant pressure eP, defined by the equation

eP = lim QP/!l.T = dQP/dT (4.15)


dT-+ 0

Here dQv is the amount of heat absorbed by the system during an infinitesimal reversible
process carried out at constant volume and dQP that absorbed in a similar process carried
out at constant pressure.
In an infinitesimal reversible process the work w done on a closed hydrostatic system
may be written

w = -pdV

so that the first law for such a process is

dQ = dU+ pdV
Then,

ev = dQvldT = (iJU/iJT)v (4.16)


and

(4.17)

An apparatus for measuring heat capacities is called a calorimeter. A common way of


measuring the heat capacity of a system is to do electrical work on the system and
measure the resulting change in temperature of the system. However, as soon as the
temperature of the system differs from that of the surroundings the system absorbs heat
from the surroundings, and allowance must be made for this. In particular, if the system
contains an embedded resistor of resistance R and a constant current I flows through this
resistor for a time dt, the energy supplied to the system as electrical work is given by
PRdt. If the mass of the system ism and the temperature rise is dT, then, in the presence
of an adiabatic boundary

32
When the boundary of the system is diathermic this equation becomes

where dQ is the heat absorbed from the surroundings. This equation may be written

J2R + dQ!dt = mcPdT!dt

or

P + dQ!dt = mcPdT!dt

where P is the power dissipated in the resistor.


A major problem in the practical determination of heat capacities is the elimination or
estimation of dQ!dt. In many simple laboratory situations the major heat loss from a
system is by convection (see Chapter 14 for more details.). Newton proposed a general
equation to describe heat loss by convection, which may be written in the following way.
The rate at which heat is transferred from surroundings at a temperature T. to a system
of surface area A and temperature Tis dQ!dt, given by

dQ!dt = - hA(T - T.) (4.18)

where h is a quantity known as the convection coefficient or the heat transfer coefficient.
The major problem in the analysis of heat transfer by convection is to predict the value of
h. For forced convection generally, and for natural convection when T- T. is small, his
a constant. Equation (4.18) may then be written

dQ!dt = - K(T- T.) (4.19)

In this form it is known as Newton's law of cooling.

4.8 Worked
Examples on
Heal CapacitieS 4.7 At low temperatures the specific heat capacity at constant pressure cP for many metals is given by

cP =aT+ bT3

where a and bare constants. For copper the value of a is 10.0 X 10- 3 J kg- 1 K- 2 and that of b is
7.62 x 10- 4 J kg- 1 K- 4 • Calculate the amount of heat that a mass of 3.5 kg of copper must absorb
for its temperature to change from 1.5 K to 3.0 K.

Solution The system is the mass of 3.5 kg of copper. If there is no work interaction the heat q absorbed by
the system when its temperature rises by an amount dT under conditions of constant pressure is
given by

where m is the mass of the system and cP is the specific heat capacity at constant pressure. In a finite
reversible process, in which the temperature of the system changes from T; to Tr, the heat absorbed
by the system is Q, given by

For copper at low temperatures

33
rr
so that

Q m (aT + bT3 ) dT
Tj

giving
Q
that is,

Q = m [a(Ti - Tf)/2 + b(n - T{)/4]


Now substitute the given values to obtain
10.8 X 10- 3 (J k - 1 K- 2 )
Q = 3.5 (kg) ( 2 g [(3.0f (K2 ) - (1.5) 2 (K) 2 ]

+ 7.62 X 10- 4 (J k - 1 K- 4 ) )

4 g [(3.0) 4 (Kt - (1.5t (Kt]

0.178 J
Q is positive, showing that the transfer of energy is to the system.

4.8 A vertical glass tube 1.00 m long and closed at both ends contains a small amount of mercury.
The result of rapidly inverting the tube twenty times in succession is to raise the temperature by
1.40 K. Calculate the mean specific heat capacity of mercury. Assume that the acceleration of free
fall is 9.81 m s- 1 •

Solution The system is the small mass of mercury, of mass m, say, contained in the tube. When the tube is
rapidly inverted the mercury acquires a potential energy relative to the lower end of the tube given
by mgL, where g is the acceleration of free fall and L is the length of the tube. This potential
energy is transferred to kinetic energy as the liquid falls in the tube and becomes internal energy of
the mercury when the mercury is brought to rest, ignoring any energy lost to sound waves, or going
to augment the internal energy of the tube.
If the tube is inverted n times in succession, the total amount of energy transferred to the
internal energy of the mercury is nmgL. This energy produces a temperature rise f). T in the
mercury such that

nmgL = mcP f).T

where cP is the mean specific heat capacity of mercury at constant pressure. Therefore,
cP ngL/f).T
20 X 9.81 (ms- 2 ) x 1.00 (m)
1.40 (K)
140 J kg- 1 K- 1

4.9 Three different liquids A, B and C are in equilibrium states at temperatures of 285 K, 292 K and
301 K, respectively. When 1.00 kg of liquid A is mixed with 1.00 kg of liquid B the resulting
equilibrium temperature is 289 K, while on mixing 1.00 kg of liquid B with 1.00 kg of liquid C the
resulting equilibrium temperature is 296 K. Calculate the equilibrium temperature when 1.00 kg of
liquid A is mixed with 1.00 kg of liquid C. Assume that the specific heat capacity at constant
pressure is a constant for each liquid.

Solution Let the specific heat capacities at constant pressure of liquids A, B and C be cpA> cpB and cpc.
respectively. In the first mixing, the system is 1.00 kg of liquid A and 1.00 kg of liquid B. If it is
assumed that there is no interaction with the surroundings, then

where f). U is the change in the internal energy of the system, f). U A is that for the 1. 00 kg of liquid A
and f).UB is that for the 1.00 kg of liquid B. Now

34
and

where QA is the heat absorbed by the mass of liquid A during the mixing and W A is the work done
on it. If the work interaction between liquids A and B is neglected

Let Tr be the final equilibrium temperature when liquids A and B are placed in thermal contact.
Then, if TiA is the initial equilibrium temperature of liquid A and TiB is that of liquid B,

QA = mAcpA(Tr - TiA) = 1.00 (kg) x cpA x (289 (K) - 285 (K))

QB = mBcpB(Tr- TiB) = 1.00 (kg) X cpB X (289 (K) - 292 (K))

Therefore,

or

(4.20)

Similarly, when liquids B and C are mixed:

QB 1.00 (kg) X CpB X (296 (K) - 292 (K))

Qc 1.00 (kg) X Cpc X (296 (K) - 301 (K))

or

(4.21)

From Equations (4.20) and (4.21) it follows that

(4.22)

When liquids C and A are mixed, let the final equilibrium temperature be Tr. Then, using the
reasoning above,

Qc = 1.00 (kg) X Cpc X (Tr - 301 (K))

and

QA = 1.00 (kg) X CpA X (Tr - 285 (K))

Cpc X (Tr - 301) + cpA X (Tr - 285) = 0

so that

cpA/Cpc = -(Tr- 301)/(Tr - 285) (4.23)

Combining Equations (4.22) and (4.23) gives

-(Tr - 301)/(Tr - 285) = 15/16

so that

35
15 Tf - 15 X 285 - 16 Tf + 16 X 301

giving

Tf = 293.3 K

4.10 A piece of lead of mass 0.600 kg and at a temperature of 373.0 K is placed in a copper vessel of
mass 0.200 kg which contains 0.500 kg of water at a temperature of 290.0 K. The final steady
temperature of the mixture is 292.8 K. If the specific heat capacity of copper is 386 J kg- 1 K- 1 and
that of water is 4180 J kg- 1 K-\ calculate the specific heat capacity of lead.

Solution The system consists of the piece of lead, the copper calorimeter and the water. Assume that there
are no thermal interactions with the surroundings and no work interactions between any parts of
the system or the surroundings. With the given restrictions the internal energy U of the system is
unchanged by the process; that is,

flU= 0

flU is given by

flU= flU(lead) + flU(copper) + flU(water)


where flU(lead) is the change in the internal energy of the lead, and so on. Because it is assumed
that there are no work interactions, for each part of the system the change in the internal energy is
equal to the heat absorbed. For example,

flU(lead) = Q(lead)
where Q(lead) is the heat absorbed by the lead during the process, and is given by

Q(lead) = m(lead) X cP(lead) X (Tf - T;)(lead)

where m is the mass, cP is the specific heat capacity at constant pressure, T; is the initial
temperature and Tf is the final temperature. Then, remembering that flU for the whole system is
zero,

0 m(lead) x cp(lead) x (Tf- T;)(lead) + m(copper) X cp(copper)


x (Tf- T;)(copper) + m(water) x cp(water) x (Tt - T;)(water)

Substituting the given values gives

0 = 0.600 (kg) X cp(lead) X (292.8 (K) - 373.0 (K))


+ 0.200 (kg) X 386 (J kg- 1 K- 1 ) x (292.8 (K)- 290.0 (K))
+ 0.500 (kg) X 4180 (J kg- 1 K- 1) X (292.8 (K) - 290.0 (K))

which gives

4.11 For a certain closed hydrostatic system the value of the molar heat capacity at constant pressure
Cp,m at a temperature Tis given by

where a, b and c are constants. Show that the mean specific heat capacity over the temperature
range T1 to T2 differs from the value at the temperature (T1 + T2 )12 by the amount

36
Solution When the molar heat capacity at constant pressure cp,m is given by

Cp,m = a + bT + cT 2
the mean molar heat capacity at constant pressure ( Cp,m) over the temperature range T1 to T2 is
given by

Now,

and

so that
1/(T2 - T 1 ) [a(T2 - T 1 ) + (b/2)(T2 - T1)(Tz + Tt)
+ (c/3)(T2 - T 1)(T; + T1 T2 + Ti)]
a + (b/2)(T2 + T 1 ) + (c/3)(T; + T 1 T2 + Ti)

At the temperature (T1 + T2 )/2 the value of Cp,m is given by


Cp,m a + b(T1 + T2 )/2 + c(T1 + T2 ) 2/4
a + b(T1 + T2)/2 + c(T~ + 2T1 T2 + T;)/4

The difference between <cp,m) and cp,m is, therefore,

which equals

or

that is,

4.12 The temperature of a body falls from 373 K to 333 K in three minutes when the temperature of the
surroundings is constant and equal to 283 K. How long will it take for the temperature to fall from
333 K to 293 K? Assume that Newton's law of cooling is obeyed.

Solution The system is the body that is losing heat to surroundings at the constant temperature T5 • Assume
that the temperature of the body is uniform throughout and that the rate of heat loss is given by
Newton's law of cooling. When the temperature of the body is T the rate of heat loss to the
surroundings is dQ/dt, given by
dQ/dt = - hA(T- T5 )

37
where A is the surface area of the body and his the appropriate (constant) convection coefficient.
Now, when the body absorbs a quantity of heat dQ it suffers a change in temperature dT, given by

where CP is the heat capacity at constant pressure of the body. Substituting for dQ gives

CP dT/dt = - hA(T- T8 )

or,
dT hA
--dt
T- Ts cp
Integrating this equation over the time interval t', during which the temperature of the body falls
from Ti to Tf

fq

Ti
dTI(T - T8 ) = - fr

0
(hA/Cp) dt

which gives

ln(T- T8 ) = - (hA/Cp) t' + constant


When t' is zero the value of T is Ti so that

constant = ln(Ti - T8 )

Then

ln(T- T8 ) -(hA/Cp) t' + ln(Ti - Ts)

or

-(hA/Cp) t' = ln(T- Ts) - ln(Ti - Ts)

The temperature of the body falls from 373 K to 333 Kin 3.0 minutes so that

- (hA/Cp) x 3.0 (min) = ln(333 (K) - 283 (K)) -ln(373 (K) -283 (K))
and

-(hA/Cp) = - 0.195 928 9

The total time for the temperature of the body to fall from 373 K to 293 K is given by

0.195 928 9 t' = ln(373 (K) - 283 (K)) - ln(293 (K) - 283 (K))

giving

t' = 11.2 min

Therefore, the time taken for the temperature of the body to fall from 333 K to 293 K is equal to

11.2 - 3.0 = 8.2 min


4.13 To determine the specific heat capacity of a certain oil, a piece of copper of mass 0. 700 kg at a
temperature of 368 K was placed in a copper calorimeter of mass 0.150 kg, containing 0.500 kg of
the oil at a temperature of 288 K. If time is measured from the instant at which the mass of copper
was placed in the oil, the temperature T of the oil varies with timet, as shown in the table below.

t (s) 0 30 60 90 120 150


TIK 288 293 297 300 299 298

38
Determine the approximate temperature which the calorimeter and its contents would have
reached in the absence of any heat losses to the surroundings and, consequently, calculate the
specific heat capacity of the oil. Assume that the specific heat capacity of copper is 386 J kg- 1 K- 1
and that the temperature of the surroundings is constant and equal to 288 K

Solution The system consists of the piece of copper, the copper calorimeter and the oil. Assume that the
work interactions between any parts of the system and the surroundings may be neglected. While
the temperature of the system is reaching its maximum value let the thermal interaction with the
surroundings be Q'. Then

Q' = f't...U

where f't...U is the change in the internal energy of the system, given by

f't...U = f't...U(oil) + f't...U(calorimeter) + f't...U(copper)


The variation of Twith t has the form shown in Fig. 4.3. Assume that the rate of loss of heat to
the surroundings is proportional to (T - T5 ), where T5 is the temperature of the surroundings.

T/K

304
302 T'm
300
Tm
298
296
294
292
290
288o· Ts
Figure 4.3 The dependence 0
K
of temperature T on time t. 0 20 40 60 80 100 120 140 160
t(s)

Further, assume that the maximum in the temperature-time graph occurs when energy transfer
between the piece of copper and the calorimeter and oil ceases. Then, the rate of fall of
temperature along the part of the curve AB in Fig. 4.3 corresponds to the maximum rate of heat
transfer to the surroundings, that is, when the temperature excess is a maximum and when the
temperature of the system is close to being uniform. The average excess temperature over the
curve 0' A is one half of the maximum excess temperature. Therefore, if a time interval KL is
marked, such that KL equals one half the interval OK, the temperature fall f... Tin this time interval
is equal to the temperature fall over the interval OK. Therefore, the true maximum temperature
T/n is equal to the observed maximum temperature Tm plus 11. T:

From Fig. 4.3, the value of f't...T is 1.6 K, so that

T/n = 300 (K) + 1.6 (K) = 301.6 K

Using the corrected temperature change is equivalent to assuming that the system is isolated.
Then,

Q(oil) + Q(copper) + Q(calorimeter) = 0

or,

m(oil) x cp(oil) x [T/n- Ti(oil)] + m(copper) x cp(copper) x [T/n- T/copper)]


+ m(calorimeter) x cp(calorimeter) x [T/n- Ti(calorimeter)] = 0

where m is the appropriate mass, cP the specific heat capacity at constant pressure and Tithe initial
temperature. Substitution of the values gives

39
0.500 (kg) X cp(oil) X [301.6 (K) - 288 (K)] + 0.700 (kg) X 386 (J kg- 1 K- 1 )
X [301.6 (K) - 368 (K)] + 0.150 (kg) X 386 (J kg- 1 K- 1)
X [301.6 (K) - 288 (K)] =0
from which

0.500 X cp(oil) X 13.6 + 0.700 X 386 X (-66.4) + 0.150 X 386 X 13.6 =0


giving

4.14 A block of metal of mass 3.0 kg has a small resistor embedded in it. The block is placed in a
constant temperature enclosure and a steady current passed through the resistor. At a series of
different values for the power P dissipated in the resistor, the equilibrium temperature Te of the
block of metal is measured and the following results obtained:

P(W) 0 5.00 10.0 15.0 20.0 25.0 30.0 35.0 40.0


T.IK 293.0 294.3 295.5 296.5 297.5 298.4 299.2 299.9 300.6

The block of metal then has its temperature raised to a value over 305 K when the current through
the resistor is switched off. The temperature T of the block is found to vary with time t as in the
following table:

t(s) 60.0 120 180 240 300 360 420


TIK 302.1 300.1 298.5 297.3 296.3 295.5 294.9

Calculate the specific heat capacity at constant pressure of the metal at 298.0 K.

Solution The system is the block of metal and the embedded resistor. In an infinitesimal process electrical
work dW of amount / 2 R dt is done on the system when a current I is passed through the resistor of
resistance R for a time dt. The system absorbs heat dQ from the surroundings in the same time
interval when there is a temperature difference between the system and its surroundings. In
general, the corresponding change in the internal energy of the system is dU, given by

dU = dQ + dW (4.24)

Provided that the work done on the system by the atmospheric pressure as a result of volume
changes of the system can be neglected, the change in the internal energy may be written

dU = mcP dT (4.25)

(see Equation (4.17)), where m is the mass of the system, cP is the specific heat capacity at constant
pressure and dT is the change in the temperature of the system. Substituting for dQ and dW in
Equation (4.24) gives
mcP dT = fR dt + dQ (4.26)

Equation (4.26) may be written

mcP dT/dt = JZR + dQ/dt


or,

mcP dT/dt = P + dQ/dt


where Pis the power dissipated in the resistor. In the first part of the experiment dT/dt is zero so
that, in each equilibrium state,

p + dQ/dt = 0 (4.27)

In the second part of the experiment P is zero and so

mcP dT/dt = dQ/dt (4.28)

40
When the system is at a particular temperature the rate of loss of heat to given surroundings is the
same, whether the temperature of the system is constant or varying. (The variation is supposed to
be so slow and the rate of heat transport within the system so high that the temperature is uniform
throughout the system.) Therefore, for a given temperature in both parts of the experiment,
Equations (4.27) and (4.28) may be combined to give

P = - mcP dT/dt (4.29)

The required temperature of the system is 298 K. At this temperature the power needed to
maintain the temperature of the system constant is obtained from the graph of P against Te (Fig.
4.4). The value is 22.5 W. The rate of fall of temperature in the absence of a work input is obtained
from the slope of the graph ofT against t (Fig. 4.5). When Tis 298 K the value of dT/dt is negative
and equal to - 0.0198 K s- 1 • Substitution in Equation (4.29) gives

22.5 (W) =- 3.0 (kg) X cp X (- 0.0198) (K s- 1)


so that

cP 22.5/(3.0 x 0.0198)
379 J kg- 1 K- 1

T,/K

300
299

Figure 4.4 The dependence


of equilibrium temperature Te
on power input P.

Figure 4.5 The dependence


of temperature T on time t.

4.15 The mean heat capacity at constant pressure (cp) for a closed hydrostatic system over the
temperature range Ti to Tt is defined by the equation

(cp) (Tt - Ti) = rt cPdT


Ti
where cP is the heat capacity at constant pressure at a temperature T. Show that, when cP is given
by the equation

cP =a+ bT

where a and b are constants, (cp) is equal to the heat capacity evaluated at the temperature
(Ti + Tt)/2.

r
Solution For the given dependence of cP on T, the defining equation for (cp) becomes
(cp) (Tt- Ti) = (a+ bT) dT
Ti

41
Integrating gives

or,

a( Tt- T;) + (b/2)(Tj - Tt)


which may be written

a(Tt - T;) + (b/2)(Tt - T;)(Tt + T;)


giving

which is the required result.

4.9 Exercises
4.1 Estimate the difference in temperature between water at mine the mass of aluminium filings in the mixture placed in the
the top and bottom of Niagara Falls, where the drop is 50.0 m. vessel. The specific he_at. capacity at constant pressure is
Why is the calculated temperature difference not observed? 900 J kg- 1 K- for alummmm, 386 J kg- 1 K- 1 for copper and
Take the acceleration of free fall as 9.81 m s- 2 and the specific 4180 J kg- 1 K- 1 for water. [0.193 kg]
heat capacity at constant pressure of water as 4180 J kg- 1 K- 1 •
[0.12K] 4.6 A calorimeter of heat capacity 420 J kg- 1 K- 1 contains
0.200 kg of an oil with specific heat capacity at constant pressure
4.2 A car of mass 1000 kg travelling at 40.0 m s- 1 is brought to equal to 3.20 J kg- 1 K- 1 • The calorimeter is fitted with a small
rest by the application of brakes acting on the front wheels. If the resistor of resistance 2.0 0 and negligible temperature coef-
effective mass of each brake is 2.0 kg and the specific heat ficient. When no current is flowing the temperature of the calori-
capacity of the material of the brake is 800 J kg- 1 K- \ estimate meter and its contents falls from 333 K to 313 K in 4 min when
the maximum temperature rise of the brakes during the braking the temperature of the surroundings is 288 K. Determine the
operation. [250 K] value of the current through the resistor needed to maintain the
temperature of the calorimeter and its contents at 323 K in the
4.3 One method of measuring the temperature of a furnace is same surroundings. Assume that the thermal interaction is de-
to place a sphere of copper in the furnace until it reaches equilib- scribed by Newton's law of cooling. [0.21 A]
rium and then drop it into a copper can containing a known mass
of water and observe the temperature rise. If the sphere is of 4.7 Two brass cones fit one inside the other. The outer cone is
mass 100 g, the copper can is of mass 150 g and contains 200 g of rotated about a vertical axis at 720 r.p.m. while the inner one is
water at an equilibrium temperature of 290 K, and the final held fixed by an arm whose length, measured from the axis of
equilibrium temperature of the water is 311 K, calculate the rotation, is 15.0 em, and to which a horizontal force of 2.00 N is
temperature of the furnace. Assume that there is no loss of water applied at right angles to the arm. The total mass of the two
by evaporation. Take the specific heat capacity at constant press- cones is 0.250 kg and the inner one contains 0.025 kg of water. If
ure to be 386 J kg- 1 K- 1 for copper and 4180 J kg- 1 K- 1 for the initial temperature of the arrangement is 293 K, calculate the
water. [797 K] temperature it reaches at the end of five minutes of rotation.
Neglect any heat absorbed from the surroundings. [327 K]
4.4 Over the temperature range 300 K to 1500 K the molar
heat capacity of oxygen Cf!.m varies with temperature T accord- 4.8 In a high-energy experiment a beam of hydrogen atoms
ing to the empirical equatiOn passes through a vacuum to strike a brass target of mass
0.500 kg. If the hydrogen atoms have a speed of 2.0 X 107 m s- 1
and 1015 atoms strike the target per second, calculate the time
needed for the temperature of the target to rise by 100 K.
where a, band care constants which, for oxygen, have the values The mass of a hydrogen atom is 1.67 x 10- 27 kg and the
specific heat capacity at constant pressure of brass is 380 J kg- 1
a 3.068 K- 1 • [56.9 s]
b 1.638 X 10- 3 K- 1
4.9 When a piece of lead of mass 0.5 kg at a temperature of
c -0.512 X 10- 6 K- 1 323 K is placed in a vacuum flask containing 0.090 kg of water at
a temperature of 288 K, the resulting steady temperature is
and R is the molar gas constant. Calculate the heat absorbed by a 292 K. When 0.090 kg of water at a temperature of 323 K are
system consisting of two moles of oxygen when its temperature is added to a piece of lead of mass 0.5 kg at a temperature of 288 K
raised from 300 K to 1000 Kat constant pressure. [45.3 kJ] contained in the same flask, the steady temperature is 311 K.
Calculate the specific heat capacity at constant pressure of lead
4.5 A copper vessel of mass 0.200 kg contains 1.00 kg of water and the heat capacity of the vacuum flask, given that the specific
at a temperature of 293.0 K. When 0.400 kg of a mixture of heat capacity at constant pressure of water is 4180 J kg- 1 K- 1 •
copper and iron filings at a temperature of 373.0 K are placed in [140 J kg- 1 K- \ 125.4 J K- 1]
the vessel the final equilibrium temperature is 297.5 K. Deter-

42
Topic Guide
5.1

5.2
The Clausius Statement of
the Second Law
Heat Engines
5 The Second Law
5.3

5.4
Worked Examples
on Heat Engines
Kelvin's Statement of the
of Thermodynamics
Second Law
5.5 Carnal's Theorem and its
Corollary
5.6 Universal Temperatures
5. 7 Thermodynamic
Temperature
5.8 Worked Examples
on Heat Engines
and Thermodynamic
Temperature
5.9 Celsius Temperature
5.10 Problem-Solving
5.11 Worked Examples on Heat
Engines and the Second
Law of Thermodynamics
5.12 Exercises

5.1 The Clausius Not all processes allowed by the first law of thermodynamics actually occur; there are
Statement of the limitations that are expressed in a number of generalisations of experience that are
known as the second law of thermodynamics. The simplest statement of the second law is
Second Law that when two closed systems interact thermally, the hotter system always becomes
cooler and the cooler system always becomes hotter. In terms of the heat flow that
occurs, this statement may be put in the form: when two closed systems are placed in
thermal contact, the direction of the energy transfer as heat is always from the system at
the higher temperature to that at the lower temperature. This is the Clausius statement
of the second law.
An important alternative statement of the second law, due to Lord Kelvin, is based on
observations of the behaviour of real (cyclic) heat engines. These will now be considered
briefly.

5.2 Heat Engines A heat engine is a device that does work on its surroundings as a result of extracting heat
from its surroundings. From a thermodynamic viewpoint a heat engine consists of a
system, known as the working substance, that can interact thermally and mechanically
with its surroundings. Practical heat engines are cyclic, that is, they repeat a regular cycle
of operation, so that an unlimited amount of work may be done if an unlimited amount
of fuel is available. The thermal efficiency YJ of a heat engine is defined by the equation
YJ = - W/Q! (5.1)

where Q1 is the heat extracted from the surroundings per cycle and W is the work done
on the surroundings per cycle. Many heat engines effectively operate between two heat
reservoirs at fixed temperatures. Q1 is then the heat absorbed by the working substance
per cycle from the high-temperature reservoir.
When the cycle of operation begins and ends in an equilibrium state, so that the
internal energy U is well defined, the change in U for one cycle is zero, that is, I:!U = 0.
Then, applying the first law to one cycle,

I:!U = Q + W = 0

43
where Q is the total heat absorbed by the working substance in each cycle and W is the
work done on the working substance in each cycle. When the engine operates between
two heat reservoirs, designated 1 and 2,

where Q1 is the heat absorbed by the working substance per cycle from reservoir 1 and Q2
is the heat absorbed in each cycle from reservoir 2.
When a heat engine is operated in reverse it becomes a heat pump, if the objective is to
transfer heat to a system or reservoir at a high temperature, or a refrigerator, if the
objective is to extract heat from a cold system or reservoir. The figure of merit for these
arrangements is the coefficient of performance (C.o.P.). For the heat pump the C.o.P. is
defined by

C.o.P.(h) -Q/W (5.2)

and for the refrigerator by

C.o.P.(r) (5.3)

Here Q2 is the heat absorbed by the working substance from the low-temperature
reservoir or body in each cycle.

5.3 Worked
Examples on
Heat Engines 5.1 In Newcomen's heat engine (1712) 0.830 MJ of work were done on the surroundings for the
consumption of 38.10 kg of coal. Calculate the thermal efficiency of the engine, taking the calorific
value (the energy released in burning) of coal to be 28.0 MJ kg- 1 •

Solution This example provides a simple illustration of the use of the sign convention for the interactions in
a cyclic process. The system is the working substance of the heat engine, the boundary of which
..,..---- ' may be represented by the dashed line in Fig. 5.1.
The energy input is derived from burning the coal. Let the engine perform n complete cycles
____
/
nW 1 Working substance\ nO
while 38.10 kg of coal are burned. Inn complete cycles the work done on the system is n W, where
\ , ,.../
W is the work done on the system per cycle, and the heat absorbed by the system is nQ, where Q is
the heat absorbed per cycle.
Figure 5.1 Representation of The efficiency T) of the engine is given by
the working substance of a
heat engine and the T) = -nW!nQ
interactions. The arrows
indicate the conventional For the system under consideration
positive directions of the
energy transfers. nW = -0.830 MJ

The negative sign indicates that the work is actually done on the surroundings by the working
substance. Also

nQ 38.10 (kg) X 28.0 (MJ kg- 1 )


1066.8 MJ

This quantity has a positive sign, as the working substance extracts this amount of heat from the
surroundings. Then,

T) - (- 0.830) (MJ)/1066.8 (MJ)


0.000 78
0.078%

44
The efficiency of the earliest heat engines was very low. More important was their reliability,
together with a good supply of cheap fuel.

5.2 A domestic freezer is maintained at a constant temperature. In this condition the rate at which
work is done on the working substance of the freezer is 400 J s- 1 and the rate at which heat is
rejected to the surroundings is 1250 J s- 1 • Calculate the coefficient of performance of the freezer.

Solution The system is the working substance of the freezer, that is, the fixed mass of refrigerant that
circulates in the machine. Heat is absorbed by the system from both the surroundings and the
freezer compartment and work is done on the working substance.
---_----.l-d-:,-/d- 1
--Surroundings The arrangement may be represented as in Fig. 5.2, where the horizontal lines represent the two
bodies maintained at constant temperature (that is, the surroundings and the freezer compart-
ment) and the dashed line represents the boundary of the system. Let the rate at which heat is
f
/ '-+----dW!dt absorbed from the surroundings be dQ/dt, the rate at which heat is absorbed from the freezer be
dQ 2/dt and the rate at which work is done on the working substance be dW/dt. These interactions
' .... ~-/ are also shown in Fig. 5.2, with the arrows indicating the conventional positive directions of energy
ld02 /dt transfer.
-----'-----Freezer (constant The coefficient of performance is given by
temperature)

C.o.P. = Q21W = (dQ 2 /dt)/(dW/dt)


Figure 5.2 The
representation of a freezer
In one cycle of operation AU= 0 so that
operating between bodies at
two fixed temperatures.

or,

dQ/dt + dQ 2 /dt + dW/dt =0


Now

dW/dt = + 400 J s- 1

because the work is done on the working substance and

dQ/dt = -1250 J s- 1
because heat is rejected to the surroundings. Therefore,

dQzfdt - dQ/dt - dW/dt


- (-1250)(J s- 1 ) - 400(J s- 1 )
+850Js- 1

This is the rate at which heat is extracted from the freezer by the working substance. The
coefficient of performance is therefore 850/400 = 2.125.

5.3 A heat engine operating between two reservoirs has an efficiency of 25% and does 2000 J of work
on the surroundings in each cycle. Calculate the amount of heat absorbed in each cycle from both
the hot reservoir and the cold reservoir.

Solution The system is the working substance of the heat engine. The reservoirs and the interactions may be
represented as in Fig. 5.3, using the symbolism of Fig. 5.2. In one cycle of operation the working
substance absorbs heat Q 1 from the high-temperature reservoir, heat Q2 from the low-temperature
reservoir and has work W done on it by the surroundings. The thermal efficiency lJ of the heat
engine is given by

For the engine considered

45
----------~-0-,-------~ w = -2000 J
-, w since the work is done on the surroundings and, as TJ is 0.25,

'-ta:/
~----
\ -1--,....!.!.-
- (- 2000) (J)/0.25
8000 J
----------~-----------~

Figure 5.3 Representation of In one complete cycle the change in the internal energy of the working substance is zero so that
a heat engine operating
between two heat reservoirs.

Then

8000 (J) + Q2 + (-2000) (J) =0


giving

Q2 2000- 8000
-6000 J

The minus sign indicates that this heat is rejected to the low-temperature reservoir.

5.4 A certain heat engine has two moles of molecules of an ideal gas as its working substance and
undergoes a cycle of operation made up of the four consecutive steps (a)-( d) given below.

(a) The temperature of the gas is raised reversibly from 300 K to 400 K while the volume occupied
by the gas remains constant at 2.0 x 10- 3 m3 •
(b) The gas then expands reversibly and isothermally at a temperature of 400 K until the volume it
occupies is 10.0 x 10- 3 m3 •
(c) The temperature of the gas is then lowered reversibly from 400 K to 300 K while the volume
occupied by the gas remains constant at 10.0 x 10- 3 m3 •
(d) The gas is then compressed reversibly and isothermally at a temperature of 300 K until it again
occupies a volume of 2.0 x 10- 3 m 3 •

Taking the molar heat capacity at constant volume to be a constant equal to 12.47 J mol- 1 K- \
calculate

1. the heat absorbed by the gas in each of the stages (a)-( d);
2. the work W done on the gas in each cycle; and
3. the thermal efficiency of the engine.

Solution The system is the working substance of the heat engine and consists of 2 mol of molecules of an
ideal gas. This system undergoes a reversible cyclic process, made up of four steps, shown on the
pressure p against volume V graph in Fig. 5.4. Stages (a) and (c) are reversible isovolumic
processes while stages (b) and (d) are reversible isothermal processes. Consider the four stages in
turn.

(c)

Figure 5.4 The reversible


cyclic process represented in
the p-V plane. v

46
Stage (a)
Applying the first law to this stage gives

where Qa is the heat absorbed by the system during process (a), Wa is the work done on the system
during the process and !!..U is the resulting change in the internal energy. Since the volume of the
system remains constant during stage (a), Wa is zero and Qa is given by

where n is the number of moles of molecules constituting the system, Cv,m is the molar heat
capacity at constant volume and T1 and T2 are the temperatures corresponding to points 1 and 2,
respectively, in Fig. 5.4.
Now T1 = 300 K, T2 = 400 K, n = 2 mol and Cv,m = 12.47 J K- 1 mol- 1 • Therefore,

Oa 2 (mol) X 12.47 (J K- 1 mol- 1 ) X (400 (K) - (300) (K))


2494 J

Stage (b)
For this stage, using an obvious notation,

When an ideal gas undergoes a reversible isothermal process it suffers no change in its internal
energy (see p. 31), so that !l.Ub is zero and

The value of wb is obtained from

Wb = - fv3 p dV
vz
where V2 and V 3 are the volumes of the system corresponding, respectively, to points 2 and 3 in
Fig. 5.4.
Now, using the ideal gas equation

pV = nRT

so that

Wb= - fv
vz
3
p dV = - f v (nRT 1V) dV
Vz
3
2

= - nRT2 1n(V3 1V2 )


= - 2 (mol) x 8.315 (J K- 1 mol- 1 ) X 400 (K)
X ln(10.0 X 10- 3 (m 3 )/2.0 X 10- 3 (m3 ))
=- 10 705 J

The minus sign indicates that the work is done by the system on the surroundings. Consequently,

Qb = - wb = + 10 705 J
Stage (c)
Here, as in stage (a), the work interaction is zero, so that the heat absorbed by the system, Qc, is
given by

Qc nCv,m (T4 - T3)


2 (mol) X 12.47(J K- 1 mol- 1 ) X (300 (K) - 400 (K))
-2494 J

47
Stage (d)
This stage is similar to stage (b): there is no change in the internal energy of the system and the
work done on the system is Wct, given by

Wct = - f v1
p dV = - f vl
nRTiV dV
v4 v4
= - nRT 4 ln(V/V4 )
= - 2 (mol) X 8.315 (J K- 1 mol- 1) X 300 (K) X ln(2.0 X 10- 3 (m 3 )/10.0 X 10- 3 (m 3 ))
=+8029J

The positive sign indicates that the work is done on the system. Since Qd = - Wd,

Qd = -8029 J

The total amount of work done on the system in one cycle is W, given by

W = Wa + Wb + We + Wct
0 + ( -10 705) (J) + 0 + 8029 (J)
- 2676 J

The efficiency TJ is given by

TJ = - W/Qin

where Q; 0 is the heat extracted from the surroundings by the system, that is, the heat absorbed,
which has a positive sign. Then,

Qin Qa + Qb = 2494 (J) + 10 705 ( J)


13 199 J

Therefore,

T] = - (- 2676) ( J)/13 199 ( J)


0.20
20%

5.4 Kelvin's The limitations on the operation of cyclic heat engines are contained in Lord Kelvin's
Statement of the statement of the second law of thermodynamics: it is impossible to devise a machine
which, operating in a cycle, produces no effect other than the extraction of a quantity of
Second Law heat from its surroundings and the performance of an equal amount of work on the
surroundings.
One consequence of Kelvin's statement is that no heat engine can have an efficiency of
100%; there is always some rejection of heat in the cycle of operation.

5.5 Carnot's A very important cyclic process is that known as the Carnot cycle. This is an ideal cyclic
Theorem and its process and the (ideal) engine that performs it is a Carnot engine. In a Carnot cycle the
working substance performs a reversible cyclic process involving two heat reservoirs. The
Corollary important result known as Carnot's theorem does not depend on the temperature scale
used to specify the temperatures of the reservoirs, so an empirical temperature 8 will be
used in this section. The cyclic process is made up of the following four stages.

1. The working substance at a temperature 81 is placed in thermal contact with the heat
reservoir at a temperature 81 and undergoes a reversible isothermal expansion. In this
process the working substance absorbs a quantity of heat Q1 from the reservoir.
2. The working substance is then placed inside an adiabatic boundary and allowed to

48
undergo a reversible adiabatic expansion until its temperature falls to 82 , which is that
of the other reservoir.
3. The working substance is then placed in thermal contact with the reservoir at a
temperature 82 and undergoes a reversible isothermal compression at that tempera-
ture. During this process the working substance absorbs a quantity of heat Q2 from
the reservoir.
4. The reversible isothermal compression is stopped at such a state that, when the
working substance undergoes a reversible adiabatic compression it reaches its initial
state when its temperature has risen to el.

In the complete cycle an amount of work W is done on the working substance.


Two important results hold for the Carnot cycle and will be quoted here. Proofs may
be found in the standard textbooks. The first is known as Carnot's theorem and states
that:

No cyclic engine operating between two reservoirs can be more efficient than a Carnot
engine operating between the same reservoirs.

Since the Carnot engine operates reversibly all other types of engine must be irreversible.
The second result is the corollary to Carnot's theorem:

All reversible engines operating between the same two reservoirs have equal efficiencies.

From this latter result it follows that the efficiency of a reversible engine operating
between two reservoirs is independent of the nature of the working substance and must
be a function of the temperatures of the reservoirs only. If YJR is the efficiency of a
reversible (Carnot) engine operating between reservoirs at temperatures 8 1 and 82 ,

(5.4)

where f is a universal function of el and ez.

5.6 Universal The existence of a property - the thermal efficiency of a Carnot engine - that depends
Temperatures only on temperatures and not on the properties of any substance or class of substances
provides a convenient thermometric property. Equation (5.4) may be written

(5.5)

where <j> is another universal function of 8 1 and 82 • The negative sign is included because
Q1 and Q2 must be of opposite sign if the Kelvin statement of the second law is not to be
violated. As it stands, Equation (5.5) is not a basis for defining temperature, but it is
readily shown that <j>(Bu 82 ) is separable into the form F(8 1)/F(82 ) so that

(5.6)

where F is a universal function of temperature. Equation (5.6) may be used to define


temperature by choosing a function for F(8). Each of these functions of 8, which may be
called universal temperature functions, defines a property called a universal tempera-
ture. The only essential condition to be satisfied by the choice of F(8) is that the
isotherms be uniquely numbered. It is convenient if, further, the numerical value
increases with hotness, as with other scales of temperature, such as the ideal gas scale of
temperature.

5. 7 Thermodynamic The function adopted makes F(8) equal to T, that is, the universal temperature function
Temperature defines a temperature T by the equation

49
(5.7)

This temperature function is called the thermodynamic temperature function and


temperatures defined by this function are thermodynamic temperatures. The unit of
thermodynamic temperature is the kelvin, symbol K, and used already as the unit of the
ideal gas scale of temperature. The two scales are, in fact, identical, though thermodyn-
amic temperature is the fundamental concept and will be used exclusively in the rest of
this book.
The kelvin is the fraction 11273.16 of the triple point of water. When a Carnot engine
operates between a reservoir at a temperature T, measured in kelvins, and one at a
temperature of 273.16 K, the thermodynamic temperature Tis given by

T (K) = - 273.16 QIQ 3 (5.8)

where Q is the heat absorbed in each cycle at the temperature T and Q3 that absorbed at
the triple point temperature of water.
The smallest value that Q can take is zero and the corresponding value of Tis zero,
known as absolute zero. Absolute zero is that temperature at which no heat is absorbed
by a system when it undergoes a reversible isothermal process. This definition holds for
all systems and is an operational definition in terms of large-scale properties of the
system. Nothing is ascribed to the properties or behaviour of the constituent molecules,
atoms or ions.

5.8 Worked
Examples on Heat
Engines and
Thermodynamic
Temperature 5.5 The efficiency of a Camot engine depends only on the temperatures of the reservoirs and not on
the nature of the working substance. Derive an expression for the efficiency lJ of a Camot engine
operating between reservoirs at temperatures of T1 and T2 (T1 > T2 ).

Solution The system is the working substance of the Camot engine. The arrangement may be represented
by the diagram shown previously as Fig. 5.3, in which the horizontal lines represent the heat
reservoirs and the dashed line represents the boundary around the working substance. In each
cycle of operation let the working substance of the Carnot engine absorb heat Q 1 from the
reservoir at a temperature T1 , heat Q2 from the reservoir at temperature T2 and have work W done
on it. In Fig. 5.3 the arrows representing these interactions indicate the conventional positive
directions.
In each complete cycle there is no change in the internal energy of the working substance, so that

The thermal efficiency lJ is defined by

Substituting for W in this expression gives

lJ = - (- Ql - Qz)IQl
= (Ql + Qz)IQl
= 1 + (Q2/Ql)
Now, from the definition of thermodynamic temperature

50
and, substituting for Q 1 /Q2 ,

'11 = 1 + (- T2 1T1 )
= (Tt - T2 )1T1

5.6 A Carnot engine operates between a reservoir at a temperature of 500 K and one at a lower
temperature. In each cycle of operation the engine rejects 567 J of heat to this lower temperature
reservoir. If the efficiency of the engine is 0.35, calculate the work done on the working substance
in each cycle, the heat absorbed in each cycle from the high temperature reservoir and the
temperature of that reservoir.

Solution The system is the working substance of the Carnot engine. In each cycle let the system absorb heat
Q1 from the high-temperature reservoir, heat Q2 from the low-temperature reservoir and have
work W done on it. Since, in each complete cycle of operation there is no change in the internal
energy of the system, the first law gives

that is,

Ql + (-567) (J) + w= 0 (5.9)

From the definition of thermodynamic temperature

where T1 is the temperature of the high-temperature reservoir and T2 that of the low-temperature
reservoir. For the engine considered,

Q1 /(-567) (J) = -500 (K)IT2 (5.10)

The efficiency of the engine is given by

'11 = - W/Q!

which gives

0.35 = - WIQ 1 (5.11)

Substituting for W from Equation (5.11) into Equation (5.9) gives

Ql - 567 - 0.35 Ql = 0

or,

0.65 Ql = 567
that is

From Equation (5.11)

W = -0.35 X Q1

so that

W = -0.35 X 872 (J)


= -305 J

51
The value of T2 may now be obtained from the equation defining thermodynamic temperature:

Substituting the known values gives

872 (J)/(- 567) (J) -500 (K)/T2

or,

T2 = (500 X 567)/872
= 325 K

5. 7 A Carnot engine, operating between reservoirs at temperatures T1 and T2 , with T1 greater than T2 ,
is operated in reverse as a heat pump and, in each cycle, extracts a quantity of heat Q2 from the
reservoir at a temperature T2 • Derive an expression for the coefficient of performance of the heat
pump and calculate the amount of heat that it delivers to the reservoir at the higher temperature in
each cycle and the amount of work done on the working substance.

Solution The system is the working substance of the heat pump, which has thermal interactions with the two
heat reservoirs and a purely mechanical interaction with another part of the surroundings. The
conventional representation of the heat pump and its interactions is that given earlier in Fig. 5.3.
The arrows indicate the conventional positive directions of the interactions, and the dashed line the
boundary of the system.
In each cycle of operation there is no change in the internal energy of the working substance so
that, from the first law,

(5.12)

where Q1 is the heat absorbed per cycle by the working substance from the high-temperature
reservoir and W is the work done on the working substance in each cycle.
Since the cyclic process is reversible, the equation defining thermodynamic temperature must
hold, that is,

(5.13)

The coefficient of performance (C.o.P.) is defined by the equation

(C.o.P.)(h) = - Q1 1W (5.14)

From Equation (5.12)

so that, substituting into Equation (5.14),

(C.o.P.)(h) = Q1/(Q1 + Qz)


= 1/(1 + QziQ 1 ) (5.15)

Finally, substituting for Q2 /Q 1 from Equation (5.13),

(C.o.P. )(h) = 11[1 + (- T2 /T1)]


= T1/(T1 - T2)

As expected for a Carnot heat pump, the coefficient of performance depends only on the
temperatures of the reservoirs. The value of W may then be obtained from Equation (5.14)

W = - Q/(C.o.P.)
= - (Q1IT1)(Tl - T2)

52
5.9 Celsius The widely used Celsius temperature, symbol t, has as its unit the degree Celsius, symbol
Temperature °C, which is equal to the kelvin. The Celsius temperature scale is defined by the equation

t COC) = TIK- 273.15 (5.16)

thas no independent definition. Differences in temperature may be expressed in kelvins


or °C, but the former is preferred.

5.10 Problem- The approach to solving problems relating to closed systems may now be extended as in
Solving the following scheme.

1. Define the matter that comprises the system.


2. Define clearly the location and nature of the boundary of the system.
3. Decide what macroscopic coordinates are descriptive of equilibrium states of the
system, define the process and represent it on an appropriate coordinate diagram, if
that is helpful.
4. Identify the work interaction.
5. Identify the thermal interaction.
6. Write down the first law of thermodynamics for the process.
7. Write down the second law of thermodynamics for the process. At this stage this can
only be done for systems operating in a cyclic process between two heat reservoirs,
using the definition of thermodynamic temperature in the form

or in the form

11 (irreversible engine) :::; 11 (Carnot engine)

More generally useful forms of the second law will be considered in the next chapter.

5.11 Worked
Examples on Heat
Engines and the
Second Law of
ThermodynamiCS 5.8 A certain Carnot engine extracts 1500 J of heat in each cycle from a reservoir at a temperature of
500 K and rejects waste heat to a reservoir at a temperature of 350 K. Calculate the amount of
heat rejected in each cycle, the work done on the working substance in each cycle and the efficiency
of the engine.

Solution The system is the working substance of the Carnot engine. In each cycle of operation the working
substance absorbs a quantity of heat Q1 from the reservoir at the higher temperature T1 and a
quantity of heat Q2 from the reservoir at the lower temperature T2 • The work done on the working
substance in each cycle is W.
From the first law, the change in the internal energy !J..U of the system in any process is given by

!J..U =Q+W

However, in any number of complete cycles the change in the value of any thermodynamic
coordinate must be zero. In particular, there is no change in the value of the internal energy, that
is,

!J..U =0
53
so that

O=Q+W
Now

so that

(5.17)

The second law, applied to the Carnot engine, gives

(5.18)

Using the agreed sign convention, the given values are

Ql = 1500 J; T1 = 500 K; T2 = 350 K


Substituting these values in Equation (5 .18) gives

1500 (J)/Q 2 = -500 (K)/350 (K)

or,

Q 2 = - (1500 X 350)/500
= -1050 J
The minus sign indicates that the direction of heat transfer between the working substance and the
reservoir at a temperature T2 is from the system to the reservoir.
Substituting the values of Q1 and Q2 in Equation (5.17) gives

0 = +1500 (J) + (-1050 (J)) + w


or,

w= -450 J

The negative sign indicates that the work is done by the system on the surroundings. The thermal
efficiency 'YJ is given by

and substitution of the values gives

'Yj = - (- 450 (J))/(1500 (J))


= 0.3

or 'YJ is 30%. This is the maximum possible efficiency for a heat engine working between reservoirs
at temperatures of 500 K and 350 K. Even the ideal Carnot engine does not have an efficiency of
100%.

5.9 A certain mass of an ideal gas is contained in a cylinder by means of a well-fitting, frictionless
piston. Starting from an equilibrium state A, in which the pressure p exerted by the gas is 1.00 x
105 Pa and the volume V occupied by the gas is 2.00 x 10- 3 m\ the gas undergoes the reversible
cyclic process shown in Fig. 5.5. In equilibrium state B the pressure exerted by the gas is 2.00 x
105 Pa and in equilibrium state C the volume occupied by the gas is 3.28 x 10- 3 m3 . The process
represented by the line BC is a reversible adiabatic process and is described by the equation

p Vl. 4 = constant
54
Calculate the (pressure-volume) work done on the gas in one cycle of operation.

Pa ····---- B

Figure 5.5 The cyclic


process performed by the
given sample of ideal gas.

Solution The system here is the mass of gas and the boundary is the inner surface of the containing cylinder
and piston. The only type of work interaction between the system and its surroundings is
pressure-volume work.
For a fixed mass of gas that occupies a volume V under a pressure p, the work w done in an
infinitesimal reversible process may be written dW and is given by

dW = -pdV

In a finite reversible process, in which the volume of the system changes from V; to Vt. the work W
done on the system is given by

W =- Jvf pdV
V;
To evaluate the integral the relationship between p and V must be known.
Consider first the process represented by AC in Fig. 5.5. For this process V; = vf = VA and,
therefore, WAc is zero.
For the process represented by BC, the relationship between p and V is given by

where K is a constant.
Then,

Now, K may be written as p 8 V~ 4 or as PcV~ 4 • Appropriate substitution gives

WBc = -11(- 0.4)(pcVc- PBVB)


= (1/0.4)(pcVc- PBVB)

To use this expression Pc must be determined. Now

and substituting the known values gives

2.00 X 105 (Pa) X (2.00 X 10- 3 (m3))1. 4 = Pc X (3.28 X 10- 3 (m 3))1. 4

from which

Pc = 1.00 x lOS Pa

55
Therefore,

Wac = (1/0.4)(1.00 X 105 (Pa) X 3.28 X 10- 3 (m 3 ) - 2.00 X 105 (Pa) X 2.00 X 10- 3 (m 3 ))
= -180 J

The minus sign indicates that the energy transfer is from the system to the surroundings.
For the process CA the pressure remains constant with Pc = p A = 1.00 X 105 Pa. The work done
on the gas is WcA• given by

WcA =-IvA pdV = -pA IvA dV


Vc vc
-pA (VA- Vc) = PA CVc- VA)
1.00 X 105 (Pa) (3.28 X 10- 3 (m3 ) - 2.00 X 10- 3 (m 3 ))
128 J

This energy transfer is from the surroundings to the gas.


The total amount of pressure-volume work done on the system in the complete cycle is W, given
by

= 0 + (-180) (J) + 128 (J)


=-52 J

The net energy transfer is from the gas to the surroundings and of amount 52 J.

5.10 On a day when the outside temperature is 273 K, the interior of a certain building is maintained at
a temperature of 294 K by means of a heat pump extracting heat from the atmosphere. If the rate
of flow of heat from the building to the surroundings is 4.0 x 107 J hr- 1 , calculate the minimum
mechanical power needed to maintain the interior temperature constant at 294 K.

Solution Take the working substance of the heat pump as the system. The working substance has thermal
interactions with the building and the air outside. Both of these remain at constant temperature
and, in that sense, may be treated as reservoirs. There is also a work interaction with the motor
---------.---------2~K
driving the pump. The system and surroundings may, therefore, be represented by Fig. 5.6, in
which the symbols indicate the values of the interactions per unit time, and the arrows show the

,"
~-l~, '-+--:..:W_
conventional positive directions of the energy transfer. The horizontal lines represent the reser-
voirs and the dashed line the boundary of the working substance.
',_T;,~/ When conditions are steady, the rate at which the working substance rejects heat to the interior
of the building equals the rate at which heat is lost to the outside and is 4.0 x 107 J hr- 1 • Therefore,
-----------'----------273 K the value of Q1 is

Figure 5.6 Representation of Q1 = 4.0 X 107 (J hC 1 )/3600 (s) = 1.1111 X 104 (W)
a heat pump operating in a
cycle. Apply the first law to the operation of the heat pump for unit time, assuming that the working
substance completes a whole number of cycles in that time. Then, there is no change in the internal
energy of the working substance and

(5.19)

Substituting for Q1 and applying the sign convention gives

0 = -1.1111 X 104 (W) + Q2 + W (5.20)

The minimum value will be required when the heat pump operates reversibly, that is, when it is a
Carnot heat pump. Then, application of the equation defining thermodynamic temperature gives

56
or

-1.1111 x Ht (W)/Q2 = 294 (K)/273 (K)


giving

Q2 = 1.0317 X 104 W

Therefore,

W = -Q1- Qz
= -(-1.1111 X 104 (W)) -(1.0317 X 104 (W))
= 0.0794 X 104
= 794 w
Alternatively, combining Equations (5.19) and (5.20) gives

or,

W = - Q1[(T1 - T2 )1T1]
= - ( -1.1111 X 104 (W))(294 (K) - 273 (K))/294 (K)
= 794 (W)

5.11 Calculate the minimum amount of work that may be obtained from 2.0 kg of water at a tempera-
ture of 370 K when a reservoir at a constant temperature of 290 K is available. The specific heat
capacity at constant pressure for water may be taken as a constant and equal to 4200 J kg- 1 K- 1

Solution To obtain the maximum amount of work from a body and a reservoir a Carnot engine should be
operated in (infinitesimal) cycles between the body and the reservoir until the temperature
difference between them is zero and the Carnot engine is no longer able to operate. Take as the
system the working substance of the Carnot engine. The system has thermal interactions with the
body and the surroundings and a work interaction with other surroundings. For the temperature of
the body to remain sensibly constant while the system is absorbing heat from it the process must be
infinitesimal. The arrangement may be represented as in Fig. 5.7, where the quantities shown
relate to one infinitesimal (reversible) cycle of operation, and the arrows point in the conventional
positive direction for the various quantities. The dashed line indicates the boundary of the system.

=====:;=====• Body, temperature T


- ~~~~ '
Figure 5.7 A Carnot engine ( -- ,',---d::.:Wc:...
operating between a body at a ', ~ --/

temperature T and a reservoir --ld~o


at a temperature T0 • - - - - - L _ _ - - - - R e s e r v o i r , temperature T0

Let the temperature of the body, that is, of the mass of water, be T and that of the 'reservoir be
T0 • In one complete infinitesimal cycle of operation of the Carnot engine there will be no change in
the internal energy of the system, so that applying the first law to the system gives

dQ + dQ 0 + dW = 0 (5.21)

In any infinitesimal cycle the temperature of the body is sensibly constant so that it may be treated
as a reservoir. The second law may then be written for the system

dQ/dQ 0 =- TIT0 (5.22)

57
Combining Equations (5.21) and (5.22) gives

dQ + (- dQ X T0 fT) + dW = 0

or

dW = dQ [(T0 - T)fT] (5.23)

The absorption of an infinitesimal quantity of heat dQ by the system from the body does, of
course, produce an infinitesimal change dT in the temperature of the body. This temperature
change is given by

where m is the mass of the body and cP is the specific heat capacity at constant pressure of its
material. The minus sign occurs because the sign convention applies to the system and not the
body, that is, it considers the interactions from the point of view of the system.
Therefore,

dW = - mcP dT X (T0fT) + mcP dT

so that

where Ti is the initial temperature of the body.


Now, Ti = 370 K and T0 = 290 K, so that

W = - 2.0 (kg) X 4200 (J kg- 1 K- 1 ) X 290 (K) X ln(290 (K)f370 (K))


+ 2.0 (kg) x 4200 (J kg- 1 K- 1 ) X (290 (K) - 370 (K))
= 593 463.3 - 672 000
= -78 500 J

The negative sign indicates that the energy transfer is from the system to the surroundings.

5.12 A student claims to have constructed a prototype cyclic engine that operates between two
reservoirs at temperatures of 500 K and 350 K, respectively. In each cycle the engine extracts
6000 J of heat from the high-temperature reservoir, rejects 3900 J of heat to the low-temperature
reservoir and performs 2100 J of work on the surroundings. Would you be prepared to give
financial support to the development of this engine?

Solution The system is the working substance of the engine. To assess the feasibility of the prototype engine
its performance may be compared with that allowed by the first and second laws of thermodyn-
amics. In each cycle of operation the system absorbs a quantity of heat Q 1 from the reservoir at a
temperature T 1 and a quantity of heat Q2 from the reservoir T2 • It also has work W done on it by
the surroundings in each cycle. The engine may be represented as in Fig. 5.3, where the arrows are
drawn in the conventional positive direction for the respective interactions and it is assumed that T 1
is greater than T2 •
In one cycle of operation the change d U in the internal energy of the working substance is zero,
so that the first law becomes

dU = Q1 + Q2 + W = 0 (5.24)

For the prototype engine, Q 1 = +6000 J, Q2 = -3900 J and W = -2100 J. Substituting these
values in Equation (5.24) gives

+ 6000 (J) + (- 3900) (J) + (- 2100) (J) = 0

which shows that the engine satisfies the first law.


To consider the behaviour of the engine in terms of the second law, it is convenient to compare

58
the efficiency with that of an ideal (Carnot) engine operating between the same reservoirs. The
efficiency 11 of the prototype engine is given by

11 = -W/Ql
= - (- 2100 (J))/6000 (J)
= +0.35 = 35%
For a Carnot engine operating between the same reservoirs the efficiency lJc is given by

lJc = (T1 - T2 )1T1


= (500 (K) - 350 (K))/500 (K)
= 0.30 = 30%

Carnot's theorem, which is based on the second law of thermodynamics, states that no engine can
be more efficient than a Camot engine operating between the same two reservoirs. The prototype
engine that is claimed to have been constructed has an efficiency in excess of the theoretical limit!
Note that the Camot engine is not 100% efficient, although it is the ideal engine. The value of lJc
depends on the temperatures of the reservoirs and can only be 100% if T2 equals zero.

5.13 A fixed mass of an ideal gas undergoes the reversible cyclic process represented on the pressure p
against volume V graph of Fig. 5.8. Derive an expression for the efficiency of the process.

p
b
Po.Pc ---------.------------,.c

Figure 5.8. The reversible Pa ---------.a


cyclic process performed by
the fixed mass of ideal gas. V8 ,V0 v

Solution The system is the fixed mass of ideal gas and it undergoes three processes making up the cyclic
process. These processes are as follows.

1. An increase in pressure from Pa to Pb at constant volume Va. There is no work interaction


during this process and the ideal gas equation, written as p V IT = constant, shows that the
temperature must increase from Ta in the state represented by the point a in Fig. 5.8 to Tb in the
state represented by point b. The heat absorbed by the system in this constant-volume process is
Cv(Tb - Ta), where Cv is the heat capacity at constant volume. This is a positive quantity.
2. An increase in volume from Va ( = Vb) to Vc at constant pressure Pb· The ideal gas equation
shows that this process must result from the temperature of the gas changing from Tb to Tc. The
heat absorbed by the system in this isobaric process is Cp(Tc - Tb), where CP is the heat
capacity of the system at constant pressure. This is also a positive quantity.
3. The pressure changes from Pb ( = Pc) to Pa while the volume changes proportionately from Vc to
Va. In this process the temperature falls and heat is rejected to the surroundings: the sign of the
thermal interaction in this process is negative.

The thermal efficiency 11 is given by

11 = - W/Ql

where W is the work done on the system in each cycle and Q1 is the heat extracted from the
surroundings in each cycle, that is, those heat quantities having a positive sign.
The work done is given by

W = - J pdV = -I area of the triangle abc I


= - CVc - Va)(Pb - Pa)/2

59
The value of Q1 is given by

so that

Now, from the ideal gas equation,

so that

and

or, since Pc = Pb and Va = Vb,

and

Substituting in the expression for 11 gives

5.12 Exercises
5.1 A Carnot engine has an efficiency of 30% and rejects 200 J 5.5 Calculate the minimum amount of work that must be ex-
of heat in each cycle to a reservoir at a temperature of 300 K. pended to change the temperature of 5.0 kg of water from 290 K
Calculate the amount of heat absorbed in each cycle from the to 280 K at constant atmospheric pressure when the temperature
low-temperature reservoir, the temperature of that reservoir and of the surroundings is 295 K. For water the value of the specific
the work done on the working substance in each cycle. heat capacity at constant pressure may be taken as a constant
[+ 308 J; 462 K; - 108 J] equalto4200 Jkg- 1 K- 1 • [7390 J]

5.2 A refrigerator with a coefficient of performance of 3.5 5.6 For a given Carnot engine, is a greater increase in thermal
absorbs 80 J in each cycle from the freezer compartment. Calcu- efficiency achieved by increasing the temperature of the high-
late the minimum amount of work that must be done on the temperature reservoir by a given amount or by decreasing the
refrigerant in each cycle and the amount of heat expelled to the temperature of the low-temperature reservoir by an equal
surroundings. [ + 23 J; - 103 J] amount?
[Decrease the temperature of the low-temperature reservoir.]
5.3 A cyclic power plant operates in a Carnot cycle between
heat reservoirs at temperatures of 500 K and 300 K, respect- 5. 7 Two identical bodies, each of mass 5.0 kg and specific heat
ively. The work output of this device is used to provide the work capacity 800 J kg- 1 K - 1 , are initially at a temperature of 300 K.
input of a reversible heat pump operating in a reversed Carnot The temperature of one body is then raised to 700 K and a
cycle between reservoirs at temperatures of 290 K and 300 K, reversible heat engine operated between the two bodies until
respectively. Calculate the ratio of the heat delivered in each they reach a common temperature. Calculate this final common
cycle to the high-temperature reservoir of the heat pump to that temperature. [458 K]
extracted in each cycle from the high-temperature reservoir of
thepowerplant. [11.6:1]
5.8 A heat pump operating between reservoirs at temperatures
of 269 K and 294 K, respectively, requires an electrical input of
5.4 A certain bungalow requires an energy input of 4.0 kW to 1.4 kJ in each cycle. Assuming that the heat pump operates
maintain the inside temperature at 293 K when the outside reversibly, calculate the coefficient of performance of the heat
temperature is 273 K. Compare the costs of maintaining the pump, the amount of heat absorbed from the high-temperature
indoors temperature at 293 K: reservoir in each cycle and also the heat absorbed in each cycle
from the low-temperature reservoir.
(a) by passing an electric current through a resistor (electric [11.8;- 16.5 kJ; + 15.1 kJ]
'heating'); and
(b) using a Carnot heat pump that is driven by an electric motor
that has no losses.

[(cost of electric 'heating')/(cost of running heat pump) = 14. 7]

60
Topic Guide

6 Entropy
6.1 The Inequality of Clausius
6.2 Entropy
6.3 Worked Examples on
Entropy
6.4 Entropy and Work
6.5 Worked Examples on
Entropy and Work
6.6 The Entropy Form of the
First Law
6.7 Worked Examples on the
Entropy Form of the First
Law
6.8 Entropy and Irreversible
Processes
6.9 Worked Examples on the
Law of the Increase of
Entropy
6.10 Maxwell's Relations
6.11 Some Useful Mathematical
Relations
6.12 Problem-Solving
6.13 Worked Examples on
Entropy
6.14 Exercises

6.1 The Inequality An important result that follows from the Kelvin statement of the second law of
of Clausius thermodynamics is the inequality of Clausius. This states that when a closed system
undergoes any cyclic process, starting and ending in a given equilibrium state, then

~ q!T::; 0 (6.1)

where, in an elementary part of the cyclic process, the system absorbs an infinitesimal
quantity of heat q from a source at a temperature T.
When the cyclic process is, in fact, reversible, Tis also the (uniform) temperature of
the system. For a reversible cyclic process it may be shown that the relation (6.1) takes
the form

~ q/T = 0 (6.2)

a result known as the theorem of Clausius. Therefore, in relation (6.1) the equality holds
when the cyclic process is reversible and the inequality holds when any part of the cyclic
process is irreversible.

6.2 Entropy The theorem of Clausius may be used to show that in any reversible process the integral

(6.3)

has a constant value irrespective of the path taken between the initial equilibrium state i
and the final equilibrium state f. The subscript R indicates that the absorption of heat by
the system takes place reversibly.
The existence of a path-independent integral describing the behaviour of a system
indicates the existence of a property of the system, the change in the value of which in
any process depends only on the initial and final equilibrium states of the system. The

61
change in the value of this property as the state of the system changes from state i to state
f is given by the integral (6.3).This property of the system is called the entropy of the
system, symbol S, so that

!:J.S = St - Si = f. qR/T f. dQR/T


f

I
=
f

I
(6.4)

Since the heat transfer takes place reversibly, qR may be written dQR. The unit of
entropy is J K- 1 and, in an infinitesimal reversible process

dQR = TdS (6.5)

Since the change in entropy of a system is path-independent the entropy of a system is a


thermodynamic coordinate (or state variable or state function). Note that Equation (6.4)
only defines changes in entropy.
When a system undergoes a change from a given equilibrium state i to a given
equilibrium state f the change in the entropy of the system is always the same, whatever
the nature of the process. However, only when the process is reversible is it possible to
calculate the change in the entropy of the system by means of Equation (6.4). When the
process is irreversible it must be replaced for the purposes of calculation by a notional
reversible process joining the two equilibrium states. This notional calculation does not,
of course, give the change in the entropy of the surroundings. A separate notional
calculation is needed for that.
Equation (6.5) enables the principal heat capacities of the system to be expressed in
terms of the entropy of the system:

(6.6)

and

Cv = T(iJS/iJT)v (6.7)

6.3 Worked
Examples on
Entropy 6.1 A piece of ice of mass 0.500 kg at a temperature of 273.15 K and under a constant pressure of one
atmosphere melts to water at constant temperature and in so doing extracts 17 x 104 J of heat from
the atmosphere, which is at a temperature very slightly greater than that of the ice. Calculate (a)
the change in the entropy of the ice when it has all melted, (b) the corresponding change in the
entropy of the atmosphere and (c) the change in the entropy of the universe resulting from the
melting process.

Solution The system is the mass of ice that melts to form water. It melts because it absorbs heat from the
surrounding atmosphere. Since the temperature of the atmosphere is only very slightly greater
than that of the melting ice, this absorption of heat may be treated as a reversible process. Let the
amount of heat absorbed by the ice in melting be Q and let the melting take place at a temperature
T, which may be taken as the temperature of the ice, water and atmosphere. Then the change in
the entropy of the system is LiS( system), given by

ilS(system) = Q/T

Using the sign convention, Q = + 17 x 104 J and, as T = 273.15 K,

ilS(system) = 17 x 104 (J)/273.15 (K)


= 622.4 J K- 1

The heat is extracted reversibly from the (constant temperature) surroundings. Therefore, the
entropy change of the surroundings is ilS(surroundings), given by

62
~S(surroundings) = -17 x 104 (J)/273.15 (K)
= -622.4JK- 1
= - ~S(system)

The change in the entropy of the universe is ~Su, given by

~Su = ~S(system) + ~S(surroundings)

= 622.4 (J K- 1) - 622.4 (J K- 1)
=0

6.2 An insulated metal rod has each end in thermal contact with a heat reservoir. One reservoir is at a
temperature of 333 K and the other at a temperature of 288 K. When a steady state is reached it is
found that 200 J of heat are transferred along the rod in 10.0s. Calculate the change in the entropy
of each reservoir during this period and the corresponding change in the entropy of the universe.

Solution The system is the metal rod and the surroundings are the two reservoirs, as shown in Fig. 6.1. The
system absorbs heat from the reservoirs, but there is no other thermal interaction as the rod is
insulated. The process is the transfer of heat from the reservoir at a temperature T1 to that at a
temperature T2 when conditions are steady. This is an irreversible process as there is a finite
temperature gradient in the rod, so that the rod is not in an equilibrium state, but conditions are
steady so that the condition of the rod does not change with time. As the heat reservoirs have
effectively infinite thermal capacities, the thermal interactions with them may be treated as
reversible processes.
Insulation
Heat I Heat
Figure 6.1 The insulated reservoir reservoir
metal bar transferring heat Insulated rod
from the reservoir at a r, r.
temperature T 1 to that at a I
temperature T 2 • System boundary

In a timet let the system absorb a quantity of heat Q1 from the reservoir at a temperature T1 and
a quantity of heat Q 2 from the reservoir at a temperature T2 • Then, Q1 = + 200 J and Q2 =
- 200 J. Therefore, in the same timet, the reservoir at a temperature T1 absorbs a quantity of heat
- Q1 while the reservoir at a temperature T 2 absorbs a quantity of heat - Q 2 • Then, the change in
the entropy of the high-temperature reservoir in the time t is ~S1 , given by

while the change in the entropy of the reservoir at a temperature T2 is ~S2 , given by

Then, since T1 = 333 K and T2 = 288 K,

~S 1 = -200 (J)/333 (K)


= -0.6006 J K- 1

and

~S2 = - (- 2oo) (J)/288 (K)


= +0.6944 J K- 1

The change in the entropy of the universe is ~Su, given by

~su = ~s1 + ~Sz

= -0.6006 (J K- 1 ) + 0.6944 (J K- 1 )
= +0.0938JK- 1

63
6.3 A copper sphere of mass 0.500 kg and at a temperature of 363 K is placed in a large lake, the
temperature of which is 283 K, and reaches thermal equilibrium. Calculate the change in the
entropy of the sphere produced by the process, the corresponding change in the entropy of the lake
and the change in the entropy of the universe.
Assume that the specific heat capacity at constant pressure of copper is independent of tempera-
ture over the range 200 K to 400 K, having the value 386 J kg- 1 K- 1 •

Solution The system is the copper sphere which undergoes a thermal interaction with the water of the lake.
In this process the temperature of the sphere changes from an initial equilibrium value of 363 K to
a final equilibrium value of 283 K, while the temperature of the lake remains at 283 K. The initial
finite temperature difference between the system and its surroundings ensures that the energy
transfer occurs through an irreversible process. Since entropy is a state function the change in the
entropy of the system depends only on the initial and final equilibrium states but, to calculate the
change in the entmpy of the system arising from this irreversible process, the real process must be
replaced by a notional reversible process linking the same initial and final equilibrium states. For
the change produced in the copper sphere this may be achieved by the use of an infinite array of
heat reservoirs, the temperature of the coolest being 283 K and that of the hottest being 363 K.
The other reservoirs in the array have temperatures such that the temperature of each differs from
that of its neighbours by the infinitesimal amount dT. Part of the array of reservoirs is shown in
Fig. 6.2.

System

tdO

Figure 6.2 Part of the 1


infinite array of heat T
reservoirs.

To produce the change of state of the copper sphere in a reversible manner, the sphere is first
placed in thermal contact with the reservoir at a temperature of 363 K and allowed to come to
equilibrium. When this equilibrium is reached the sphere is in an equilibrium state identical to its
initial equilibrium state in the real process. The sphere is then placed in thermal contact with the
reservoir whose temperature is only infinitesimally less than 363 K. In reaching thermal equilib-
rium with this reservoir the sphere undergoes an infinitesimal thermal interaction and is always
infinitesimally close to a state of equilibrium. This procedure is repeated with each of the other
reservoirs of the array in their respective turn, until the reservoir is at a temperature of 283 K.
When the sphere reaches equilibrium while in thermal contact with this reservoir it is in an
equilibrium state identical to that at the end of the real process. At no stage in this notional process
is the temperature of the sphere different from that of the reservoir with which it is in contact by
more than an infinitesimal amount. Therefore, the process is one in which the heat transfer takes
place reversibly.
When the system, that is, the sphere, is placed in thermal contact with the reservoir at a
temperature T it absorbs reversibly a quantity of heat dQ to reach thermal equilibrium and the
change in the entropy of the system resulting from this infinitesimal process is dS, given by

dS(system) = dQ/T
Since

where m is the mass of the system and cP is its specific heat capacity at constant pressure,

In the complete process, in which the temperature of the system changes from Ti to Tt> the change
in the entropy of the system is AS, given by

AS= mcP fTf

T;
dTIT

= mcP 1n(Tr1Ti)
64
Now, for the copper sphere, m = 0.500 kg; cP = 386 J kg- 1 K-\ Ti = 363 K; and Tt = 283 K.
Therefore,

~S(system) = 0.500 (kg) X 386 (J kg- 1 K- 1 ) X ln(283 (K)/363 (K))

= 0.500 X 386 X ( - 0.249)

= -48.1 J K- 1

The lake acts as a reservoir and, therefore, the absorption of heat by the lake may be treated as a
reversible process. The amount of heat that is absorbed by the copper sphere as its temperature
changes from Ti to Tt is given by

so that the amount of heat absorbed by the lake is given by

Therefore, the change in the entropy of the surroundings, that is, of the lake, is ~S(surroundings),
given by

~S(surroundings) = - m cP (Tt - Ti)!Tt

since the temperature of the lake is Tf. Substitution gives

~S (surroundings) = - (0.500 (kg) x 386 (J kg- 1 K- 1 )) x (283 (K) - 363 (K))/283 (K)
= 54.6 J K- 1

The change in the entropy of the universe is ~Su, given by

~Su = ~S(system) + ~S(surroundings)

= -48.1 (J K- 1 ) + 54.6 (J K- 1 )

= 6.5 J K- 1

6.4 Calculate the change in the entropy of the universe when a system consisting of 11 moles of
molecules of an ideal gas undergoes a free expansion from an initial equilibrium state i, characte-
rised by pressure Pi, volume Vi and temperature Ti, to a final equilibrium state f, with pressure Pt,
volume Vt and temperature Tt.

Solution The system is the mass of ideal gas undergoing the free expansion. A general free expansion is
characterised by the absence of both work and thermal interactions, so that the internal energy of
the system has the same value in both the initial and final equilibrium states of the system. The free
expansion process itself is highly irreversible and cannot be described by the thermodynamic
coordinates of the system. To obtain information about the values of quantities such as the entropy
in the initial and final equilibrium states, the real irreversible process must, for the purposes of
calculation, be replaced by a notional reversible process linking the initial and final equilibrium
states.
For an ideal gas there is no change in temperature as the result of a free expansion, that is,
Tt = Ti. Therefore, a suitable notional reversible process for calculating the change in the entropy
of the system is a reversible isothermal expansion at a temperature Tb which ensures that ~ U is
zero.
Applying the first law to the notional process gives

~U= 0= Q +W
where Q is the heat absorbed by the system during the notional process and W is the work done on
it. When a gas changes its volume from vi to vf the work w done on the gas is given by

65
where p is the pressure exerted by the gas when it occupies a volume V. Since the system is an ideal
gas, the equation of state is

pV = nRT

rt
so that

W - (nRT)V) dV
V;

Therefore,

Q = - W = + nRTi ln(Vf/Vi)

The change AS(system) in the entropy of the gas is, therefore, given by

AS(system) = Q/Ti = [nRTi ln(Vt!VJ]!Ti

But,

so that

AS(system) = [Pi Vi ln(Vt/Vi)]/Ti

AS( system) is positive: the entropy of the gas has increased.


For the real process the change in the entropy of the surroundings is zero so that the change in
the entropy of the universe is equal to the change in the entropy of the gas.

6.4 Entropy and Consider a Carnot engine operating between reservoirs at temperatures T and T0 • In one
Work cycle of operation let an amount of work W be done on the system by the surroundings.
W is given by

W = -(Q + Qo) (6.8)

where Q and Q0 are the quantities of heat absorbed in each cycle by the working
substance from the reservoirs at temperatures T and T0 , respectively. From the definition
of thermodynamic temperature

so that

W = -[Q - (Q T0 1T)]
= -(QIT)(T- To)

= -~S(T- T0 ) (6.9)

where ~S is the change in the entropy of the working substance that occurs in the
reversible isothermal process at the temperature T, that is, occurs by virtue of the heat
absorbed at the temperature T. For an engine working in a forward sense W is negative
and Q is positive. Equation (6.9) shows that a unit quantity of entropy is capable of doing
a unit quantity of work when it falls reversibly through a unit temperature difference.

66
6.5 Worked
Examples on
Entropy and Work 6.5 A Camot engine operates between heat reservoirs at temperatures T and T0 , respectively. The
value of Tis 560 K and, in its interaction with that reservoir, the working substance absorbs
21 000 J of heat in each cycle. If the work output of the engine in each cycle is 5025 J, calculate the
value of T0 •

Solution The system is the working substance of the Camot engine. It undergoes a thermal interaction with
each of the reservoirs and a work interaction with some other part of the surroundings. Let the
system absorb a quantity of heat Q during its interaction with the high-temperature reservoir, the
temperature of which is T. The change in the entropy of the system resulting from this interaction
with the reservoir at a temperature T is !:iS, given by

!:iS= Q/T

Therefore,

!:iS = 21 000 (J)/560 (K)


= 37.5 J K- 1

If the temperature of the cooler reservoir is T0 , the work W done on the system in each cycle of
operation of the Camot engine is given by

W = -!:l.S(T- T0 )

Now W = -5025 J since the net work is done on the surroundings. Therefore,

-5025 (J) = -37.5 (J K- 1 )(560 (K) - T0 )

giving

To 560 (K) - (5025 (J)/37.5 (J K- 1 ))


426 K

6.6 Calculate the maximum amount of work that may be obtained from 2.0 kg of water at a tempera-
ture of 370 K when a reservoir at a temperature of 290 K is available. Take the specific heat
capacity of water at constant pressure to be a constant equal to 4200 J kg- 1 K- 1 •

Solution Let the fixed mass of water be called the body. To obtain the maximum amount of work from the
body a Carnot engine is operated between the body and the reservoir. Since the Carnot engine
must operate reversibly, the absorption of heat by the working substance from both the body and
the reservoir must take place reversibly. This can only be the case if the temperature of the body
remains sensibly constant during the thermal interaction with the working substance. This means
that the amount of heat absorbed by the working substance from the body in each cycle must be
infinitesimal, that is, the Carnot engine must operate in infinitesimal cycles.
The system in this problem must be the working substance of the Carnot engine. It is to the
system that the sign convention relating energy transfers during the interactions applies. The
arrangement of body, reservoir and working substance may be represented as in Fig. 6.3, in which
the dotted line marks the boundary of the system and the narrow rectangle represents the body.
The arrows indicate the conventional positive directions of the energy transfers.

c::===:::::;:====:::J!Body, temperature T
__ id__a_
/' ', dW
Figure 6.3 A Carnot engine I
\
~
I
operating in infinitesimal ' /

cycles between a body and a --t~:


reservoir. -------'-----Reservoir, temperature T0

67
In one infinitesimal cycle let the system absorb a quantity of heat dQ from the body, which is at a
temperature T. Then the work done on the system in that particular complete infinitesimal
reversible cycle is dW, given by

dW = -dS (T - T0 )
where dS is the change in the entropy of the system resulting from the interaction with the body
and T0 is the temperature of the reservoir.
Now,

dS = dQ!T

where dQ is the heat absorbed by the system from the body when the latter is at a temperature T.
When the system absorbs a quantity of heat dQ from the body there is a temperature change d Tin
the body, given by

dQ = -m cP dT

where m is the mass of the body and cP is its specific heat capacity at constant pressure. This gives

dS = -m cP dT

and

dW = -( -m cP dTIT)(T - T0 )
= m cP dT - (m cP T0 /T)dT

Continued repetition of the cyclic process gradually reduces the temperature of the body until the
value reaches T0 • There is then no temperature difference to operate the Carnot engine and the
operation ceases. Therefore, the total amount of work W done on the system is given by

W = JdW =J
~
mcP dT-
~
JT; (me T0 /T) dT
T; p

where T; is the initial temperature of the body. Then,

W = mcp (T0 - T;) -mcpTo ln(T0 1T;)


= 2.0(kg) X 4200 (J kg- 1 K- 1) X (290 (K) - 370 (K)) - 2.0 (kg)
x 4200 (J kg- K-1 1) X 290 (K) X ln(290 (K)/370 (K))
= 593 463.4- 672 000
= -78 500 J

The negative sign indicates that the energy transfer in the form of work is from the system to the
surroundings.

6.6 The Entropy When a closed system undergoes any infinitesimal process between an initial equilibrium
Form of the First state and a final equilibrium state, the change dU in the internal energy is given by

Law dU = q + w (6.10)

where q is the heat absorbed by the system in the infinitesimal process and w is the work
done on the system. If the process is reversible

q = dQ = TdS

where T is the temperature of the system and S is its entropy. Further, if the system is a
closed hydrostatic system, exerting a pressure p and occupying a volume V, in a
reversible process
68
w = dW = -pdV
Substituting for q and win Equation (6.10) gives

dU = TdS- pdV (6.11)

From its derivation it might seem that Equation (6.11) applies only to reversible pro-
cesses. However, all the quantities appearing in Equation (6.11) are, in fact , state
functions (thermodynamic coordinates). Therefore, when the system changes from an
equilibrium state ito an equilibrium state f, the changes in U, T , S, p and V depend only
on i and f and not on the nature of the process. It follows that Equation (6.11) applies to
any process between the equilibrium states i and f. However, only when the process is
reversible is it possible to equate TdS to q and - pdV to w. Equation (6.11) is a very
important equation in thermodynamics and is often called the entropy form of the first
law, though it is sometimes given the title 'the central equation of thermodynamics'.
From Equation (6.11) it follows that

Cv = T(CJS!iJT)v = (iJU!iJT)v
and

where Cv is the heat capacity at constant volume and CP is the heat capacity at constant
pressure.

6.7 Worked
Examples on the
Entropy Form of
the First Law 6.1 One mole of molecules of an ideal gas is contained in a cylinder fitted with a frictionless piston.
The gas is caused to expand reversibly from an initial volume of 1.0 x 10- 3 m 3 to a final volume of
2.0 x 10- 3 m3 , while its temperature is maintained constant. Calculate the change in the entropy of
the gas that results from the reversible isothermal expansion.

Solution The system is the fixed mass of ideal gas. It undergoes a reversible isothermal process in which
there is both a work interaction and a thermal interaction with the surroundings. The system and
Cylinder its surroundings may be represented as in Fig. 6.4, in which the dashed line marks the boundary of
the system.
Apply the entropy form of the first law to an infinitesimal part of the process. Then ,
Piston
dU = TdS- pdV

Figure 6.4 A system where U is the internal energy of the system, p its pressure and V its volume when its temperature
consisting of a fixed mass of is T. When an ideal gas undergoes a reversible isothermal process there is no change in the internal
gas contained in a cylinder by energy of the gas; that is, dU is zero. Then,
a well-fitting frictionless
piston.
TdS = pdV

or

dS = (p!T)dV

The equation of state of the system is the ideal gas equation

since the system consists of one mole of molecules. Therefore,

69
and

St - S; = R fvmf
' dV/Vm
vm,i
= R ln(Vm,f/Vm)

= 8.315 (J K- 1 mol- 1) x ln(0.002 (m3 )/0.001 (m3 ))


= 5.76 J K- 1 mol- 1

6.8 A system consists of n moles of molecules of an ideal gas with constant principal molar heat
capacities. The system is caused to expand from an initial equilibrium state characterised by
pressure P; and volume V; to a final equilibrium state characterised by pressure Pt and volume Vf.
Calculate the change in the entropy of the system produced by this process.

Solution The system is the fixed mass of ideal gas, which may be pictured as contained in a cylinder with
diathermic walls by means of a well-fitting piston. The system undergoes both work and thermal
interactions with the surroundings. Let the temperature of the system in the initial equilibrium
state be T; and in the final equilibrium state be Tt. The equation of state of an ideal gas ensures that

The nature of the process by which the system changes from the initial equilibrium state to the final
equilibrium state is not specified and is, of course, irrelevant to the change in the entropy of the
system that is produced by the process, since entropy is a state function. However, to calculate this
change in entropy, it is necessary to consider a notional reversible process that brings about the
same change. Apply the entropy form of the first law to an infinitesimal part of this notional
process. Then,

dU = TdS- pdV

or,

TdS = dU + pdV

where p is the pressure exerted by the system when it occupies a volume Vat a temperature T, U is
its internal energy and Sits entropy. For n moles of an ideal gas

dU = Cv dT = nCv.m dT
and

pV = nRT

Substituting in the expression for TdS gives

TdS = nCv.m dT + (nRTIV) dV

or,

dS = (nCv,miT)dT + (nR/V) dV
Thus far the notional reversible process has not had to be specified in detail, but this is now
necessary so that the integration may be performed. Any convenient path may be chosen. Choose
a process in which the temperature of the system is changed from T; to Tt while the volume is held
constant at V;, followed by a process in which the volume changes from V; to Vt while the
temperature is held constant at Tt. This allows the two terms in the expression for dS to be
integrated separately to give

70
where Sf is the value of the entropy of the system in the final equilibrium state and Si is that in the
initial equilibrium state. Now,

so that, in terms of the given quantities,

6.8 Entropy The inequality of Clausius can be used to show that when a system undergoes any process
and Irreversible from an initial equilibrium state ito a final equilibrium state f, the change in the entropy
sf - si satisfies the condition
Processes
(6.12)

where q is the heat absorbed by the system in an infinitesimal part of the process from a
source at a temperature T. As might be expected from Equation (6.3), the equality sign
in relation (6.12) holds for reversible processes and the inequality sign for irreversible
processes.
When the system under consideration has an adiabatic boundary, q is equal to zero
and relation (6.12) becomes

(6.13)

or, in an infinitesimal process, dS ~ 0. This important result is known as the law of the
increase of entropy. It states that the entropy of a thermally isolated system can never
decrease. When such a system undergoes a reversible process its entropy remains
unchanged; when it undergoes an irreversible process its entropy always increases. For a
closed system that is isolated from its surroundings, that is, a system that cannot undergo
either a thermal interaction or a work interaction, relation (6.13) must hold but, in
addition, the internal energy is unchanged by the process, whether the process is
reversible or irreversible.
When two systems A and B interact only with each other, relation (6.13) becomes

(6.14)

when all processes are reversible and

f'l.SA + f'l.SB > 0 (6.15)

when there is any irreversibility present.

6.9 Worked
Examples on the
Law of the Increase
of Entropy 6.9 A body A of constant heat capacity at constant pressure CP is in equilibrium at a temperature TA
and a similar body B is in equilibrium at a temperature TB. The two bodies are then placed in
thermal contact under conditions of constant pressure. Calculate the final equilibrium temperature
Te of the two bodies and the change in the entropy of the universe brought about by the process.
If the original bodies are brought to a common temperature by operating a Carnot engine
between them, rather than by placing them in direct thermal contact, calculate the final equilib-
rium temperature and the work done on the working substance of the Carnot engine.

71
Solution The two bodies A and B interact only with each other. Therefore, they may be treated as if they
are enclosed by a rigid adiabatic boundary, as in Fig. 6.5. When the two bodies are brought into
..,., _ - - - - _..... Rigid adiabatic thermal contact, the resulting process produces no change in the total internal energy of the two
I
,. ,.----- boundary
\
bodies. Therefore, if Q is the total amount of heat absorbed by the two bodies in the interaction
I(£) @1 and W is the total amount of work done on the two bodies, application of the first law gives
\ I
'....
------ ....."" Q + W= 0
Figure 6.5 Two interacting
bodies A and B. Now,

where QA is the heat absorbed by body A when the two bodies are in thermal contact and QB is the
heat absorbed by body B. Then, assuming that the work interaction may be neglected,

Now,

and

Therefore,

or,

To determine the change in the entropy of the two bodies, the real irreversible process must be
replaced by a notional reversible process. Since the real process involves a heat transfer and the
corresponding changes in temperature, the notional change is effected by means of an infinite array
of heat reservoirs, as shown in Fig. 6.2. In the present example, the temperatures of these
reservoirs range from TA to TB. When body A is at a temperature T- dT and is then placed in
thermal contact with the reservoir at a temperature T, it absorbs a quantity of heat dQ given by

and its entropy changes by dSA• given by

dSA = dQ!T = ( Cp/T) dT

In the complete notional process the change in the entropy of body A is ASA• given by

Similarly, for body B,

The change in the entropy of the universe ASu is then given by

Substituting for Te gives

ASu = Cp{ln[(TA + TB)/2TA] + ln[(TA + TB)/2TB]}


Cp{ln[(TA + TB)/2(TATB) 112] 2 }
= 2 cp ln[(TA + TB)/2(TATB) 112]

72
The bodies A and B may be brought to a common temperature TE by operating a Carnot engine
between them. As the extraction of heat from the finite bodies causes a change in their tempera-
tures, this Carnot engine must operate in infinitesimal cycles to ensure that all heat transfers take
place reversibly. Take the working substance of the Carnot engine as the system. The arrangement
may be represented as in Fig. 6.6, where, in each infinitesimal cycle of the Carnot engine, the
working substance absorbs heat dQA from body A, heat dQs from body B and has work dW done
on it.

c=========;=======~IBodyA

--
-- t!d.9: . . .
I
/ '\ dW
I -T---"-'-'

Figure 6.6 Carnot engine ' /

operating between two bodies '---~~:--


of finite heat capacity. c=================~!BodyB

In a complete cycle of operation of the Carnot engine there is no change in the internal energy of
the system, so that

When the temperature of body A increases by dT the heat absorbed by the system is dQA, given by

Similarly, for body B

Then,

- CP dT - CP dT + dW = 0

and

or,

giving

In a complete number of cycles of operation the system will suffer no net change in its entropy,
since entropy is a state function. In its interaction with body A the change in the entropy of the
system is given by

Similarly, the change in the entropy of the system resulting from its interaction with body B is
given by

Since the change in the entropy of the system is zero when a whole number of cycles is considered,

73
Then,

giving

or,

Substituting for TE in the expression for W gives

6.10 A cyclic reversible engine operates between reservoirs at temperatures of 400 K, 350 K and 250 K,
respectively. In each cycle the engine extracts 1500 J of heat from the reservoir at 450 K and does
150 J of work on the surroundings. Calculate the amount of heat absorbed by the engine from the
reservoirs at temperatures 350 K and 250 K in each cycle.

Solution Take the working substance of the reversible engine as the system. In each cycle of operation there
will be no change in the internal energy or in the entropy of the system, both these quantities being
state functions. Let the temperatures of the three reservoirs be T1 , T2 and T3 , respectively, and, in
each cycle, let the amount of heat absorbed by the system from each reservoir be Q1 , Q2 and Q 3 ,
respectively. Then, applying the first law to one complete cycle of operation of the engine gives

(6.16)

where W is the work done on the system in each cycle. Since all processes are reversible, the
change in the entropy of the system in each cycle, f).S, is given by

But in one complete cycle f).S is zero, so that

(6.17)

Now,

T1 = 400 K; T 2 = 350 K; T3 = 250 K; Ql = 1500 J; and

w= -150 J.

Therefore, from Equation (6.16),

1500 (J) + Q2 + Q3 + (-150) (J) = 0

and from Equation (6.17)

1500 (J)/400 (K) + Q/350 (K) + Q 3/250 (K) =0


Therefore

Q2 + Q3 = 150 - 1500 = -1350 (J)

and

3.75 + Q2/350 + Q/250 =0


Solving these two simultaneous equations gives

74
Q3 = 93.8 J and Q2 = -1443.8 J
The system extracts 93.8 J of heat from the reservoir at a temperature of 250 K and rejects
1443.8 J of heat to the reservoir at a temperature of 350 K.

6.10 Maxwell's In theoretical discussions of the behaviour of systems, the entropy is a useful quantity
Relations when heat transfer is considered because it is a state function, whereas quantity of heat is
not. However, to relate changes in the entropy to quantities that are determined
experimentally, it is necessary to relate changes in the entropy to changes in the values of
the primitive coordinates. This may be done by means of the equations known as
Maxwell's relations. These equations do not describe processes, but give relationships
between the entropy and the primitive coordinates that must hold when the system is in
any equilibrium state. For a closed hydrostatic system Maxwell's relations are

(CJV/CJT)p = -(CJSICJp)r (Ml)

(CJV!dS)p = ( CJT!CJp )s (M2)

(CJp!CJT)v = ( CJS/CJV)r (M3)

(CJTICJV)s = -(CJp/CJS)v (M4)

The numbering follows that used by Maxwell in his book The Theory of Heat.

6.11 Some Useful Thermodynamics is concerned with the equilibrium states of systems. Such states are
Mathematical characterised by the values of a small number of parameters known as the thermodyn-
amic coordinates. Each equilibrium state is represented by a unique point in the appropri-
Relations ate coordinate space and the totality of equilibrium states is represented by a continuous
surface in this coordinate space. The equation of this surface is the equation of state.
Each reversible process is represented by a continuous line on the surface in coordinate
space and relationships between the coordinates may be obtained by applying some
theorems of the partial differential calculus to reversible processes. The two most useful
theorems are the reciprocal theorem and the reciprocity theorem.
Most of the systems considered in this book have their equilibrium states described by
three coordinates, two of which are independent while the third is prescribed by the
equation of state. For example, the equilibrium states of a fixed mass of gas are
represented by a surface in pressure-volume-temperature space.
For a particular thermodynamic system with two independent coordinates, let the
coordinates be x, y and z, related by the equation of state, which may be written

F(x,y,z) = 0

Then, it may be shown that

(ax!az)y = l!(CJz!CJx)y

This is the reciprocal theorem. Further, it may be shown that

(CJx!CJy)z (CJy!CJz)x (CJz!CJx)y = -1

a result known as the reciprocity theorem.

6.12 Problem- The basic approach to the solution of problems in thermodynamics may now be given in
Solving a fairly complete form.

75
1. Identify the system and its surroundings and, where it is helpful, draw a diagram
representing the system and its surroundings.
2. Decide what macroscopic coordinates are descriptive of the system, define the process
and, where it is helpful, represent it on a coordinate diagram.
3. Identify any interactions.
4. Write down the first law for the process, possibly starting with the form for an
infinitesimal part of the process.
5. Write down the second law for the process or an infinitesimal part of it. This could
take the form

(a) the entropy form of the first law:

dU = TdS- pdV

(b) the law of the increase of entropy:

for a thermally isolated system, together with the result

~U= 0

for a completely isolated system.


(c) the defining equation for entropy:

~QJT; = 0

for reversible interactions between a series of bodies that are isolated from the
rest of the universe.
(d) the defining equation for thermodynamic temperature:

6. When none of the above approaches is effective, the quantities of interest should be
considered, appropriate reciprocal or reciprocity relations should be written down
and, if necessary, transformed using Maxwell's relations. For example, if the depen-
dence of volume on temperature at constant pressure is of interest, the partial
derivative

(iJV/iJT)p

makes a possible starting point. This can be used to form the reciprocity relation

(iJV!iJT)p = -(iJp!iJT)v (iJV!iJp)T

6.13 Worked
Examples on
Entropy 6.11 A resistor of resistance 10.0 0 is submerged in a stream that has a temperature of 305 K. A current
of 0.500 A is passed through the resistor for 10.0 s. Calculate the change in the entropy of the
resistor and of the stream.

Solution The system is the resistor. It has electrical work done on it by an external source and undergoes a
thermal interaction with the stream. When the current flows through the resistor, energy is
transferred to the resistor and its temperature starts to rise. As soon as the temperature of the
resistor rises above that of the stream there is a heat flow from the resistor to the stream. When the
current is stopped an equilibrium state is reached in which the resistor is in the same equilibrium
76
state as it was initially: the net effect of the process is to transfer a quantity of energy from the
electrical source to the stream. This transfer is in two stages: (a) the performance of electrical work
on the system, and (b) the transfer of heat from the system to the surroundings.
The net change in the entropy of the system, that is, of the resistor, is zero, as its state is
unchanged by the process.
The stream behaves as a reservoir and absorbs the heat in an effectively reversible manner. The
heat Q transferred to the surroundings is equal to the work done on the system and is given by

where I is the current, which flows for a time t in the resistor of resistance R. Then

Q = (0.500?(A) 2 X 10.0(!1) X lO.O(s)


= 25.0 J

The change in the entropy of the stream, which is at a temperature T, is !:!..S, given by

!:!..S = Q!T = 25.0 (J)/305 (K)


= 0.082 J K- 1

6.12 Two identical bodies, of constant heat capacity at constant pressure CP, are in equilibrium at a
temperature Ti. A Carnot refrigerator then operates between the two bodies until the temperature
of one of them has fallen to T2 • Calculate the temperature achieved by the other body and show
that the total amount of work W needed to bring about the change is given by

Solution The system is the working substance of the refrigerator, which undergoes thermal interactions with
the two bodies, A and B, say, and has work done on it by another part of the surroundings. Since
the temperatures of A and B change during the operation of the refrigerator, it is necessary for the
refrigerator to operate in infinitesimal cycles for the heat transfer to be reversible. In each
infinitesimal cycle, let the heat adsorbed by the system from the body A be dQA, that from the
body B be dQ 8 and let the work done on the system be dW. Then, applying the first law to one
cycle of operation gives

(6.18)

since there is no change in the internal energy of any system in one complete cycle.
Applying the second law to one cycle of operation of the system gives

(6.19)

since there is no change in the entropy of any system in a cyclic process. When the system absorbs
heat dQA from body A, the temperature of body A changes by dT, given by

Similarly,

Then Equation (6.19) becomes

or,

(6.20)

Let the final equilibrium temperature of body A be T2 and that of body B be T1 • Then, integrating
Equation (6.20) gives

77
or,

CP ln(T/Ti) = - CP ln(T2 /Ti)


= CP ln(TJT2 )
giving

or,

From Equation (6.18)

or,

giving

Then,

6.13 A certain ideal gas has a molar heat capacity at constant volume Cv,m• given by

Cv,m =A+ BT

where A and B are constants. n moles of molecules of this gas are in an initial equilibrium state
characterised by a volume Vi and a temperature T;. The state of the gas is then changed to an
equilibrium state in which it occupies a volume Vr at a temperature Tr. Calculate the change in the
entropy of the gas resulting from this process.

Solution The system consists of n moles of ideal gas that is able to undergo both work and thermal
interactions with the surroundings. The system is initially in an equilibrium state and the process
leaves it in a final equilibrium state. Since entropy is a state function, the change in the entropy of
the system AS is independent of the nature of the process. Therefore, for the purposes of
calculation, any convenient notional process linking the initial and final equilibrium states may be
considered. It is convenient to choose as the notional process one that is a reversible path
consisting of two steps:

(a) In step 1 the system changes its volume from V; to Vr while its temperature remains constant at Ti.
(b) In step 2 the temperature of the system is changed from Ti to Tr while its volume remains
constant at Vr.

The entropy form of the first law may be applied to both steps in the notional reversible process
used for calculation. Then,
dU = TdS- pdV

where U is the internal energy of the system and pis its pressure. For an arbitrary mass of any ideal
gas (Joule's Law, p. 31)

78
where Cv is the heat capacity at constant volume. Therefore, for a system consisting of n moles of
ideal gas,

dU = nCv,m dT
Then,

TdS = dU + pdV
= n Cv,m dT + pdV
The equation of state of an ideal gas is

pV = nRT
giving

TdS = nCv,m dT + nRT dV/V


For step 1 ofthe notional process T = Ti and dT is zero. Therefore, the change in the entropy ot
the system is L1S1 , given by

= J nRdV/V
Vf
L1S 1
Vj

= nR ln(Vf/Vi)

For step 2 of the notional process V = Vt and dV is zero. Then,

Therefore, the total change in the entropy of the system is L\S, given by

6.14 Show that the area enclosed on a temperature-entropy graph for a reversible engine working in a
cycle is equal to the magnitude of the net amount of work done on the working substance. Draw
the shape of the graph for a Carnot engine operating between reservoirs at temperatures of T1 and
T2 , respectively (T1 > Tz).

Solution The system is the working substance of the reversible cyclic engine. Since the engine is reversible,
the entropy form of the first law may be interpreted term by term, that is, in the equation

dU = TdS- pdV

TdS represents the heat absorbed by the system in the infinitesimal process in which the internal
energy changes by dU, and - pdV is the work done on the system in the same process. Here Tis
the temperature of the system, p the pressure and V the volume occupied by the system. In a cyclic
process this equation becomes

~ dU = ~ TdS - ~ pdV
Since U is a state function

~ dU = 0

so that

79
Now, ~ TdS is the area enclosed by the path of the cycle in the temperature-entropy plot and - ~ pd V
is the work done on the system per cycle, so the result follows.
A Carnot engine operates between two heat reservoirs and the cycle consists of two reversible
isothermal stages alternating with two reversible adiabatic stages. In the reversible isothermal
stages the temperature of the system remains constant while in the reversible adiabatic stages the
entropy remains constant. The representation of the Carnot cycle in the temperature-entropy plot
is, therefore, a rectangle, as shown in Fig. 6. 7, irrespective of the nature of the working substance.

r, _____________ a T - - - - - - - - ; b

Figure 6. 7 The
representation of the Carnot r. ------------(1,+-i- - - - - - - - - - l , c
cycle in the temperature-
entropy plot. ab and cd are
reversible isothermal
processes; da and be are 5
reversible adiabatic processes.

6.15 Show that, for a system consisting of one mole of molecules of an ideal gas

(a) Cv,m = dUmldT


(b) Cp,m- Cv,m = R

where Um is the molar internal energy of the gas, CP ,m and Cv,m are, respectively, the molar heat
capacity at constant pressure and at constant volume, and R is the molar gas constant.

Solution A fixed mass of ideal gas is an example of a closed hydrostatic system. For 1 mol of any closed
hydrostatic system the entropy form of the first law is

and the molar heat capacity at constant volume Cv,m is given by

Therefore, in general,

However, for an ideal gas, the internal energy is a function of temperature only (Joule's law).
Therefore, at constant temperature, Um does not depend on either pressure or volume so that the
molar heat capacity at constant volume may be written

The total derivative may be used instead of the partial derivative at constant volume. Note that this
result holds only for ideal gases (and any other systems for which Um is a function of temperature
only) and not for closed hydrostatic systems in general.
Substituting for dUm in the entropy form of the first law for 1 mol of ideal gas gives

Cv,m dT = TdSm - pdVm

or,

TdSm = Cv,m dT + pdVm

80
Differentiating with respect to T, keeping p constant gives

Now, T(aSm!aT)p is Cp,m• the molar heat capacity at constant pressure, and, for 1 mol of ideal
gas

so that

Therefore,

Cp,m = Cv,m +R
or

Cp,m - Cv.m = R

6.14 Exercises
6.1 A copper sphere of mass 0.50 kg is in an equilibrium state 6.4 A car of mass 1500 kg is travelling at 100 km hr- 1 when it
at a temperature of 363 K. It is then placed in a lake that is at a crashes into a wall and is rapidly brought to rest. If the tempera-
temperature of 283 K. Assuming that the specific heat capacity ture of the air is 293 K, calculate the change in the entropy of the
of copper is a constant over the range of temperatures involved, car. State clearly the assumptions that you make.
equal to 390 J kg- 1 K-\ calculate (1.97 X 103 J K- 1]

(a) the change in the entropy of the sphere; 6.5 Two moles of molecules of a certain ideal gas occupy a
(b) the change in the entropy of the lake; volume of 0.04 m3 at a temperature of 400 K. The gas is then
(c) the change in the entropy of the universe. allowed to undergo a free expansion in which its volume in-
creases to 0.08 m3 • Calculate

(a) the change in the entropy of the gas and


6.2 A Carnot engine operates between reservoirs at tempera- (b) the change in the entropy of the universe.
tures of 300 K and 550 K, respectively. In each cycle the engine
extracts 1000 J of heat from the hotter reservoir. Calculate

(a) the change in the entropy of the working substance in each 6.6 The temperature of a mass of water is reduced from 295 K
of the four stages of the cycle; to 280 K by means of a refrigerator operating between the water
(b) the change in the entropy of the working substance in a and the atmosphere which is at a temperature of 295 K. Calcu-
complete cycle of operation. late the minimum amount of work needed to bring about this
change if the mass of the water is 5.00 kg and the specific heat
[(a) 1.82 J K-\ 0, -1.82 J K-\ 0; (b) 0] capacity of water at constant pressure is taken as a constant
equal to 4180 J kg- 1 K- 1 • [8251 J]
6.3 A heat engine operates between reservoirs at temperatures
of 800 K and 300 K, respectively. In each cycle of operation the 6. 7 Two bodies A and B have heat capacities at constant pressure
working substance extracts 1200 J of heat from the high- S,A and CpB and initial temperatures of TA and T8 , respectively.
temperature reservoir and does 300 J of work on the surround- The bodies are then placed in thermal contact and exchange heat
ings. Calculate at constant pressure, reaching an equilibrium temperature Te.
Determine an expression forTe and also for the total change .1S
(a) the efficiency of the engine; in the entropy of the two bodies.
(b) the change in the entropy of the two reservoirs in each cycle; (Te = (CpATA + Cp 8 T8 )/(CpA + C£.8 );
(c) the change in the entropy of the universe in each cycle; .1S = CpA ln(Te/TA) + CpB ln(TellB)]
(d) the amount of work that would be done in each cycle on a
Carnot engine operating between the same reservoirs and
extracting the same quantity of heat from the hotter reser-
voir.

[(a) 0.25;(b) -1.5 J K-\hotter) and 3.0 J K- 1 (colder);


(c) 1.5 J K- 1 ; (d) -750 J]

81
Topic Guide

7 Thermodynamic Potential
7.1 The Helmholtz Function
(Helmholtz Free Energy)
7.2 Worked Examples on the
Helmholtz Function
7.3 The Gibbs Function (Gibbs
Free Energy)
Functions
7.4 Worked Examples on the
Gibbs Function
7.5 Useful Work and Availability
7.6 Worked Examples on
Availability
7.7 Exercises

7.1 The Helmholtz The law of increase of entropy is the most general statement of the second law of
Function (Helmholtz thermodynamics, but its very generality means that it is often not the most convenient
form of statement in a particular situation. Many real situations may be modelled as the
Free Energy) interaction of a system with a single reservoir and the law of increase of entropy may be
applied as follows.
Let any system with well-defined boundary and surroundings undergo any process that
takes it from an initial equilibrium state i to a final equilibrium state f. If the process is
infinitesimal the law of increase of entropy may be written

dS + dSo 2:: 0 (7.1)

where dS is the change in the entropy of the system and dS0 is the change in the entropy
of the surroundings. The equality sign holds when the process is reversible.
If, during the course of this infinitesimal process, the system absorbs an infinitesimal
quantity of heat q and has an infinitesimal amount of work w done on it, the change in
the internal energy U of the system is given by

dU = q +w (7.2)

Assume now that the surroundings consist of a reservoir at a uniform and constant
temperature T0 • Then, all changes taking place in the surroundings as a result of heat
transfer may be treated as reversible and, in particular, dS0 may be written

so that, substituting in Equation (7.1) gives

dS- q!T0 2:: 0 (7.3)

Combining Equations (7.2) and (7.3) then gives

dU - T0 dS ":5 w (7.4)
For a finite process Equation (7.4) must be integrated and, as w is path-dependent, the
conditions under which the process takes place must be known.
Make the additional assumption that the system has a diathermic boundary. Then, in
equilibrium, the temperature T must be uniform and equal to T0 , the temperature of the
surroundings. Equation (7.4) may then be written

d(U- TS) ":5 w (7.5)

or, putting U - TS equal to F,

82
dF~ w (7.6)

The quantity F is a function of the state of the system, known as the Helmholtz function.
For an infinitesimal reversible process the equality holds and

dF= w

and, in a finite reversible process

(7.7)

Here W is the total work of all types done on the system as it changes from state i to state
f under reversible isothermal conditions. Therefore, in a finite reversible isothermal
process the total amount of work done on the system is equal to the increase in the
Helmholtz function of the system. When a system does work on its surroundings in a
reversible isothermal process, the total amount of work done is equal to the decrease in
the Helmholtz function of the system.
Since F is a state function, the total amount of work done on the system in a reversible
isothermal process depends only on the initial and final equilibrium states of the system.
Therefore, when the process is reversible, it is only necessary that the initial and final
temperatures of the system have the same value for Equation (7. 7) to hold.
In a finite irreversible process

(T = T0 ) (7.8)

Since the only natural changes that occur in a system that is in thermal contact with a heat
reservoir do so in the direction of decreasing F, the Helmholtz function is known as a
thermodynamic potential.

7.2 Worked
Examples on the
Helmholtz Function
7.1 Show that, for an ideal gas, with equation of state pVm = RT, the molar Helmholtz function Fm is
given by

Fm = f Cv,m dT- T f (Cv,miT) dT- RTln Vm- (constant x T) +constant

where p is the pressure of the gas when it occupies a molar volume V m at a temperature T and Cv,m
is the molar heat capacity at constant volume.

Solution The molar Helmholtz function Fm of a closed hydrostatic system is defined by

where Urn is the molar internal energy of the system and Sm is its molar entropy. For an ideal gas
(see Example 6.15)

dUm= Cv,m dT

where Cv,m is the molar heat capacity at constant volume. Therefore, Urn is given by

Urn= JCv,m dT + constant


The molar entropy Sm of an ideal gas may be obtained from the expression

83
Forming the differential dSrn from this expression gives

Now,

and, using Maxwell relation (M3),

gives

The equation of state for 1 mol of an ideal gas is

from which

Therefore,

srn =I (Cv,rniT) dT + f (R/Vrn) dVrn +constant


=J(Cv,rniT) dT + R In Vrn +constant
Substituting the expressions for Urn and Srn in the expression for Frn gives the required result.
This result indicates that F (and, as will be seen later, the Gibbs function G) cannot be tabulated
in the way that U and S can since the values have a 'double arbitrariness'. The value ofF contains
arbitrary values arising from the values of both U and S. Therefore, tables of values of Urn and Srn
in terms of p, Vrn and T may be prepared for a chosen system, using an arbitrarily chosen reference
state to give the zero values of Urn and Srn, and the differences of Um and of Srn obtained from such
tables are meaningful.
These values could then be combined arithmetically to give formal values for Frn (and Grn)·
However,

which, using the entropy form of the first law, may be written

This last equation shows that the values of Fm would be meaningless, except for their differences at
identical temperatures, when

7.2 Show that, when a closed hydrostatic system in an equilibrium state characterised by pressure p,
volume V and temperature T undergoes an infinitesimal change of state, the change dF in the
Helmholtz function is given by

dF = - (p~V + S) dT + pVKr dp
where ~ is the cubic expansivity of the system, S is its entropy and Kr is the isothermal compress-
ibility.

Solution The Helmholtz function F of a closed hydrostatic system is defined by the equation

84
F = U- TS

where U is the internal energy of the system, S is its entropy and Tis its temperature. In an
infinitesimal process the change in F is dF, given by

dF = dU - TdS - SdT

Using the entropy form of the first law to substitute for dU, that is,

dU = TdS- pdV

gives

dF = -pdV- SdT

where p is the pressure of the system and V is its volume.


The cubic expansivity ~ of the system is defined by the equation

~ = (1/V) (iJV/iJT)p

and the isothermal compressibility Kr by the equation

Kr =- (1/V) (iJV/iJp)r

These equations suggest that it would be useful to consider treating the volume of the system as a
function of its temperature and pressure, to allow the expression for dF to be developed in terms of
dp rather than in terms of dV.
Therefore, let V be written in the form

V = V(p,T)
Then,

dV = (iJV/iJp)r dp + (iJV/iJT)p dT

and, substituting for dV in the expression for dF gives

dF= -p[(iJV/iJp)rdP + (iJV/iJT)pdT]- SdT

Then, putting (iJV!iJT)p = ~V and (iJV/iJp)r = - VKr gives

dF = - p(- VKrdP + ~VdT) - SdT

and, rearranging,

dF = - (p v~ + s) dT + p VKr dp

7.3 The molar Helmholtz function Fm for a certain closed hydrostatic system is given by

Fm = -(a/Vm) - RTln(Vm - b) + f(T)

where Vm is the molar volume when the pressure is p and the temperature of the system is T, a and
bare constants, R is the molar gas constant and f(T) is a function of temperature only. Derive an
expression for the pressure exerted by this system in terms of the primitive coordinates.

Solution The molar Helmholtz function Fm for a closed hydrostatic system at a temperature Tis given by

where Um is the molar internal energy and Sm is the molar entropy. In an infinitesimal process from
an equilibrium state the change in Fm is dFm, given by

85
The entropy form of the first law of thermodynamics states that

where p is the pressure exerted by the system and V m is the molar volume. Substituting for dUm
gives

dFm = TdSm- pdVm- TdSm- SmdT


= -pdVm- SmdT

Therefore,

For a closed hydrostatic system with Fm given by

Fm = -(a/Vm)- RT ln(Vm- b)+ f(T)

Therefore,

which is the equation of state. In more familiar form it is

[p + (a/V~)](Vm- b)= RT

that is, the van der Waals equation (see p. 140).

7.3 The Gibbs When a closed hydrostatic system has a boundary that is both diathermic and free to
Function (Gibbs move without friction, both thermal and work interactions may occur with the surround-
ings. Assume that the surroundings are not only represented by a heat reservoir at a
Free Energy) uniform and constant temperature T0 , but also exert a uniform and constant pressure p 0 •
Thermodynamic equilibrium is then achieved when the temperature T of the system is
uniform and equal to T0 and, also, when the pressure p in the system is uniform and
equal to p 0 •
If it is assumed that the only work interaction between the system and its surroundings
is pressure-volume (displacement) work, and that this work is carried out reversibly,
then, in an infinitesimal process from an equilibrium state, the work w done on the
system is given by

where dV is the increase in volume of the system. Substituting for win Equation (7.4)
gives

dU- T0 dS ::S -pdV

or,

(7.9)

Since p =p 0 and T = T0 this expression may be written

d( U + p V - TS) ::S 0 (7 .10)

86
The function U + p V - TS has the dimensions of energy and is a function of the state of
the system when p = p 0 and T = T0 • It is known as the Gibbs function G of the system, so
that Equation (7 .10) may be written

dG :S 0 (7.11)

Two simple deductions follow from Equation (7 .11). When a natural (irreversible)
process occurs under the conditions T = T0 and p = p 0 it does so in the direction of
decreasing G, so that the Gibbs function of a system is a thermodynamic potential. When
a closed hydrostatic system undergoes a reversible process at constant temperature and
pressure, its Gibbs function remains unchanged.

7.4 Worked
Examples on the
Gibbs Function 1.4 The molar Gibbs function Gm for a certain closed hydrostatic system that exerts a pressure p at a
temperature T is given by

Gm = RT lnp +A + Bp + (Cp 2/2) + (Dp 3/3)


where R is the molar gas constant and A, B, C and D are functions of temperature only. Derive an
expression for the equation of state of this system.

Solution The molar Gibbs function Gm of a closed hydrostatic system is given by

where Um is the molar internal energy, Vm is the molar volume and Sm is the molar entropy when
the system is at a temperature Tunder a pressure p. In an infinitesimal process from an equilibrium
state the change in Gm is given by

The entropy form of the first law for a closed hydrostatic system consisting of one mole is

so that substituting for dUm gives

Therefore,

Applying this result to the equation for Gm gives

Vm = (RT!p) + B + Cp + Dp2
or,

7.5 Derive an expression for the molar Gibbs function Gm of an ideal gas.

Solution The system is one mole of molecules of an ideal gas, which is a closed hydrostatic system.The
molar Gibbs function Gm for a closed hydrostatic system is given by

87
where Um is the molar internal energy, V m is the molar volume, Sm is the molar entropy and p is
the pressure of the system when the temperature is T.
For an ideal gas

where Cv,m is the molar heat capacity at constant volume (see worked example 6.15). Further, for
an ideal gas

Cp,m - Cv,m = R

where Cp,m is the molar heat capacity at constant pressure and R is the molar gas constant (see
worked example 6.15). Therefore,

dUm= Cv,m dT
= (Cp,m- R) dT

so that

Um = JCp,m dT - JRdT + constant


or,

Um = J Cp,m dT - RT + constant
The form of the Gibbs function suggests that, to find an expression for Sm, the entropy of the
system should be treated as a function of temperature and pressure; that is, Sm should be written

Then,

Now,

the molar heat capacity at constant pressure. The second partial derivative may be transformed
using Maxwell relation (M1):

Therefore,

The equation of state of an ideal gas is

so that

Then,

dSm = (Cp,m!T) dT- (Rip) dp

which integrates to give

Sm = J(Cp,m/T) dT - R lnp + constant


88
or

TSm = T f (Cp,m/T) dT- RT lnp + (T x constant)


Combining the expressions for Urn and TSm and writing RT for p V m gives

Gm = JCp,m dT- RT + constant+ RT- T J(Cp,m!T) dT + RT lnp - (T X constant)

or,

Gm = RT lnp + f Cp,m dT- T f (Cp,m!T) dT- (T x constant) +constant


This result confirms the previous statement about the tabulation of values of Gm for a given
system.
When the reference state and temperature range are chosen so that Cp,m may be treated as a
constant, the equation for Gm becomes

Gm = RT lnp + Cp,m T - TCp,m lnT + (T x constant) + constant

7.6 A fuel cell is a device with two electrodes in an electrolyte and which produces electrical energy by
the chemical reaction of two reagents, fed to the two electrodes, respectively.
In a particular fuel cell hydrogen is fed to one electrode at a temperature of 298 K and at a
pressure slightly greater than atmospheric. Oxygen at the same temperature and pressure is fed to
the other electrode. Within the cell these gases react isobarically and isothermally to form liquid
water, If the reaction occurs reversibly, and the cell has negligible internal resistance, calculate the
e.m.f. of the cell using the data below.

Molar enthalpy Hm/J mol- 1 17 200 8100 -269 300


Molar entropy Sm/J K- 1 mol- 1 201 128 66.7
(assuming sm. 0 = 0)

Solution The basic reactions taking place in the cell are

2H2 (g)- 4H+ + 4e-

and

where e- is an electron. The net reaction is, therefore,

In this reaction, four electrons pass through the external circuit from anode to cathode (and work is
done on the surroundings) for each molecule of 0 2 reacting. If 1 mol of liquid H 2 0 is taken as the
final state of the system, the initial state consists of 1 mol of H 2 molecules and ! mol of 0 2
molecules. The useful work done by the system results from 2 N A electrons passing through the
external circuit, where NA is Avogadro's constant.
This reaction takes place at constant temperature and pressure so that the Gibbs function can be
used to discuss the behaviour of the system.
Take the total quantity of the reactants as the system and the reaction of 1 mol of H 2 molecules
and ! mol of 0 2 molecules to give 1 mol of H 2 0 molecules as the process. Then, if the process is
considered to be reversible, the change in the Gibbs function of the system in an infinitesimal part
of the process is dG, given by

dG = dU + pdV + Vdp - TdS - SdT

Here U is the internal energy of the system, Sis the entropy and Vis the volume occupied by the
system at a temperature T and under a pressure p.

89
The system does both pressure-volume (displacement) work and electrical work, since the
energy of the reaction appears partly as electrical energy and partly as a heat traflsfer to the
surroundings. Therefore, the change in the internal energy dU must be written

dU = TdS- pdV +we

where we is the electrical work done on the system in the infinitesimal process. Therefore,
substituting for dU, the expression for dG becomes

dG = We + Vdp - SdT
so that, at constant temperature and pressure,

In a finite process under these conditions, when G changes from Gi to Gf,

(7.12)

where We is the total amount of electrical work done on the cell.


When a charge Q flows under an e.m.f. E from the anode to the cathode in the external circuit,
the work we done on the system is given by

7.13

Combining Equations (7.12) and (7.13) gives

(7.14)

Now

G = U + p V - TS = H - TS

where His the enthalpy of the system. Therefore, Equation (7.14) may be written

(7.15)

For the system and process under consideration, Equation (7.15) becomes

where e is the elementary charge, w denotes water, H 2 denotes hydrogen and 0 2 denotes oxygen.
Substitution of the given values gives

[- 269 300 (J mol- 1 ) - 298 (K) X 66.7 (K- 1 mol- 1] - [8100 (J mol- 1 ) - 298 (K)
x 128 (J K- 1 mol- 1) +! 17 200 (J mor 1) -~ 298 (K) x 201 (J K- 1 mol- 1 )]
= 2 X 6.022 X 1023 (mor 1 ) X 1.602 X 10- 19 (C) X E

which gives a value for E of

E = 1.23 V

Note that this problem could be tackled more generally using the availability function (see Section
7.5), but, where the temperature and pressure of the system are constant and equal to the
respective values for the surroundings, the availability and the Gibbs function are identical.

7.5 Useful Work Frequently, when dealing with the behaviour of closed hydrostatic systems, only
and Availability pressure-volume work is considered. Since this form of work involves only pushing back
the surrounding atmosphere it is often called 'useless' work. However, in addition to
interacting with the hydrostatic surroundings, a system may be coupled to a second

90
system which will be called the body, and which will be treated as having an adiabatic
boundary, so that the only interaction between this body and the system is a work
interaction. Then, when the system undergoes an infinitesimal process from an initial
equilibrium state, the work interaction w may be written

w= W 1 + w"
where w 1 is the non-pressure-volume work interaction with the body (called useful work
when it is done on the surroundings) and w" is the useless work interaction with the
hydrostatic surroundings.
Consider a closed hydrostatic system in surroundings that are at a uniform and
constant temperature T 0 and exert a uniform and constant pressure Po on the system. The
heat absorbed by the system in any infinitesimal process must satisfy relation (7.3), that
is,

where Sis the entropy of the system. Then, the change in the internal energy U of the
system is given by

dU = q + w ::; T0 dS +w
If the pressure-volume work is carried out reversibly

w" = -p 0 dV

so that

W 1 2: dU + p 0 dV- T0 dS

Since p 0 and T0 are both constant, this relation may be written

W 1 2: d(U + p 0 V- ToS)

or

W 1
2: dB (7.16)

where B = U +Po V- T 0 S and is known as the availability of the system. B is, of course,
a property of the system and the surroundings, but it is a useful function only because p 0
and T 0 are constant.
w 1 is the non-pressure-volume work done in a given change in given surroundings.
When this work is done by the system on the body the work is useful work. dB is then
negative: the useful work done on the body in a reversible process is equal to the
decrease in the availability of the system and is the maximum amount of useful work that
can be obtained in the given situation. Note that B is not identical to the Gibbs function
of the system. That condition is fulfilled only when the temperature of the system equals
that of the surroundings and also the pressure exerted by the system equals that of the
surroundings.

7.6 Worked
Examples on
Availability 7. 7 Two moles of an ideal gas expand reversibly at a constant temperature of 300 K from a pressure of
3.0 X 105 Pa to a pressure of 1.5 X 105 Pa in surroundings that exert a uniform and constant
pressure of 1.0 x 105 Pa and are at a uniform and constant temperature of 300 K. Calculate the
amount of useful work that is done during the expansion.

91
Solution The system consists of 2 mol of ideal gas. For the gas to remain in equilibrium when its pressure
differs from that of its surroundings and no frictional forces are present, the system must be
coupled to a body. It is this coupling that allows useful work to be done. For the sake of
definiteness, the gas may be considered to be contained in a cylinder by means of a well-fitting
frictionless piston, the piston being connected mechanically to a second system, or body, by means
of a connecting rod, as in Fig. 7.1
When the surroundings are at a uniform and constant temperature T0 and exert a uniform and
constant pressure p 0 , the maximum amount of useful work that the system can perform is given by
Figure 7.1 The system the decrease in the availability B of the system. In an infinitesimal part of any reversible process
consisting of a fixed mass of the change in the availability is given by
gas interacting with both the
atmosphere and a body.
dB = dU + p 0 dV - T0 dS

where U is the internal energy of the system, V its volume and Sits entropy.
As the process is reversible, the work w' done by the body on the system is given by

w' =dB= dU + p 0 dV- T0dS

For an ideal gas the internal energy is a function of temperature only (Joule's law) so that, when
the temperature of the system is constant, dU is zero and

w' = p 0 dV- T0dS (T = T0 )

The entropy of the system may be treated as a function of the temperature and pressure of the
system. Then,

S = S(T, p)

and

dS = (iJS/iJT)P dT + (iJS/iJp)r dp
In an isothermal process dT is zero and, using Maxwell relation (Ml), the above equation
becomes, for the process considered,

dS = (iJS/iJp)r dp = -(iJV!iJT)p dp (T = T0)

The equation of state for a system consisting of n moles of ideal gas is

pV = nRT

where R is the molar gas constant. Therefore,

(iJV!iJT)P = nR/p

so that

w' = (nToRip) dp + p 0dV

In a finite process, when the volume of the system changes from V; to Vt, the non-pressure-volume
work done W' is given by

which, using the equation of state, may be written

Now, Po = 1.0 X lOS Pa; T0 = 300 K; P; = 3.0 X 105 Pa; Pt = 1.5 X 105 Pa; and n = 2 mol. Taking
R = 8.315 J K- 1 mol- 1 gives

92
W' = {2 (mol) X 8.315 (J K- 1 mol- 1 ) X 300 (K) X ln[l.5 X 105 (Pa)/3.0 X 105 (Pa)]}
+ {2 (mol) X 1.0 X 105 (Pa) X 8.315 (J K- 1 mol- 1 ) X 300 (K) X [1/{1.5 X 105 (Pa)}
- 1/{3.0 X 105 (Pa)})}
= -1800 J

As expected, W' is negative, indicating that the system does useful work on the body.
This problem may also be approached directly by considering the total amount of work done on
the system and subtracting the amount of pressure-volume work. In an infinitesimal reversible
process at the constant temperature T0 , the total amount of work w done on the system is given by
(see Section 7.1)

w = dF = d(U- T0 S) = dU- T0 dS

The entropy form of the first law is, for this situation,

dU = T0 dS - pdV
so that

w = -p dV

This work is made up of the pressure-volume work w" done by the hydrostatic surroundings and
the work w' done by the body. The work done by the hydrostatic surroundings in an infinitesimal
reversible process is given by

w" = -p 0 dV

Therefore, w' is given by

w' w- w" = -pdV + p 0 dV


From the equation of state

p nRTIV

giving

w' - (nRT0 /V) dV + p 0 dV

which on integrating gives

W' - nRT0 ln(Vt!V;) + Po(Vt - V;)


nRT0 ln(ptiP;) + Po(Vt - V;)
as before.

7.8 Calculate the maximum amount of useful work that may be obtained from 106 kg of water at a
temperature of 373 K, when a reservoir at a temperature of 298 K is available. The specific heat
capacity of water may be taken as a constant equal to 4200 J kg-1 K- 1 •

Solution The system that is supplying the energy for the performance of useful work is the mass m of water,
having a specific heat capacity at constant volume Cv. The amount of useful work obtained will be a
maximum when all the processes involved are reversible, and will be given by the decrease in the
availability of the system. In any infinitesimal process from an equilibrium state the change in the
availability B of the system is dB, given by

dB = dU + p 0dV - T0 dS

where U is the internal energy of the system, S is its entropy and Vis the volume it occupies when
its temperature is T. The surroundings are at a uniform and constant temperature T0 and exert a
uniform and constant pressure p 0 • When the infinitesimal process is reversible the non-pressure-
·volume work w' done on the system is given by

93
w' =dB

From the entropy form of the first law dU is given by

dU = TdS- pdV

where p is the pressure of the system. Then

w' = TdS - pdV + p 0 dV - T0 dS


If it is assumed that the system remains at constant volume during the process (this cannot be
exactly true, but is likely to be a good approximation), dV is zero and dS is given by

dS = (mcv!T) dT

Then

so that, for the finite process in which the system starts at a temperature Ti and ends at a
temperature Tr = T0 , the non-pressure-volume work W' is given by

W' = JTo mcv dT- fro (T0mcviT)dT


Tj Tj

= mcv(T0 - Ti) - T0 mcvln(TJTi)

Ti = 373 K; Tr = T0 = 298 K; m = 106 kg; and Cv = 4200 J kg- 1 K- 1 • Therefore,

W' = 106 (kg) x 4200 (J kg-1 K-1 )[298 (K) - 373 (K)]
- [298 (K) X 106 (kg) X 4200 (J kg- 1 K- 1 ) X ln{298 (K)/373 (K)}]
=-34x109 J

The minus sign indicates that the work is done by the system on a coupled body and is, therefore,
useful work.
This problem could be approached more directly by considering the processes taking place in
more detail. This has already been demonstrated in Example 6.6. The advantage of using the
availability is that there is no need to describe the processes by which the work is realised.

7.9 A solid body, having a heat capacity at constant volume of 4000 J K- 1 , is at a temperature of
300 K. An identical body is at a temperature of 500 K. Calculate the maximum amount of useful
work that can be obtained from the two bodies in the absence of any other bodies or reservoirs.

Solution As in the previous example, the temperature difference between the two bodies may be used to
produce useful work by operating a heat engine between them. To obtain the maximum amount of
such work the heat engine should be a reversible engine (Carnot engine), which means that it must
operate in infinitesimal cycles if the isothermal changes in the bodies of finite heat capacity are to
be reversible. The problem may also be tackled by using the availability function and by analysing
the performance of the Carnot engine in detail after the manner of Problem 6.9.
Consider the approach using the availability of an appropriate system. Both bodies contribute to
the work interaction and the system consists of both bodies. The maximum amount of useful work
obtained will be equal to the decrease in the availability of the system. The surroundings here only
interact mechanically with the system (by means of a Carnot engine) so that there is no specified
temperature or pressure for the surroundings. However, in the operation of the Carnot engine,
energy is extracted from the body at the higher temperature and some is rejected to the body at the
lower temperature. Eventually the two bodies reach a common temperature T0 , say, when the
Carnot engine ceases to operate. This final equilibrium temperature may be taken as the (effective)
temperature in the expression for the availability of the system.
In an infinitesimal process from an equilibrium state for either body the change in the
availability B of the body is given by

dB = dU + p 0dV - T0 dS

94
From the entropy form of the first law

dU = TdS- pdV
where U is the internal energy of either body, V the volume and S the entropy, so that

dB = - (p - p 0 ) dV + (T - T0) dS
If the processes are assumed to be reversible and to take place at constant volume

dV= 0 and dS = ( Cv!T) dT

The non-pressure-volume work w' done on one of the bodies in the infinitesimal process is then
given by

w' = dB= [Cv(T- T0 )/T] dT

If the initial temperature of body 1 is T1 , the non-pressure-volume work done on this body in the
complete process is w~, where

W'1 ro (
Tl
TCviT)dT - ro (
Tl
T0 Cv!T) dT

Cv ( T0 - T1 ) - CvTo In ( T0 /T1 )

Similarly, for body 2, the non-pressure-volume work done on the body during the complete
process is W~, given by

where T2 is the initial temperature of body 2. The total non-pressure-volume work W' is,
therefore, given by

The next problem is to determine the value of T0 • Since all the processes taking place are
reversible, the total entropy change of all the interacting bodies and devices is zero. The auxiliary
Carnot engine suffers no entropy change in a complete number of cycles so that, if AS 1 is the
entropy change for body 1 and AS 2 that for body 2

In an infinitesimal reversible process taking place at constant volume at a temperature T, the


change in the entropy of a body is given by

dS = (Cv!T) dT

Therefore, for bodies 1 and 2, treated as an isolated system,

0
AS= JT (Cv!T) dT + JTo (Cv!T) dT = 0
Tl T2

which integrates to give

or

giving

95
Therefore,

Now

so that

and

For the bodies concerned

T0 = (500 (K) x 300 (K)) 112 = 387.3 K


Therefore,

W' = 4000 (J K- 1) x (387.3 (K) - 500 (K)) + 4000 (J K- 1) x (387.3 (K) - 300 (K))
= 4000 X ( -112.7) + 4000 X 87.3

- 450 800 + 349 200


-102 X 103 J

The sign of W' is negative, indicating that the non-pressure-volume work is done on the surround-
ings and is, therefore, useful work.
This problem can also be examined by treating the working substance of the Carnot engine as
the system and considering its behaviour as it performs the sequence of infinitesimal cycles, as in
Example 6.9. A consideration of that approach shows that the use of the availability in the present
example is not as helpful as it was in Example 7.8.

7.10 Calculate the maximum amount of useful work that may be obtained from 10.0 m3 of ideal gas at a
temperature of 283 K and at a pressure of 1.1 x 106 Pa, when a heat reservoir at a temperature of
283 K and exerting a pressure of 1.1 X 105 Pais available.

Solution The maximum amount of work that can be performed by a system is equal to the decrease in the
availability of the system. In this example the system is a mass of ideal gas interacting with
surroundings at the same temperature and coupled mechanically to a body so that non-pressure-
volume work may be performed.
The availability B of the system is given by

B=U+p0 V-T0 S

where U is the internal energy of the system, V is its volume and S its entropy, while p 0 is the
pressure of the surroundings and T0 their temperature. In an infinitesimal change of the system
from an equilibrium state the change in B is dB, given by

dB = dU + p 0dV - T0dS

and, from the entropy form of the first law,

dU = TdS- pdV
where p is the pressure exerted by the system when it occupies a volume V at a temperature T.
Substituting for dU gives

96
dB TdS - T0 dS+ p 0 dV - pdV
(T- T0 )dS + (p 0 - p)dV

To obtain the maximum amount of useful work the process must take place reversibly so that, for
the given system and surroundings, T must equal T0 • Then

dB = (p0 - p)dV

Let there ben moles of molecules in the system. Then, since the system is an ideal gas, its equation
of state is

pV = nRT

so that

p = nRTIV = nRT0 /V

since T = T0 • The expression for dB then becomes

dB = [p0 - (nRT0 1V)] dV

Therefore, in a finite process, in which the volume of the system changes from V; to Vt. the change
in B is AB, given by

AB = p 0 (Vc- V;) - nRT0 ln(Vr/V;)

Since the temperature of the system remains constant during the process

(Boyle's law)

Substituting the given values to obtain Vc gives

1.1 X 105 (Pa) X Vc

that is,

From the equation of state

n = p;V;IRT = p;V;IRT0

Then,

AB = p 0 (Vc- V;) - (p;V; IRT0 )RT0 ln(Vr!V;)


= Po(Vc - V;) - P; V; ln(Vr!V;)
1.1 x 105 (Pa) x (100 (m3 ) - 10.0 (m3 ))
- 1.1 X 106 (Pa) X 10.0 (m3 ) X ln(100 (m3 )/10.0 (m 3 ))
-15.4 X 106 J

AB is negative so that the non-pressure-volume work is useful work and the value is the maximum
- Po
amount of useful work that can be performed by the system in the given surroundings.
PI-Pt
l====it.o-..:;.:bodv This problem can also be tackled directly by considering the work done by the gas against the
v.-v,
To - To surroundings and on a body coupled by a mechanical link only.
The situation is represented in Fig. 7 .2. The total amount of work W done on the gas is given by
Figure 7.2 Gaseous system,
in surroundings at a
W =- f Vf

Vj
pdV
temperature T0 and pressure
Po, connected to a body. and, substituting for p from the equation of state,

97
= - J (nRT0 /V) dV
Vf
W
Vi

Since Pi Vi is a constant

Of this total work, the work W" done by the atmosphere on the system is given by

Therefore, the work W' done by the body on the system is given by

W' W-W"

as before.

7.11 Calculate the maximum amount of useful work that may be obtained from 10.0 m 3 of an ideal
monatomic gas at a temperature of 373 Kanda pressure of 1.1 x 106 Pa, when a heat reservoir at a
temperature of 283 K and exerting a pressure of 1.1 x 105 Pa is available. The molar heat capacity
at constant volume is 3R/2, where R is the molar gas constant, and may be taken as a constant.

Solution The system is the mass of ideal monatomic gas, occupying a volume V at a pressure p and a
temperature T. The maximum amount of useful work that can be performed by the system is equal
to the decrease in the availability of the system. The availability B of the system is given by

B = U + Po V - T0 S
where U is the internal energy of the system, S is its entropy and p 0 and T0 are, respectively, the
pressure and temperature of the reservoir. In any infinitesimal change of the system from an
equilibrium state the change in B is given by

dB = dU + Po dV - To dS

The entropy form of the first law is

dU = TdS- pdV

so that the expression for dB may be written

dB = (T- T0 ) dS + (p 0 - p) dV

To proceed further an expression is needed fordS, and also the relationship between p and V.
The latter is, of course, the equation of state:

pV= RT

Let S be treated as a function of T and V, that is, write

S = S(T,V)

Then,

Now T(a S/a T)v is Cv, the heat capacity of the system at constant volume, and, from Maxwell
relation (M3),

98
(as!aV)r = (ap!aT)v

Therefore,

dS = (CviT)dT + (ap!aT)vdV
From the equation of state

(a pia T)v = nRIV

where n is the number of moles of molecules of gas. Therefore,

dS = (Cv!T)dT + (nR!V)dV
and the expression for dB may be written

dB = (T- T0 )[(CviT)dT + (nR!V) dV] + (p 0 - p)dV

or, since p = nRTIV,

dB = (T- T0 )[(CviT)dT + (nRIV)dV] + [p0 - (nRTIV)]dV

giving

dB = (T- T0 )(Cv!T)dT + [Po - (nRT0 1V)]dV


In a finite process, in which V changes from Vi to Vt and Tchanges from Ti to Tt ( = T0 ), the change
in B is !1B, given by

11B = Bt- Bi = ro
~
CvdT- To ro~
(CviT)dT f
+ Vt PodV- nRTo Vt (l!V)dV
~
f
~

= Cv(T0 - Ti) - T~ Cv ln(T0 1Ti) + PoCVt - Vi) - nRTo ln(Vt!VD

In this problem, Ti = 373 K; Tt = T0 = 283 K; Pi = 1.1 X 106 Pa; Pt = 1.1 X 105 Pa; and
vi= 10.0 m 3 •
It is also necessary to determine Cv and Vt· Now,

Cv =n Cv,m

where Cv,m is the molar heat capacity at constant volume. Since

pV = nRT
the initial conditions give

1.1 106 (Pa) X 10.0 (m 3 )


X
8.315 ( J K- 1 mol- 1 ) X 373 (K)
= 3546.7 mol

Therefore,

Cv = 3546.7 (mol) X ~ X 8.315 (J K- 1 mol- 1)


= 44 236.2 J K- 1
Using the equation of state in the form

99
gives

Vt = (pYJt)!(ptTi)
1.1 X 106 (Pa) X 10.0 (m3 ) X 283 (K)
1.1 X lOS X 373 (K)

Therefore, AB equals

AB = 44 236.2 (J K- 1) X (283 (K) - 373 (K)) - 283 (K) x 44 236.2 (J K- 1)


X ln(283 (K)/373 (K)) + 1.1 X 105 (Pa) X (83.46 (m3 ) - 10.0 (m3 ))
- 1.1 x 105 (Pa) x 83.46 (m3 ) x ln(83.46 (m 3)/10.0 (m3 ))
-1.19 X 107 J

As AB is negative, this indicates that the non-pressure-volume work is negative, that is, it is useful
work.

7. 7 Exercises
7.1 For a closed hydrostatic system, the equilibrium states of 7.4 A certain polymer sample may be treated as a perfectly
which are specified by the values of the pressure p, volume V and elastic solid with equation of state
temperature T, derive the equations

U= F- T(iJF/iJT)v= -T2 (iJ(FIT)/iJT)v


where L 0 is the unstretched length of the sample at a tempera-
and ture T, Lis the length under a load J' and a and b are constants.
Derive expressions for the change AU in the internal energy
and the change AG in the Gibbs function of the sample when its
length is changed reversibly from L 0 to 2L0 at a constant tem-
perature T.
7.2 A closed rigid cylinder contains an internal piston that Hint: use the appropriate analogues of the Gibbs-Helmholtz
moves without friction. On either side of the piston the cylinder equations, together with the result that, in a reversible iso-
contains 1 mol of an ideal gas, the volume of gas on one side of thermal process, the work done on a system is equal to the
the piston initially being 1.0 x 10- 2 m3 and on the other side 1.0 increase in the Helmholtz function.
x 10- 3 m3 • The walls of the cylinder are diathermic and the [AU= (L~)(2aT- 3bT2 ); AG = (SL~)(aT- bT 2 )]
cylinder is immersed in a large bath of liquid at a temperature of
273 K. The piston is connected purely mechanically to an outside 7.5 Two moles of an ideal gas are contained in a cylinder and
body. Calculate the work done on this body when the piston occupy a volume of 1.0 x 10-4 m3 at a temperature of 373 K.
moves reversibly so that the final volumes of the gas on the two The temperature of the gas then falls to 293 K, which is the
sides of the piston are 6.0 x 10- 3 m3 and 5.0 X 10- 3 m3 , respec- temperature of the surroundings. During this process the volume
tively. [ -2490 J] of the gas varies, but the final volume is again 1.0 x 10-4 m3 •
Calculate the maximum amount of non-pressure-volume work
(Try this problem both by using the Helmholtz function and by that can be done on the gas if the molar heat capacity at constant
direct integration of - pdV.) volume of the gas is a constant and equal to 12.47 J K- 1 mol- 1 •
[-231 J]
7.3 Show that, for a closed hydrostatic system, the following
relations hold. 7.6 Calculate the maximum amount of non-pressure-volume
work that can be done on a system consisting of two identical
H = G - T (a G/iJ T)p =- T 2 [iJ( GIT)/iJ T]p bodies, one of which is at a temperature of 373 K and the other is
at a temperature of 283 K. Each body has a mass of 1.0 X 106 kg
CP =- T (iJ 2 GiiJ T 2 )p and a specific heat capacity of 4.2 x 103 J kg- 1 K- 1 •
[ - 2.6 X 1010 J]
where G is the Gibbs function of the system, CP is the heat
capacity at constant pressure, H is the enthalpy and T is the
temperature.

100
Topic Guide
8.1 Closed Hydrostatic Systems
8.2 Worked Examples on
Closed Hydrostatic Systems
8 Some Simple
8.3 Ideal Gases
8.4 Worked Examples on Ideal
Gases
Thermodynamic Systems
8.5 Perfectly Elastic Sol ids
8.6 Worked Examples on
Perfectly Elastic Solids
8.7 Voltaic Cells
8.8 Worked Example on Voltaic
Cells
8.9 Exercises

In this chapter the behaviour of some simple thermodynamic systems will be considered.
For each of the systems the coordinates descriptive of the equilibrium states will be
given, together with the form of the first law for reversible processes, the equations
appropriate to an absorbtion of heat in a reversible process and the effect of a reversible
adiabatic process. This may involve the definition of appropriate heat capacities.

8.1 Closed A closed hydrostatic system is defined as a system of constant mass that exerts a uniform
Hydrostatic hydrostatic pressure on its surroundings. Included in this class of system are pure
substances in the gas, liquid and solid phases (including states in which different phases
Systems coexist in equilibrium), homogeneous mixtures made up of different substances in the
same phase and heterogeneous mixtures of different substances in different phases.
Each equilibrium state of a closed hydrostatic system is represented by a point in
pressure p, volume V and temperature Tspace. The totality of such equilibrium states is
a continuous surface in p-V-T space and the equation of this surface is the equation of
state of the system. Quite generally, this equation may be written

F(p, V, T) = 0 (8.1)

where F is a function that must be determined by experiment.


When the volume of the system increases by the infinitesimal amount dV in a revers-
ible process, with the pressure p remaining sensibly constant, the work done on the
system by the surroundings is well defined and may be written dW, given by

dW = -pdV (8.2)
In a finite process, in which the volume of the system changes from Vi to Vt. the work W
done on the system is given by

W=- fVj
Vf
pdV

To evaluate this integral the dependence of p on V must be known.


In an infinitesimal reversible process the heat dQ absorbed by the system at a
temperature T is given by

dQ = TdS

where S is the entropy of the system. Therefore, the first law for an infinitesimal
reversible process may be written

dU = TdS- pdV (8.3)

101
where U is the internal energy of the system.
Treating the entropy of the system as a function of temperature and volume, it may be
shown that, in an infinitesimal reversible process,

TdS= CvdT+ T(iJp!iJT)vdV (8.4)

a result known as the first TdS equation. Starting with the entropy as a function of
temperature and pressure gives

TdS = CP dT- T(iJV/iJT)p dp (8.5)

a result known as the second TdS equation.


When a closed hydrostatic system undergoes a reversible process in which there is no
thermal interaction, the process takes place at constant entropy and is described by the
so-called adiabatic equations:

(iJT/iJp) 5 = (TICP) (iJV!iJT)p (8.6)

(iJT!iJV)s = -(T!Cv) (iJp!iJT)v (8.7)

y (iJV/iJp)s = (iJV/iJp)r (8.8)

where y = CP!Cv, CP is the heat capacity at constant pressure and Cv is that at constant
volume.

8.2 Worked
Examples on Closed
Hydrostatic Systems
8.1 A block of copper is under a pressure of 1.01 x 105 Pa (1 atmosphere) at a temperature of 293 K.
The temperature of the block is then raised to 305 K. If the volume of the block is maintained
constant during this process, calculate the final pressure acting on the block.
For copper, cubic expansivity = 50.4 x 10- 6 K- 1 and isothermal compressibility = 7.20 x
10-lz Pa-l

Solution The system here is the block of copper, and it is able to undergo both thermal and work
interactions with the surroundings.
Let the volume occupied by the block be V when its temperature is T and the pressure exerted
by the surroundings is p. The block is a closed hydrostatic system and, therefore, for all equilib-
rium states, the volume may be treated as a function of temperature and pressure, that is, V may
be written

V = V(p, T)

From this it follows that, in any infinitesimal part of a reversible process, the change in volume dV
is given by

dV = (av;ap)r dp + (a v;ar)p dT
Now the cubic expansivity ~ of a closed hydrostatic system is given by

~ = (1/V) (av;ar)p

and the isothermal compressibility Kr is given by

Kr = -(1/V) (av;ap)r

Therefore, dV may be written

102
dV = ~VdT- KrVdp

In a process in which the volume remains constant, dV is zero and

~ VdT = KrVdp

For a finite process at constant volume, in which the pressure changes from Pi to Pt and the
temperature from Ti to Tt. if ~ and Kr are treated as constants,

Substituting the given values

50.4 X 10-6(K- 1 ) X (305 (K) - 293 (K)) = 7.20 X 1o-12 (Pa- 1) X (Pt- 1.01 X 105 (Pa))

giving

Pt = 605 X 105 Pa (about 600 atmospheres)

8.2 A certain block of copper has a volume of 1.0 x 10-6 m3 at a temperature of 100 K and under a
pressure of 1.1 x 105 Pa. The block of copper is then compressed reversibly and isothermally until
the pressure reaches a value of 1.3 x 108 Pa. Calculate the change in the internal energy of the
block of copper resulting from this process.
For copper, isothermal compressibility = 0.721 x 10-11 Pa- 1 and cubic expansivity = 50.4 x
1o-6 K-1.

Solution The system is the block of copper which is able to undergo both work and thermal interactions with
the surroundings.
To determine the change in the internal energy of the system, the first law indicates that both the
heat absorbed by the system and the work done on it must be determined. Consider first the work

rt
W done on the system during the reversible process.

W=- pdV
vi
where Vi is the initial volume of the system and Vf the final volume. Since, in an equilibrium state,
the volume V of the system may be treated as a function of its temperature T and its pressure p, the
change in V in an infinitesimal process is given by

dV= (av!aT)pdT+ (av;ap)rdP

which may be written

dV = ~V dT -KrVdp

where ~ is the cubic expansivity of the system and Kr its isothermal compressibility.
In a process taking place at constant temperature, dT is zero and, therefore,

dV(isothermal) =- Kr V dp

so that

W = - J
Pf
(- Kr V) p dp
Pi
where Pi is the initial pressure and Pt the final value. If Kr is insensitive to pressure and the change
in volume of the system during the process can be ignored (solids are almost incompressible), the
expression for W may be written

W +Kr V rfPi
pdp

(Kr V/2) (pi - pf)

103
Substituting the given values

= 0.122 J

The heat Q absorbed by the system during the process may be determined from the change in
the entropy S of the system. In a reversible process

Q = JTdS
and, if the process is also isothermal, the expression for Q becomes

Since S may be treated as a function of Tand p, in an infinitesimal reversible process the change in
the entropy of the system is dS, given by

dS = (astaT)p dT + (astaph dp
Now,

From Maxwell relation (M1)

and

Therefore,

dS = (CJT) dT- ~ V dp

In an isothermal process dT is zero so that

dS(isothermal) = - ~Vdp

For a finite isothermal process, in which T, V and ~ can all be treated as constants,

Q rrfdS=-~Tvrfdp
Pi Pi

- ~ T V (Pt - Pi)

Substitution of the values gives

Q -50.4 X 10- 6 (K- 1 ) X 100 (K) X 1.0 X 10- 6 (m3) X (1.3 X 108 (Pa) - 1.1 X 1o-' (Pa))
-0.655 J

The change flU in the internal energy of the system is given by

flU= Q +W
-0.655 (J) + 0.122 (J)
-0.53 J

The minus sign indicates that the internal energy of the piece of copper decreases, despite work
being done on it.

104
8.3 A block of copper is under a pressure of 1.1 x 105 Pa at a temperature of 293 K. Calculate the
change in the temperature of the copper when the pressure is increased reversibly and adiabatically
to 1.1 x 108 Pa.
For copper, cubic expansivity = 50.4 x 10- 6 K- 1 , specific heat capacity at constant pressure =
385 J kg- 1 K- 1 and density = 8.96 x 103 kg m- 3 •

Solution The system is the block of copper, which is a closed hydrostatic system. The system boundary
allows only a work interaction with the surroundings. Since the process is reversible and adiabatic
it takes place at constant entropy. Therefore, the appropriate adiabatic equation may be used to
determine how the temperature of the system changes with the pressure. The equation required is

where CP is the heat capacity of the system at constant pressure, and Vis the volume of the system
when its temperature is T and pressure p. This equation would appear to need a knowledge of the
equation of state for further progress but,

(l!V) (a VIa T)p

is the cubic expansivity ~ and so the equation may be written

Let the mass of the system be m and the specific heat capacity at constant pressure of the material
be cP. Then,

and, separating the variables in the equation,

dTIT = (~VImcp) dp (S constant)

Now the density p of the system is given by

p = m/V

so that

dTIT = (~/cP p) dp

If it is assumed that ~' p and cP can all be treated as constants, integration gives

where Pi and Ti are, respectively, the initial pressure and temperature and Pf and Tf are the
corresponding final values. Therefore,

ln(T 1293 (K)) = 50.4 X 10- 6 (K- 1 ) X (1.1 X 108 (Pa) - 1.1 x 105 (Pa))
f 385 (J kg- 1 K- 1 ) X 8.96 X 103 (kg m- 3)
= 0.001 605 5

and

1.001 606 6 K

or,

Tf = 293.47 K

Therefore, the change in temperature ll.T is

105
!lT = 293.47 - 293 = 0.47 K
Since this value of !1 Tis very small, it is a satisfactory approximation to treat T as a constant in
the differential equation and obtain

where !lp is the change in pressure.

8.4 Show that, for a closed hydrostatic system, the volume of which changes from V; to Vf when the
temperature changes from T; to Tf at constant pressure,

Vf = V; exp[~ (Tf- T;)]

provided that the cubic expansivity ~ is constant. If ~ is small, show that the expression may be
written

Vf = V; [1 + ~ (Tf - T;)]

Solution For a closed hydrostatic system the volume V may be treated as a function of the pressure p and the
temperature T and written

V = V(p,T)
Then, in an infinitesimal part of a reversible process, the change in volume dV is given by

dV = (avJap)r dp + (avJa T)p dT

If the process takes place at constant pressure dp is zero, and then dV is given by

dV = (aVJaT)p dT

Now the cubic expansivity ~ is defined by the equation

~ = (1/V) (a VIa T)p

so that, for a reversible process at constant pressure,

dV = ~ V dT

Separating the variables gives

dVIV = ~ dT
If the volume is V; when the temperature is T; and is Vf when the temperature is Tf, integrating the
above equation, assuming that ~ is a constant, gives

or

Vf = V; exp[~(Tf- T;)]

The exponential function exp(x) may be written as a series, when it becomes

exp(x) = 1 + (1/1!)x + (1/2!)x 2 + (1/3!)x 3 + ...

and, when x is small, this becomes

exp x =1+x
If ~ is assumed to be small, the expression for Vf becomes

106
8.5 Derive the following relations for a single-phase closed hydrostatic system.

(a) (aCvlaV)r = T(a(~IKr)taT]v


(b) dU = Cv dT + [(~TIKr) - p] dV
(c) dS = (Kr Cvi~T) dp + (Cpi~T V) dV
(d) (autaV)r = T 2 [a(p!T)NlT]v
(e) (avtaT)s = -(CvKri~T)
(f) a2 GtapaT = ~v

where the symbols have their usual meanings.

Solution One of the roles of thermodynamics is to provide relationships between coordinates and other
parameters. Such relationships are often of interest in themselves and frequently provide insights
into the behaviour of systems. They also provide a check on the consistency of measurements and
limit the number of experimental measurements that need to be made for a complete description of
the behaviour of a system by allowing values to be calculated from a relatively small number of
measurements that can be made under the most favourable conditions.
Students often have difficulty in deriving or verifying such relationships, the major difficulty
being knowing where to start. A successful solution is, to some extent, a matter of experience, but
the important possible starting points are the entropy form of the first law, the TdS equations or a
suitable reciprocity relation. All these may need to be supplemented by the use of the second law,
by Maxwell's relations or the reciprocal relation. Some of these approaches will be illustrated in
the answers to this example.
(a) A useful starting point when dealing with heat capacities is to recall that, in an infinitesimal
reversible process, the change dS in the entropy of a system is related to the heat dQ absorbed by
the system at a temperature T by the equation

dQ = TdS
When the process takes place at constant volume the above equation leads to

where Cv is the heat capacity of the system at constant volume. Therefore, differentiating both
sides of the equation with respect to V, keeping T constant, gives

The order of differentiation is immaterial as S is a state function. The entropy may now be
eliminated using Maxwell relation (M3)

so that

Now look at the right-hand side of the required relation. The cubic expansivity is defined by

~ = (l!V)(avtaT)p

and the isothermal compressibility Kr by

Kr =- (1/V)(avtap)r

so that

(avtaT)p
(avtap)r

107
Therefore, using the reciprocity relation

(av;ar)p (oT!ap)v (op!av)T = -1


~IKT may be written

Therefore,

and

(b) Two of the differentials of the required equation are the same as those in the entropy form of
the first law, so that this law may be tried as a starting point. Then

dU = TdS- pdV

where U is the internal energy and S is the entropy of a system occupying a volume V at a
temperature T and pressure p.
An expression for TdS may be obtained from the first TdS equation:

TdS = Cv dT + T(op/oT)v dV

where Cv is the heat capacity at constant volume. Substituting for TdS in the equation for dU gives

dU = Cv dT + T(opNJT)v dV - pdV

which is very close to the required form. Use the reciprocity relation

(ap!aT)v (aT!oV)p (oV!op)T = -1

to express (op/oT)v in the form

(aploT)v = -(oV!oT)pl(oV!ap)T
From part (a) of this example, the right-hand side is equal to ~IKT so that

dU = Cv dT + (T ~IKT) dV - pdV

or

dU = Cv dT + [(~TIKT) - p] dV

(c) The form of the required equation suggests that a suitable starting point might be to treat the
entropy S of the system as a function of pressure p and volume V. Then

S = S(p,V)

and,

dS = (oS!ap)v dp + (as;av)p dV

These partial derivatives cannot be transformed using Maxwell's relations, but they can be put into
more useful forms by means of the reciprocal relations

(aS/op)v (oploT)v (oT/oS)v =1

108
and

Substituting for (aS!ap)v and (as/aV)P gives

dS = (aS!aT)v (aT!ap)v dp + (as;ar)p (aT!aV)p dV

Multiplying through by T, and recalling that

and

gives

TdS = Cv(aT!ap)v dp + Cp(aT!aV)p dV


The reciprocity relation

(aT!ap)v (ap!av)r (av;ar)p = -1


enables a substitution to be made for (aT!ap)v. Further, (anav)p equals li~V, where~ is the
cubic expansivity. Then

TdS =- Cv(aT!aV)p (aV/ap)r dp + (Cpi~V) dV

or,

Now' the isothermal compressibility Kr equals - (1/V) (av lap h and, therefore,

(d) The entropy form of the first law is a suitable starting equation here:

dU = TdS- pdV
Therefore,

(aU/aV)r = T(aS/aV)r- p
Maxwell relation (M3) may be used to transform (aS/aV)n giving

(aU!aV)r = T(ap!aT)v- p

Now, looking at the right-hand side of the required relation, [a(p/T)/aT]v expands to
(liT) (ap!iJT)v - (p/T 2 ) so that

T 2 [a(p!T)!aT]v = T(ap/aT)v- p
Therefore,

(e) The partial derivative (aV!aT)s being [(aT/aV)s]- 1 may be transformed by a Maxwell
relation to give

(av!aT)s = l!(aT!aV)s = -1/(ap/aS)v


= - (as!ap)v = - (as!aT)v (aT/ap)v

Now, (aSNJT)v = CviT, where Cv is the heat capacity at constant volume. Therefore,

109
(iJV!·iJ1)s = - (Cvl1)(iJT/iJp)v

From part (a)

(iJT!iJp)v = Krlf3
so that

(f) A good starting equation here is that defining the Gibbs function G:

G = U + pV- TS

Then, in any infinitesimal process the change in G is dG, given by

dG = dU + pdV + Vdp - TdS - SdT

From the entropy form of the first law

dU = TdS- pdV
so that

dG = Vdp- SdT

Therefore,

(iJG/iJ1)P = - S

and

The right-hand side may be transformed using Maxwell relation (Ml) to give

and, since 13 = (1/V) (iJV!a1)P,

8.6 For a gas that obeys the van der Waals equation of state
[p + (a/V;..)] (Vm- b)= RT

determine the relationship between temperature T and molar volume Vm and between pressure p
and Vmin a reversible adiabatic process, assuming that the molar heat capacity at constant volume
is a constant. R is the molar gas constant and a and b are constants.

Solution The system is one mole of molecules of a gas that obeys van der Waals' equation. Relationships
between the primitive coordinates of the gas are sought when the gas undergoes a reversible
adiabatic process. In such a process the entropy of the system remains constant and this suggests
that an appropriate starting point is to consider treating the entropy S of the system as a function of
V and T. Then, for any closed hydrostatic system,

S = S(T,V)

and, in an infinitesimal part of a reversible process,

dS = (iJS/iJ1)v dT + (iJS/·iJV)r dV

110
Now, T (aS/aT)v = Cv, the heat capacity at constant volume, and Maxwell relation (M3) is

Therefore, for a closed hydrostatic system consisting of 1 mol of molecules

In a reversible adiabatic process dSm is zero so that

For a van der Waals gas

Therefore, in a reversible adiabatic process,

(Cv,miT) dT = -[R/(Vm- b)] dV

Integrating gives, assuming that Cv,m is a constant,

Cv,m loT= - R ln(Vm- b)+ constant

or,

In T = -(RICv,m) ln(Vm- b)+ constant


= ln(Vm- b)-Rtcv,m +constant

Therefore,

In T - In( V m - btRtcv,m = constant

or,

T(Vm- bt'cv,m = constant

Now T may be eliminated using the equation of state. This gives

[p + (a/V 2 )][(Vm- b)IR](Vm- bt'cv,m =constant

or,

[p + (a/V 2 )](Vm - b) 1 + Rtcv,m = constant

8.3 Ideal Gases An ideal gas is a closed hydrostatic system that obeys Boyle's law and Joule's law. When
temperature is measured using the thermodynamic temperature function the equation of
a system consisting of 1 mol of molecules of ideal gas is

where V m is the molar volume when the system exerts a pressure p at a temperature T
and R is the molar gas constant.
From Joule's law the internal energy of an ideal gas is a function of the temperature
only and, as has already been pointed out (p. 31), it follows that the increase dU in the
internal energy of a fixed mass of ideal gas in an infinitesimal process in which the
temperature increases by dT is given by

dU = Cv dT

111
where Cv is the heat capacity at constant volume. For a fixed mass of ideal gas with
constant heat capacities, the adiabatic equations become

TYp 1-Y = constant (8.9)

rvy- 1 = constant (8.10)

p VY = constant (8.11)

The constant has a different value in each of the Equations (8.9) to (8.11). y = CP!Cv =
Cp,m!Cv,m, where CP and Cv are the heat capacities at constant pressure and at constant
volume, respectively.

8.4 Worked
Examples on Ideal
Gases s.7 A quantity of oxygen, assumed to behave as an ideal gas, is initially in an equilibrium state i in
which it occupies a volume of 0.5 X 10- 3 m3 at a pressure of 1.1 x 10 5 Pa. The state of the gas is
then changed to an equilibrium state f, in which its volume is 2.0 x 10- 3 m3 , by three different
reversible processes:

(a) a reversible adiabatic process from state i to state f;


(b) a reversible isothermal process in which the volume increases to 2.0 x 10- 3 m 3 , followed by a
reversible isovolumic process until state f is reached;
(c) a reversible isovolumic process until the pressure is that of state f, followed by a process at
constant pressure until state f is reached.

Calculate the work done on the gas in each process. Assume CP!Cv = 1.4 for oxygen.

Solution The system is the fixed mass of oxygen, assumed to be an ideal gas. When any closed hydrostatic
system exerting a pressure p and occupying a volume V undergoes an infinitesimal reversible
process, the work w done on the system is given by

w = -p dV
and, in a finite reversible process, in which the volume changes from V; to Vr, the work W done on
the system is given by

W=-(tpdV
V;
To integrate this equation, the dependence of p on V must be known, that is, the path of the
process must be known.
(a) When the process performed by the ideal gas is both reversible and adiabatic the relation
between p and V is

p VY = constant = K
say, where y = CPICv. This process is represented by the line i-f on the pressure-volume graph
(Fig. 8.1). The work done on the system during the process is given by the shaded area.
Then,

W =- J K v-y dV
Vf

v;
= [K/(y - 1)](v~-Y - vt-Y)
Now,

so that

112
p

Figure 8.1 A reversible


adiabatic process for an ideal
gas. v

The values of V;, Vc and Pi are known, but Pc must be determined. Using the equation

pVY =K

Therefore,

Pt = 0.158 X 105 Pa

and

W = (1/0.4)(0.158 X 105 (Pa) X 2.0 X 10- 3 (m 3) - 1.1 X 105 (Pa) X 0.5 X 10- 3 (m 3 ))
=-59 J

(b) The second process is represented by the line i-f in Fig. 8.2. This process is in two stages:
i-a, representing the isothermal expansion and a-f, in which the volume of the gas remains
constant while the temperature is lowered.

Figure 8.2 A reversible


isothermal process i-a
followed by an isovolumic
process a-f. v

There is no pressure-volume work done on the gas in the part of the process a-f.
For the process i-a, the work W done on the system (shown by the shaded area in Fig. 8.2) is
given by

W=- JVf pdV


Vj

The pressure p is related to the volume V through the equation of state

pV = nRT

where n is the number of moles of molecules of gas present. Then,

113
W = - rt (nRT!V) dV
Vi
= - nRT ln(Vt!V;) (Tis constant)
= -p;V; ln(Vt!V;)
since p;V; = nRT. Therefore,

W = -1.1 X 105 (Pa) X 0.5 X 10- 3 (m3 ) X ln(2.0 X 10- 3 (m 3 )/0.5 X 10- 3 (m 3 ))
= -76J

(c) The third process is also in two stages: first a reduction in pressure at constant volume,
represented by the line i-a in Fig. 8.3, during which no pressure-volume work is done on the
system, followed by an expansion at constant pressure, represented by the linea-fin Fig. 8.3. The
work W done on the system during the process a-f is represented by the shaded area in Fig. 8.3 and
is given by

Pi----------- i

Figure 8.3 A reversible


isovolumic process i-a
followed by a reversible
isobaric process a-f v
since Pt is constant. Therefore,

W = -pt(Vt- V;)
= -0.158 X 105 (Pa) (2.0 X 10- 3 (m 3 ) - 0.5 X 10- 3 (m 3 ))
= -24 J

8.8 One mole of molecules of air in an equilibrium state, at a temperature of 300 Kanda pressure of
1.1 x 105 Pa, is compressed reversibly and adiabatically to a pressure of 2.0 x 105 Pa. Calculate the
new temperature of the mass of air and the work done on it during the compression.
After being stored at constant volume for long enough for the temperature to fall to 300 K again,
the air is used to drive an engine, thereby suffering a reversible adiabatic expansion in which its
pressure falls to 1.1 X 105 Pa. Calculate the fraction of the original work recovered, assuming that
air may be treated as an ideal gas with constant heat capacities, and that all the machinery used is
perfect. For air, take the ratio of the principal heat capacities to be 1.4. The molar gas constant is
8.315 J K- 1 mol- 1 •

Solution The system is 1 mol of molecules of air, assumed to behave as an ideal gas with constant heat
capacities. This system first undergoes a reversible compression. In this process, let the tempera-
ture of the system change from Ta to Tb and the pressure exerted by the system change from Pa to
Ph, as shown in Fig. 8.4.
To determine the value of Tb, the temperature of the state corresponding to point bin Fig. 8.4,
use is made of the adiabatic equation

or,

114
p

Pb ------------ b

Pc -----------~

Figure 8.4 Pressure-volume


graph showing the processes
undergone by the gas. v

Therefore,

300 (K) X (1.1 X 105 (Pa)t 0 .286 = Tb X (2.0 X lOS (Pa)t 0 ·286

giving

Tb = 356 K
The work W done on the system in this process is given by

or, since pV = nRT,


W = nR/(y - l)(Tb - Ta)
Then

W = (1 (mol))(8.315 (J K -• mol- 1)/0.4) (356 (K) - 300 (K))


= 1160 J

W is positive, as the work is done on the gas.


The gas is then allowed to stand at constant volume Vb until its temperature reaches 300 K. It is
then in the equilibrium state represented by the point c in Fig. 8.4. Since all equilibrium states of
the system must satisfy the equation of state, however they are reached, the pressure Pc in state cis
given by

with Vb = Vc. Substituting gives


Pc = (2.0 X lOS (Pa) X 300 (K))/356 (K)
= 1.69 X 105 Pa

When the temperature of the air has returned to 300 K, the air is expanded reversibly and
adiabatically to a pressure of 1.1 x 105 Pa, the process shown as c-d on Fig. 8.4. The temperature
Td corresponding to state d is found using the adiabatic equation

Then,

(300 (K)) 14 X (1.69 X 105 (Pa))-o.4 = T~.4 X (1.1 X 105 (Pa))- 0 .4

115
giving

Tct = 265.6 K

The work w1 done on the system in the process c-d is given by


W1 = [nRI(y- l)](Tct- Tc)
= (1 (mol) x 8.315 (J K- 1 mol- 1)/0.4) X (265.6 (K)- 300 (K))
= -715 J

The minus sign indicates that W1 is work done by the system. The fraction of the work done on the
gas that is recovered in this expansion is

715 (J)/1160 (J) = 0.62 = 62%


8.9 A horizontal cylindrical cylinder of area of cross-section 0.01 m 2 is divided into two portions, each
2.50 m long, by a frictionless piston. The space on one side of the piston contains hydrogen, while
the space on the other side of the piston contains helium. On both sides of the piston the gas
pressure is 1.0 x 105 Pa. The cylinder is then turned until its axis is vertical. When equilibrium is
achieved, the piston has moved 1.5 mm from its original position. Calculate the mass of the piston,
assuming that hydrogen and helium behave as ideal gases with constant heat capacities, that the
piston and cylinder walls constitute adiabatic boundaries and that the mass of each gas is negligible.
For hydrogen y = 1.4; for helium y = 1.67, where y is the ratio of the principal heat capacities.

Solution The system is the total mass of gas contained in the cylinder. It is isolated from the surroundings by
the rigid, adiabatic walls of the containing cylinder. When the axis of the cylinder is horizontal, the
frictionless piston is maintained in equilibrium as a result of the pressure in the two parts of the
cylinder being equal, and having the value p 0 , say. This situation is represented in Fig. 8.5.

tA«•A
~
Po Po

H2 He
Figure 8.5 The cylinder in II
the horizontal position. Lo Lo

When the cylinder has its axis changed to the vertical direction, equilibrium is achieved when the
force exerted on the piston by the gas in the lower part of the cylinder is equal to the force exerted
by the gas in the upper part of the cylinder plus the weight of the piston. Let m be the mass of the
P2 /.o +X
piston, p 1 be the pressure of the gas in the lower part of the cylinder and p 2 that of the gas in the
upper part when equilibrium is achieved with the cylinder axis vertical. This situation is repre-
He sented in Fig. 8.6. Then,

/.o-X
p, where A is the area of cross-sectional area of the piston and g is the acceleration of free fall. When
the cylinder axis is made vertical, the gas in the lower part of the cylinder suffers a decrease in
volume while the gas in the upper part suffers an increase in volume. If these two processes are
assumed to be reversible and adiabatic

Figure 8.6 The cylinder in Po X (L 0 X A)Y 1 = p1 X [(L 0 - x) X AjY 1


the vertical position.
where x is the distance moved by the piston and y1 is the value of y for the gas in the lower part of
the cylinder. Similarly, for the gas in the upper part of the cylinder

Po X (L 0 X A)Y2 = p 2 X [(L 0 + x) X AjY 2

Then, assuming that the gas in the lower part of the cylinder is hydrogen,

1.0 x lOS (Pa) X (1.25 (m) X 0.01 (m2 ))t. 4 =p1 x [(1.25 (m) - 0.0015 (m)) X O.Ql (m 2 W4

116
and

1.0 X 105 (Pa) X (1.25 (m) X 0.01 (m 2 )) 167 = p 2 X [(1.25 (m) + 0.0015 (m)) X 0.01 (m 2 }P 67

giving

p1 = 1.0017 X 105 Pa

and

p 2 = 0.9980 X 105 Pa

Therefore,

m = (p 1 - p 2) Alg
= [(1.0017 - 0.9980) x 105 (Pa) x 0.01 (m 2 )]/9.81 (m s- 2 )
= 0.38 kg

8.10 A Carnot engine has an ideal gas as the working substance. Show the states through which the
working substance passes in one cycle of operation using

(a) a pressure-volume graph;


(b) an internal energy-temperature graph.

Assume that the gas has constant heat capacities.

Solution Take the working substance of the Carnot engine as the system. In a Carnot cycle the system
undergoes a reversible isothermal expansion and a reversible isothermal contraction, these two
processes being linked by a reversible adiabatic expansion and a reversible adiabatic compression.
(a) For a fixed mass of an ideal gas a reversible isothermal process is described by Boyle's law,
that is, by the equation

where Pi and Pt are the initial and final pressures, respectively, and Vi and Vt are the initial and
final volumes, respectively. The line representing a reversible process on the p-V graph is,
therefore, a rectangular hyperbola. The equation of state of the system is

pV = nRT
where n is the number of moles of gas and Tis the temperature. Since n and R are constants, the
value of p V must be larger for the high-temperature isothermal process than for the low-
temperature isothermal process. Further, the slope of such a line is (dpi(JV)n which, from the
equation of state, is equal to - (p!V).
In a reversible adiabatic process the behaviour of a fixed mass of ideal gas with constant heat
capacities is described by the equation

where y = CP!Cv. Since CP is always greater than Cv, the value of y is always greater than unity.
The slope of the curve representing a reversible adiabatic process in the p-V plot is (ap!aV) 5 ,
which, from the equation describing the process, is equal to - yp/V. Therefore, the slope of the
line representing a reversible adiabatic process in the p-V plane is always steeper than the line
representing a reversible isothermal and passing through the same point. The appearance of the
Carnot cycle in the p-V plane when an ideal gas is used as the working substance is shown in Fig.
8. 7, where y has been given the value 1.4. In this plot, ab and cd are reversible isothermals and be
and da are reversible adiabatics.
(b) In a reversible isothermal process the internal energy of a fixed mass of an ideal gas does not
change with volume, as the internal energy of such a system is a function of temperature only.
Therefore, on the internal energy-temperature graph, the two reversible isothermal processes are
represented by points, such as a and bin Fig. 8.8.

117
p

I
I

''
''
a•'
'
''\
\
\
\
\
\
\
\
\
\
\
\\
\

\,',,,

Figure 8. 7 The Carnot cycle


represented in the p-V plane
when an ideal gas is the
working substance. v

Figure 8.8 The Carnot cycle


plotted in the U-T plane when
an ideal gas is the working
substance. T

In an infinitesimal reversible process in which the temperature of a fixed mass of an ideal gas
increases by dT, the internal energy increases by dU, given by

dU = CvdT

where Cv is the heat capacity at constant volume for the chosen mass of gas. If Cv is treated as a
constant, integration gives

U = Cv T + constant

Therefore, for such a gas, any process in which T changes is represented by a straight line in the
U-T graph. This is true of the reversible adiabatic processes in a Carnot cycle, which are therefore
represented by a straight line joining the points a and bin the U-T graph. The line is traversed
from a to b when the temperature rises and from b to a when the temperature falls.

8.11 An evacuated vessel with rigid adiabatic walls is fitted with a stopcock. When the stopcock is
opened slightly air flows in from the atmosphere, where the temperature is T0 , until equilibrium is
achieved. If the vessel has negligible thermal capacity, show that the equilibrium temperature of
the gas in the vessel is yT0 , where y is the ratio of the principal heat capacities.

Solution This is related to the problem examined as Example 2.5, except that the walls of the initially-
evacuated vessel are now adiabatic. Take as the system the mass of gas that flows from the

118
atmosphere into the cylinder. In Example 2.5 it was shown that, if V0 is the volume of the vessel,
the work W done on the gas as it flows into the vessel is given by

W =Po Vo

where p 0 is the atmospheric pressure. For an infinitesimal part of the equivalent notional reversible
process the first law is

dU =q + w
where q is the heat absorbed by the system and w is the work done on the system. Since, in this
example, the walls of the vessel are adiabatic, q is zero and, therefore,

dU= w

For the complete process (whether notional or real, since both start and end in equilibrium states
and U is a state function)

fl.U =W
For any ideal gas with constant heat capacities

fl.U = Cv fl.T

Therefore,

where Tis the final equilibrium temperature of the gas in the cylinder. But, from the equation of
state,

PoVo = nRT0
so that

Also, from Example 6.15,

Therefore,

giving

or

T = yT0

8.5 Perfectly Elastic Equilibrium states of a system consisting of a cylinder of a perfectly elastic material
Solids under an axial load are specified by the values of the coordinates axial load F, tempera-
ture T and length of the cylinder L. The totality of equilibrium states is represented by a
continuous surface in F-L-T space, the equation of which is the equation of state of the
system.
When the length of a cylinder in an equilibrium state under an axial load is increased
reversibly by an amount dL, while the load remains sensibly constant, the work dW done
on the system is given by

119
dW= FdL (8.12)

provided that pressure-volume work is negligible, that is, either that the extension takes
place in a vacuum or, more realistically, that changes in volume are negligible. The first
law for an infinitesimal extension of the perfectly elastic rod may then be written

dU = dQ + FdL (8.13)

where dU is the change in the internal energy of the system and dQ is the heat absorbed
by the system during the infinitesimal reversible process.
In a finite reversible process, in which the length of the cylinder changes from Li to Lt.
the work W done on the system is given by

(8.14)

In general, L may be written as a function of F and T, so that the change dL in the


length resulting from an infinitesimal change in both F and T may be obtained from the
differential. Writing

L = L(F,T)

gives

The linear expansivity A. of the material is given by

A.= (1/L) (iJL!iJT)F

and Young's modulus E by

E = (L!A) (iJF/iJL)r

where A is the area of cross-section of the cylinder. Therefore,

dL =LA. dT + (LIAE) dF (8.15)

When the cylinder undergoes an infinitesimal reversible isothermal extension it absorbs a


quantity of heat dQr given by

dQr = TAE A. dL (8.16)

When the cylinder undergoes an infinitesimal reversible adiabatic (isentropic) extension


it undergoes a change in temperature dT5 , obtained from

(8.17)

where CL ( = T(iJS!iJDJ is the heat capacity at constant length.

8.6 Worked
Examples on
Perfectly Elastic
Sol idS 8.12 The equation of state of a certain perfectly elastic material in the form of a rod, subjected to an
axial load F, is

120
where L 0 is the unstretched length, L is the length under a load F at a temperature T and C is a
constant. Derive expressions for the linear expansivity and the Young's modulus of the material.

Solution The system is the rod of perfectly elastic material. For such a system the linear expansivity A is
defined by the equation

A = (1/L) (iJL/iJT)F

and the Young's modulus E by the equation

E = (LIA) (iJF!iJL)T
where A is the area of cross-section of the rod.
The problem reduces to the determination of (iJL!iJT)F and (iJFiiJL)T.
Write the equation of state in the form · -

F = CT(L2 - 2LL 0 + L~)


Then

(iJF/iJL)T = CT(2L- 2L0 ) = 2CT(L- L 0 )

so that
E = 2CT(LIA)(L - L 0 )
= (2CT/A)(L 2 - LL0 )
Writing the equation of state in the form

L2 - 2LL0 + L~ = FICT

gives, on differentiating with respect to T, keeping F constant,

2L(iJL/iJT)F - 2L0 (iJLiiJT)F = - FICT 2

so that

Therefore,

8.13 The unstretched length of a certain steel rod is 5.00 m when the temperature is 300 K and its area
of cross-section is 5.0 x 10- 4 m2 • The axial stress applied to the rod is then increased reversibly
from zero to 3.0 X 108 Pa. ·

(a) If the process occurs at constant temperature, calculate the work done on the rod, the heat
absorbed by the rod and the change in the internal energy of the rod.
(b) If the process occurs adiabatically from a temperature of 300 K, calculate the change in the
temperature of the rod.

Steel has the following properties: density = 7860 kg m- 3 ; Young's modulus (assuming that
Hooke's law is obeyed) = 2.08 X 1011 Pa; linear expansivity = 1.20 X 10- 5 K- 1 and heat capacity
at constant load= 482 J kg- 1 K- 1 •

Solution The system is the length of steel rod, initially at a temperature of 300 K.
(a) When the system undergoes a reversible isothermal extension work W is done on it, it
absorbs heat Q and its internal energy changes by d U, given by

121
!l.U = Q + W

In an infinitesimal part of this process the work dW done on the system is given by

dW = FdL

where F is the axial load and L is the length of the system. The heat absorbed by the system in the
infinitesimal reversible isothermal process is dQr. given by Equation (8.16)

dQr = TAE/.. dL
The dependence of L on F is shown in Fig. 8.9 for a material of unstretched length L 0 that obeys
Hooke's law.

F, --------------------------------------·------------------

Figure 8.9 The graph ofF


against L for a Hookean
material of unstretched length
Lo. L

The work done on the system in an infinitesimal part of the process, when the length increases by
dL while F remains sensibly constant, is shown by the shaded area in Fig. 8.9. The total work W
done on the system by a gradually applied load F 1 , which extends the length to a value of L 1 , is
given by the area under the curve for values of L up to L 1 • Then,

But,

where a is the axial stress, and A is the area of cross-section, so that

W = hF; L 0 /AE)
!(<T L 0 AlE)
1 (3.0 X 108 (Pa)) 2 X 5.00 (m) X 5.0 X 10- 4 (m 2)
2 2.08 X 1011 (Pa)
= 541 J

The value of Qr is given by

so that (L 1 - L 0 ) must be determined. Using Hooke's law

a = E(L 1 - L 0 )1L0
3.0 X 10 (Pa) = 2.08
8 X 10 11 (Pa) X (L 1 - L 0 )!5.00 (m)

so that

122
(L 1 - L 0 ) = 7.212 X 10- 3 m

Therefore,

QT = 300 (K) X 5.0 X 10- 4 (m2 ) X 2.08 X 1011 (Pa) X 1.20 X 10- 5 (K- 1 )
X 7.212 X 10- 3 (m)
= 2700 J
and AU = Qr + W = 2700 (J) + 541 (J) = 3240 J
(b) The change in temperature ATs resulting from the reversible adiabatic extension may be
obtained from Equation (8.17), assuming that AT is small and that T, A, E, A and CL are all
constant. Then

For this calculation CL must be determined. Now

where m is the mass of the rod and cL is the specific heat capacity of the material at constant load.
The value of m is obtained from

m = pV

where p is the density of the material and Vis the volume of the rod. Then

m = 7860 (kg m- 3 ) X 5.00 (m) X 5.0 X 10- 4 (m2 )

and

ATs = - (300 (K) X 5.0 X 10- 4 (m2 ) X 2.08 X 1011 (Pa) X 1.2 X 10- 5 (K- 1 ) X 7.22
X 10- 3 (m))..;. (7860 (kg m- 3 ) X 5.00 (m) X 5.0 X 10- 4 (m2 ) X 482 (J kg- 1 K- 1))
= -0.29 K

8.14 A steel wire, having an area of cross-section of 1.00 x 10- 6 m2 , is stretched between two rigid
supports 1.5 m apart. When the temperature is 298 K the tension in the wire is 20.0 N. Calculate
the tension in the wire when the temperature falls to 288 K. Assume that steel is perfectly elastic
and obeys Hooke's law. For steel, Young's modulus= 2.08 X 1011 Pa and linear expansivity = 1.20
X 10- 5 K- 1 .

Solution The system is the steel wire. Let the equilibrium length of the wire beL when it is under a tension
Fat a temperature T. In an infinitesimal reversible process in which the tension is increased by an
amount dF and the temperature by an amount dT, the change in the length dL is given by Equation
(8.15):

dL = (LIAE) dF + AL dT

where E is Young's modulus for the material, A is its linear expansivity and A is the area of
cross-section of the wire.
If the system is constrained to be of constant length then, in an infinitesimal reversible process,
dL is zero and

AL dT = - (LIAE) dF

or,

AdT = - (l!AE) dF

123
In a finite reversible process, assuming that A, E and f... may be treated as constants, the change in
the tension !iF is related to the change in the temperature liT by the equation

f... liT= - (l!AE) !iF

or,

where Ti is the initial temperature of the system and Tt is its final temperature. Then

!iF= - (1.20 X 10- 5 (K- 1 ) X 1.00 X 10- 6 (m2 ) X 2.08 X 1011 (Pa) X (288 (K) - 298 (K))]
= 24.96 N
Therefore, the new value of the tension is

F = 20.0 (N) + 24.96 (N)


= 45.0 N

8.15 A cylindrical wire, of unstretched length L 0 at a temperature of 273 K, is fixed between two rigid
supports a distance L apart. If f... is the linear expansivity of the material of the wire and ~ is the
temperature coefficient of Young's modulus, find the condition for the tension in the wire to be
independent of temperature.

Solution The system is the cylindrical wire of unstretched length L 0 at a temperature of 273 K. If the area of
cross-section of the wire at a temperature of 273 K is A 0 , the tension F0 when it is stretched
between supports a distance L apart is given by

where £ 0 is the value of Young's modulus at 273 K. At some other temperature Tthe tension Frin
the wire is given by

where Lr is the unstretched length at a temperature T, when the area of cross-section is Ar and
Young's modulus is Er- Now, provided that (T- T0 ) is small (see the approach in Example 8.4),

Lr = L 0 [1 + f...(T- T0 )]
AT = A 0 [1 + 2/...(T - T0 )]

and

(The surface expansivity is twice the linear expansivity.)


The condition to be satisfied is Fr = F0 , that is,

Substitution gives

(A 0 E 0 1L0 )(L - L 0 ) = {A 0 [1 + 2J..(T - T0 )]Eo[l + ~(T- To)]}


x {L - L 0[1 + f...(T - T0 )]}1L0 [l + f...(T- T0)]
Writing T- T 0 = liT and expanding gives
(L - L 0 )(1 + f... liT) = (1 + 2 f... !l.T)(l + ~ !l.T)[L - L 0 (1 + f... liT)]

124
Neglecting terms in J...Z and A~, assuming that both A and ~ are small, gives

La A !J..T + La A !J..T- LA !J..T = L ~ !J..T- La~ !J..T

or

IJ~ =- (L - La)I(L - 2La)

8.16 The equation of state of a certain perfectly elastic material in the shape of a·rod is

where La is the unstretched length and L is the length under a load F, both at a temperature T, and
K is a constant.

(a) If A is the area of cross-section of the rod, show that Young's modulus E for the material is
given by

E = (FIA) + (3KTL~)IAL2
(b) Show that the linear expansivity A of the material is given by

A= Aa - (F!AET)

where

Aa = (l!La)(dLaldT)

Solution The system is the rod of perfectly elastic material of unstretched length La at a temperature T.
(a) Neglecting any change in volume with change in length produced by an axial load F,
Young's modulus E for the material of the rod is given by

E = (L!A)(iJF/iJL)T
where Lis the length of the rod. The quantity (iJF!iJL)T needs to be determined and is obtained
from the equation of state:

Then

and E is given by

E = (LIA)[(KT!La) + (2KTL~/L 3 )]

= (KTL/ALa) + (2KTL~!Ae)
= (KTLIALa) - (KTL~IAL 2) + (3KTL~/AL 2)
= (KTIA)[(L!La} - (La/L} 2] + (3KTL~IAL2}
= F/A + (3KTL~IAL2}

(b) The linear expansivity A is defined by

A = (1/L)(oL/iJT)F

Here (iJL!iJT)F needs to be determined from the equation of state. Put this in the form

LIL0 - L~IL 2 = FIKT

125
Then

giving

or

(aLl aT) = -'-(d_L...::.0 l_d_T-'-')[-'-L_I_L~"'-)_+____,_(2_L.::...


( 0 I_L-:-'2 )-'--]
- _-_F._'!K_T_2
F [(l!L 0 ) + (2L~/C)]

Remember that L 0 can only depend on T. Therefore,

(l/L 0) (d/L 0 /dT) - FI(LKT2 )[(l!L0 ) + (2L~/L 3 )]


= Ao - F/(LT)[(KTIL 0 ) + (2KTL~/L 3 )]

which, using the result forE from part (a), becomes

f... = Ao - (FLILTEA)
= A0 - (FIAET)

8. 7 Voltaic Cells When rods of two different metals are placed in an electrolyte, a p.d. is developed
between the rods. The rods are known as electrodes and the electrode from which the
conventional (positive) current flows into the external circuit is known as the cathode.
The other electrode is called the anode. The arrangement of electrodes and electrolyte
constitutes a voltaic cell.
For a voltaic cell to operate reversibly the reactions that occur when the current flows
in the external circuit from anode to cathode must be reversed when the current flows
from cathode to anode. This condition eliminates secondary cells from consideration and
also primary cells in which the reaction produces a gas phase.
When a voltaic cell is in an equilibrium state the e.m.f. of the cell is, in general, a
function of the temperature T of the cell and the state of charge Z. The absolute value of
the state of charge is unimportant but, when the charge flow around the external circuit is
thermodynamically reversible, the work dW done on the cell when a charge dZ flows
from the anode to the cathode in the external circuit is given by

dW = E dZ (8.18)

In a finite reversible process, in which a charge Z 1 is transferred from the anode to the
cathode in the external circuit, as a result of a current I flowing for a time t1 under a p.d.
E, the work W done on the cell is given by

W = Jzl E dZ = J'J EI dt
0 0

The first law of thermodynamics for a reversible charge flow becomes

dU = dQ + E dZ (8.19)

where U is the internal energy of the cell and dQ is the heat absorbed by the cell in an
infinitesimal reversible process.
It may be shown that, in a general reversible process,

126
dU = C2 dT + [E- T(<1E/<1T) 2 ] dZ

where Cz = T( <1S/<1T)z is the heat capacity of the cell at a state of constant charge. When
the movement of charge is both reversible and isothermal the cell absorbs heat during the
process, the amount dQT absorbed in an infinitesimal reversible isothermal process being
given by

dQT = - T( <1E/<1T)z dZ

For cells with saturated electrolytes E is a function of T only and then

dU = C2 dT + [E- T(dE!dT)] dZ

and

dQT = - T(dE!dT) dZ

When a charge Z is passed through an electrolyte the mass m of material liberated is


given by

where Mr is the relative atomic mas5, v is the valence and 7 is a constant called the
Faraday and which is the charge which liberates the equivalent mass in kilograms.

8.8 Worked
Example on
Voltaic Cells 8.17 The e.m.f. of a Daniell cell at a temperature of 273 K is 1.0934 V and the temperature coefficient is
-0.4533 mV K- 1 • Calculate the heat of reaction of the cell.

Solution The Daniell cell is the system to be considered. A Daniell cell is a thermodynamically reversible
cell with a copper anode and a zinc cathode, in which the net reaction at the electrodes may be
written

Zn + CuS04 ~ Cu + ZnS0 4
The valence for both zinc and copper is two.
For a voltaic cell with saturated electrolytes undergoing an infinitesimal reversible transfer of
charge dZ, the change in the internal energy is given by

dU = C2 dT + [E - T(dE/dT)] dZ
Here, E is the e.m.f.of the cell at a temperature T, and C2 is the heat capacity at constant charge.
When there is no volume change of a system in which a reaction is taking place at constant
temperature, the change in the internal energy AU is equal to AH, the change in the enthalpy of
the system. Movement of charge in the external circuit is a consequence of chemical reactions
taking place in the cell. When the system is such that the reactants have the same valence v, one
mole of each of the metals passes into solution at constant volume and constant temperature when
a charge AZm is drawn from the cell. The corresponding change in molar enthalpy Hm is known as
the molar heat of reaction. Therefore, the change in Hm is given by

AHm = [E- T(dE/dT)] AZm

AZm is equal to -v7, where 7is the Faraday, equal to 9.65 X 104 C.
For a Daniell cell at a temperature of 273 K

E = 1.0934 V

127
dE!dT = -0.453 mV K- 1
AZm = -2 X 9.65 X 104 C

Therefore,

AHm = [1.0934 (V)- 273 (K) X (-0.453 X 10- 3 (V K- 1))] X (-2 x 9.65
X 10 (C mol-
4 1 ))

= -235 x 103 Jmol- 1

8.9 Exercises
8.1 A system consists of 10.0 kg of mercury at a temperature of
273 K and under a pressure of 1.1 X 105 Pa. Calculate the heat 1 2 3
absorbed by the mercury when the pressure is increased revers-
ibly and isothermally to 1.1 x 108 Pa. w -4214 J 0 +5257 J
For mercury, cubic expansivity = 181 x 10- 6 K- 1 and density Q 0 4214 J -5257 J
= 13.6 x 103 kg m- 3 • [ -4000 J]
AS 0 9.56JK- 1 -9.56 J K- 1

8.2 At a temperature of 298 K and a pressure of 1.1 x 105 Pa,


the molar volume of mercury is 14.72 x 10- 6 m3 , the cubic 8.5 Show that, for a fixed mass of an ideal gas, exerting a
expansivity is 181 X 10- 6 K- 1 and the isothermal compressibility pressure p at a temperature T, the cubic expansivity ~ is given by
is 38.9 x 10- 12 Pa- 1 • The pressure on a system consisting of
1 mol of mercury is increased reversibly and isothermally until the ~=liT
molar volume has decreased by 0.1%. For this process calculate:
and the isothermal compressibility I<r is given by
(a) the increase in pressure needed to bring about the change;
(b) the change in the internal energy and in the entropy of the
system;
(c) the heat absorbed by the system and the work done on the 8.6 A compressor takes in air at a pressure of 1.1 x 105 Pa and
system in the process. a temperature of 290 K and delivers compressed air at a pressure
of4.5 x 105 Pa and a flow rate of0.81 x 10- 3 m 3 s- 1 • Assuming
[25.7 MPa; -20.2 J; that air may be treated as an ideal gas and that the compression
-0.068 J K-\ -20.4 J; 0.19 J] process is reversible and adiabatic, calculate

8.3 For a single-phase closed hydrostatic system derive the (a) the volume of air taken in per second by the compressor;
following relationships: (b) the temperature of the air after compression;
(c) the power needed to run this (ideal) compressor.
(a) (aCJaP)r = - T[a(IW)taT]p
For air, the ratio of the principal heat capacities is a constant
(b) (aH!O.p)r = - T 2 [a(VIT)taT]p equal to 1.4. [2.2 x 10- 3 m 3 s- 1 ; 432.5 K; 307.5 W]
(c) WG!ap 2 )p =- CP!T
8. 7 Calculate the change in the temperature of a nickel wire of
(d) (apNJT)s = CPIV~T length 2.00 m and area of cross-section 2.00 x 10- 6 m2 , when it
is extended reversibly and adiabatically to a final load of 4905 N,
The symbols have their usual meanings. the initial equilibrium temperature being 293 K.
For nickel, specific heat capacity at constant length = 0.44 x
8.4 A system consisting of 1 mol of molecules of an ideal gas 103 J kg- 1 K- 1 and linear expansivity = 3.0 X 10- 6 K- 1 •
with a constant molar heat capacity at constant pressure, equal [ -0.56 K]
to 29.6 J K- 1 mol- 1 , undergoes a cycle consisting of the follow-
ing consecutive processes. 8.8 The equation of state of a certain perfectly elastic material
in the form of a rod is
1. The gas expands reversibly and adiabatically from an equilib-
rium state in which its pressure is 5.0 x 106 Pa and its
temperature is 550 K, to an equilibrium state in which its
pressure is 1.0 x 106 Pa. where L 0 is the unstretched length and L is the length under an
2. The temperature of the gas is then raised reversibly at con- axial load Fat a temperature T, and K is a constant, equal, for
stant volume until it reaches 550 K again. this material, to 1.33 X 10- 2 N K- 1 •
3. The gas is then compressed reversibly and isothermally until Calculate the work done on the rod if L 0 is 1.00 m and the rod
it returns to its original equilibrium state. is extended reversibly and isothermally so that L is 2.00 m, the
temperature being 300 K. [3.99 J]
For each process calculate the work W done on the gas, the
heat Q absorbed by the gas and the change AS in the entropy of 8.9 A steel wire is in equilibrium at a temperature of 300 K and
the gas. Assume that the ratio of the principal heat capacities is a has a length of 1.00 m and an area of cross-section of 2.0 x
constant equal to 1.4. 10- 6 m 2 under zero load. The load is increased reversibly at

128
constant temperature to a value of 100 N. Calculate the work 8.11 In the silver-mercury voltaic cell the reaction within the
done on the wire in this process, the heat absorbed by the wire cell may be written
and the change in its internal energy.
For steel, Young's modulus = 2.00 X 1011 Pa and linear Ag + ! Hg2 Cl2 ~ Hg + AgCl
expansivity = 1.20 X 10- 5 K- 1 • [0.013 J; 0.360 J; 0.373 J]
The valences of both silver and mercury are unity. At a tempera-
8.10 A reversible voltaic cell is charged by passing a current of ture of 298 K the e.m.f. of the cell is 0.0455 V and the rate of
40 A through it under a p.d. of 10 V for 45 minutes. During this change of e.m.f. with temperature is 0.338 mV K- 1 • Calculate
process the heat loss from the cell to the surroundings is 300 kJ. theheatofreactionofthecell. [SA kJ mol- 1]
Calculate the change in the internal energy of the cell, assuming
that the only form of work involved is electrical work. [780 kJ]

129
Topic Guide

9 Heat Capacities and


9.1 Introduction
9.2 Heat Capacities
9.3 Worked Examples on Heat

9.4
9.5
Capacities
Equations of State
Worked Examples on
Equations of State
Equations of State
9.6 Exercises

9.1 Introduction For a closed hydrostatic system the internal energy and the entropy can be completely
determined when the equation of state and one of the principal heat capacities are
known. This knowledge gives complete information about the thermodynamic behaviour
of the system. The experimental programme necessary to describe the behaviour of a
closed hydrostatic system would appear, then, to require the determination of the
equation of state and the principal heat capacities for all equilibrium states of the system.
Values of the coordinates to which the equation of state must be fitted are measured at
equilibrium. In contrast, heat capacity measurements are made under conditions that
generally depart from equilibrium. Further, while the measurement of the heat capacity
at constant pressure CP is straightforward, the measurement of the heat capacity at
constant volume Cv is practically impossible, because of the difficulty of keeping the
volume of the system constant when the temperature is changed.
However, one of the roles of thermodynamics is to determine useful relationships
between the values of the coordinates and other parameters, and it will be shown that the
principal heat capacities are related through the equation of state so that, once CP is
known for each equilibrium state, the values of Cv may be calculated when the equation
of state is known. Further, it will be shown that the way that CP varies with pressure and
volume, at constant temperature, can also be found from the equation of state so that CP
needs to be measured for only one (convenient) equilibrium state on each isotherm.
The equation of state of a system is the equation of the surface representing the totality
of equilibrium states of the system. This surface can always be represented by sets of
tabulated coordinates, but it is often convenient to have an equation of state in analytical
form. Some examples of commonly used equations will be given.

9.2 Heat Capacities When the absorbtion of a quantity of heat Q produces a rise in temperature Ll T in a
closed system, the heat capacity C of the system at a temperature Tis defined by

C= lim Q!ilT (9.1)


l:!.T-o

Since, for a given value of Q, the value of ilT depends on the nature of the process,
further specification of C is necessary. For closed hydrostatic systems two so-called
principal heat capacities are found to be useful. For reversible processes these are
defined as follows (see Section 4.7).
The heat capacity at constant pressure CP is defined by

CP = lim QP!ilT = dQP/dT (9.2)


f'l.T__,. 0

where QP is the heat absorbed under conditions of constant pressure.


The heat capacity at constant volume Cv is defined by

Cv = lim QvlilT = dQvldT (9.3)


tl.T- 0

130
where Qv is the heat absorbed under conditions of constant volume.
When a closed hydrostatic system undergoes an infinitesimal reversible process, dQ is
given by the first law:

dQ = dU + pdV
where dU is the change in the internal energy of the system. Therefore,

Cv = dQvldT = (oU/oT)v (9.4)

and

CP = dQP/dT = (oU/oT)P + p(oV/oT)p (9.5)

= (oH!oT)p (9.6)

where H ( = U + p V) is the enthalpy of the system. Also, in a reversible process


dQ = TdS

so that

Cv = T(oS/oT)v (9.7)

and

(9.8)

9.3 Worked
Examples on
Heat Capacities 9.1 Show that, for a closed hydrostatic system, both principal heat capacities may be determined for all
equilibrium states of the system, provided that the heat capacity at constant pressure is measured
for one state on each isotherm and that the equation of state is known.

Solution The system is a closed hydrostatic system, each equilibrium state of which is specified by a
particular set of values of the primitive coordinates pressure p, volume V and temperature T. The
requirement of the problem indicates that first a relationship should be found between the heat
capacity at constant pressure CP and that at constant volume Cv. Then the way that CP varies along
an isotherm should be examined.
In view of Equations (9.7) and (9.8), a convenient starting point to examine the relationship
between CP and Cv is the entropy of the system. The entropy S of the system may be treated as a
function of any two of the primitive coordinates. It is convenient to choose T and V. Then, S may
be written

S = S(T,V)

In any infinitesimal reversible process the change in entropy is dS, given by

dS = (aS!aT)v dT + (as;av)T dV (9.9)


From Equation (9.8)

(9.10)

and Maxwell relation (M3) is

(as;av)T = (ap!aT)v (9.11)

131
Therefore, substituting into Equation (9.9) from Equations (9.10) and (9.11) gives

dS = (Cv!T) dT + (ap!aT)v dV (9.12)

which is, in fact, the first TdS equation. Now differentiate Equation (9.12) with respect to T,
keeping p constant. Then

From the definition of CP (Equation (9.7)),

so that

CP/T = CvfT + (ap!aT)v (aV/aT)p


or,

cp - Cv = T(ap!aT)v (av;ar)p (9.13)

Both partial derivatives on the right-hand side of Equation (9.13) can be obtained from the
equation of state. Therefore, if the equation of state is known, Cv may be calculated for each
equilibrium state for which CP is known. From an experimental standpoint, Equation (9.13) is
somewhat unsatisfactory, as it involves (ap/aT)v, which is difficult to determine precisely. How-
ever, the reciprocity relation

(op!aT)v (ar;av)p (av!ap)r = -1


enables Equation (9.13) to be put in the more convenient experimental form

cp - Cv =- T(av;ar)~ (ap!a.v)r (9.14)

which may be written

where 13 is the cubic expansivity and Kr is the isothermal compressibility.


Now consider the dependence of CP on p.

and, since the order of differentiation is immaterial for real functions such as the entropy,

(9.15)

Now, from Maxwell relation (M1),

(as;ap)r =- (av;ar)p

and, substituting in Equation (9.15), gives

(9.16)

The partial second derivative can be determined from the equation of state. Therefore, when the
equation of state is known and the value of CP for one state on each isotherm is known, CP may be
calculated for all states corresponding to points on that isotherm. Then, using Equation (9.13), the
values of Cv for all equilibrium states of the system can be found.

9.2 Determine the dependence of CP on p for a system consisting of an ideal gas.


132
Solution The dependence of the heat capacity at constant pressure CP on the pressure p for any closed
hydrostatic system is given by Equation (9.16):

For a system consisting of n moles of molecules of an ideal gas the equation of state is

pV = nRT

where Vis the volume of the gas at a temperature T and R is the molar gas constant. Therefore,

(oV/oT)p = nR/p
and

since n, Rand pare all constants in the differentiation. This result shows that, for an ideal gas, CP is
independent of pressure. This conclusion should also hold for real gases at vanishingly small
pressures.

9.3 A certain gas has a molar volume Vmat a pressure p and a temperature T and obeys the equation
of state

pVm = RT + B(T)p
where R is the molar gas constant and B(T) is a function of temperature only. Show that, when
dB/dT is small,

Cp,m- Cv,m = R + 2p (dB/dT)


approximately. Cp,m is the molar heat capacity at constant pressure and Cv,m that at constant
volume.

Solution The system is 1 mol of molecules of the gas. Since the equation of state is known, it is convenient to
start with Equation (9.14) which, applied to 1 mol of molecules, becomes

The equation of state

pVm = RT + B(T)p

may be written

Vm = RT/p + B(T)
giving

Writing the equation of state in the form

p =- (RT)/(Vm- B)

gives

(op/oV)T =- (RT)I(Vm - B) 2 = - pi(Vm - B)

Then,

Cp,m- Cv,m = -T[(Rip) + dB/dT] 2 [-pi(Vm- B)]

133
=[(Rip?+ 2(R/p)(dB/dT) + (dB/dTf][pTI(Vm- B)]
If dB/dT is small, (dB/dT) 2 may be neglected. Then,

Cp,m - Cv,m = R 2 T/p(Vm - B) +. 2RT (dB/dT)I(Vm - B)

Now, RTI(Vm- B)= p, so that

Cp,m- Cv,m = R + 2p(dB/dT)


9.4 Show that, for a gas with the equation of state
[p + (a/V~)] (Vm- b)= RT

the change in the molar entropy dSm in any infinitesimal process from an equilibrium state is given
by

dSm = Cv,m d(lnT) + R d[ln(Vm- b)]

Here p is the pressure exerted by the gas when the molar volume is Vm at a temperature T, a and b
are constants, R is the molar gas constant and Cv,m is the molar heat capacity at constant volume.

Solution For a system consisting of 1 mol of molecules, Equation (9.12), which gives the change in the
entropy of a closed hydrostatic system in an infinitesimal process in terms of the changes in the
volume and in the temperature, may be written

To determine dSm it is necessary to determine (ap/aT)v· The equation of state may be written in
the form

p = [RT/(Vm- b)] - (a/V;,)

Then

(ap!aT)v = RI(Vm- b)
so that

dSm = (Cv,miT) dT + [RI(Vm- b)] dVm

This result may be written

dSm = Cv.m d(lnT) + R d[ln(Vm- b)]

9.5 A system consisting of two moles of molecules of an ideal gas with constant heat capacities
undergoes a process from an equilibrium state in which the pressure is 1.1 x 105 Pa and the
temperature is 300 K to one in which the pressure is 2.0 x 105 Pa and the temperature is 700 K. If
the molar heat capacity at constant pressure Cp,m is equal to 20.97 J K- 1 mol-\ calculate the
change in the enthalpy and in the entropy of the system produced by this process.

Solution The system is 2 mol of molecules of an ideal gas. It undergoes a certain process, the initial and final
equilibrium states of which are shown in Fig. 9.1 by the points i and f, respectively.
Since the enthalpy Hand the entropy S are state functions, the change in both HandS, in going
from equilibrium state i to equilibrium state f, is independent of the path. Therefore, for the
purposes of calculation, any convenient path between these two equilibrium states may be chosen.
It is convenient to choose the two-stage path iaf, shown by the dashed line in Fig. 9 .1. This consists
of an isobaric reversible process from state i to the state represented by the point a, followed by a
reversible isothermal process from state a to state f.

134
p(Pa)

2.0x10 5 ----------------------------------- f

Figure 9.1 The initial state i ____________ !.__________ .,._ ________ a


and final state f of the 1.1X105
process, shown on the
pressure p-temperature T
graph. 300 700 T(K)

It follows from Equation (9.5) that, at constant pressure, the heat dQ absorbed by the system in
an infinitesimal reversible process is equal to dH. Therefore, for the process i-a, the heat absorbed
by the system is Q(i-a), given by

Q(i-a) = dH(i-a) = f Ta
n Cp.m dT
T
fa dT
Tj

= n Cp,m
Tj

(since cp,m is a constant)

where n is the number of moles.


Further, in an infinitesimal reversible process, the change in the entropy of the system dS is given
by

dS = dQ/T

so that, for the process i-a, the change in the entropy of the system is dS(i-a), given by

dS(i-a) = f Ta

Tj
n (Cp,m/T) dT = n Cp,m ln(Ta/Ti)
·

For the process a-f the temperature is constant and equal to Ta, which equals Tf. Therefore,
there is no change in the internal energy of the system as it is an ideal gas (Joule's law), and the
entropy form of the first law becomes

Ta dS = pdV

or,

dS = (piTa) dV

Further, since the temperature of the system remains constant during the process a-f, Boyle's law
holds and

pV = constant = K (say)

Then,

Now

H = U+ pV

135
so that

dH = dU + d(pV)
For an ideal gas undergoing a reversible process at constant temperature, U is a constant (Joule's
law) and (pV) is a constant (Boyle's law). Therefore,

dH= 0

and, in the finite process (a-f),

ilH(a-f) =0
The total change in the enthalpy of the system for the process i-f is then given by

and the change in the entropy for the same process is given by

The value of K needs to be determined and this can be obtained from the equation of state. For an
ideal gas

pV = nRT
where R is the molar gas constant. Taking the values for state a gives

1.1 X 105 (Pa) X Va = 2 (mol) X 8.315 (J K- 1 mol- 1 ) X 700 (K)

so that

Va = (2 X 8.315 X 700)/(1.1 X 105 ) m3

and

K = PaVa = (1.1 X 105 X 2 X 8.315 X 700)/(1.1 X 105 )


= 11 641 J
Then,

ilH(i-f) = 2 (mol) x 20.79 (J K- 1 mol- 1) x (700 (K) - 300 (K))


= 16 632 J
and

ilS(i-f) = 2 (mol) x 20.79 (J K- 1 mol- 1) ln(700 (K)/300 (K))


+ [(11 641 (J)/700 (K)) X ln(l.l X 105 (Pa)/2.0 X 105 (Pa))]
= 35.23 + (- 9.94)
= 25.3 J K- 1

9.6 The pressure on a copper disc, of mass 0.01 kg and volume V, is increased isentropically by 5.0 x
108 Pa. Calculate the change in the temperature of the copper if the initial temperature is 300 K
and the cubic expansivity ~ of the material is given by

~ = a/V
where a is equal to 5.46 x 10- 11 m3 K- 1 • For copper the specific heat capacity at constant pressure
may be taken as a constant equal to 385 J kg- 1 K- 1 •

136
Solution The system is the copper disc of mass m and volume V. Since the process considered is isentropic, it
is convenient to derive an expression for the change in the entropy of the system for a process that
involves a change in the pressure and in the temperature and then equate this entropy change to
zero.
The entropy S of the system may be considered as a function of temperature T and pressure p for
a closed hydrostatic system such as the copper disc. Then,

S = S(T,p)

and

dS = (oS/oT)p dT + (oS/op)T dp
In an isentropic process dS is zero so that

(as;ar)p dT = - (as;ap)r dp (S constant)

Now,

and Maxwell relation (Ml) is

Substitution gives

(S constant)

But,

(av;ar)p = ~v

where ~is the cubic expansivity. Therefore,

(S constant)

Rearranging gives

(oT/op) 5 = ~VT/CP

For this system

~ = a/V

so that

(aT!ap) 5 = aTICP

If the initial temperature and pressure of the copper are Ti and Pi, respectively, while the final
values are Tt and Pt. respectively, separating the variables and integrating for the isentropic change
of pressure gives

JTf (1/T) dT = (a/Cp)


JPt dp
Ti Pi
or,

Therefore,

137
and

so that

Tt- Ti = Ti{exp[(a/Cp) (Pt- Pi)] - 1}


The mass of the disc is 0.01 kg so that its heat capacity is 0.01 x 385 J K- 1 • Then

Tt- Ti = 300 (K) X {exp[(5.46 X 10- 11 (m3 K- 1)/3.85 (J K- 1)) (5.0 X 108 (Pa))] - 1}
= 2.13 K

9.7 For an ideal elastic rod that has a length Lunder a tension Fat a temperature T, show that

where Cpis the heat capacity under constant tension and CL is the heat capacity at constant length.

Solution The principal heat capacities for a system consisting of an elastic rod of length L under a tension F
at a temperature Tare, by comparison with those for a closed hydrostatic system:

(a) the heat capacity at constant length (analogous to that at constant volume for the closed
hydrostatic system) given by

and
(b) the heat capacity at constant load (analogous to that at constant pressure) given by

A relationship between CL and CF may be obtained by considering the entropy S as a function ofT
and L.
Then, in an infinitesimal reversible process, the change in the entropy dS is given by

dS = (astaT)L dT + (astaL)r dL

Multiplying by T gives

T dS = T(aS/aT)L dT + T(aS/aL)r dL

Then, differentiating with respect to T, keeping F constant, gives

and, from the definitions of CF and CL,

The partial derivative (astaL)r may be replaced using a Maxwell relation for an elastic rod. The
expressions for the work done on the system in an infinitesimal reversible process are

dW = -pdV

for a closed hydrostatic system and

dW = FdL

138
for an elastic rod in tension. By analogy, Maxwell relations for an elastic rod may be obtained from
those for a closed hydrostatic system by substituting F for p and - L for V. Then, the Maxwell
relation (M3) becomes

Making this substitution gives

9.4 Equations of The simplest closed hydrostatic system is a fixed mass of a pure substance. For such a
State system the totality of equilibrium states is represented by a continuous surface in
pressure p-volume V-temperature T space, the equation of this surface being the
equation of state of the system. The p-V-T surface for a pure substance is shown
schematically in Fig. 9.2.

Figure 9.2 The p-V- T


surface for a pure substance.

In this diagram the full lines are isotherms, the dashed lines mark the phase boundaries
and the shaded regions indicate states where mixed phases coexist in equilibrium.
For temperatures above Tc, known as the critical temperature of the substance, the
substance exists in the gaseous phase whatever the value of the pressure. At tempera-
tures of Tc and below it is always possible to produce a condensed phase of the substance
if the pressure is sufficiently large. For this reason the gas phase is often termed 'vapour'
when T ~ Tc, the term gas being used when T > Tc.
When the gaseous phase at the temperature T, is compressed, condensation to the
solid and liquid phases occurs simultaneously. T1 is known as the triple point tempera-
ture. (In the p-T diagram, the three phases that coexist in equilibrium are represented by
a point.)
When condensation from the gaseous phase occurs at a temperature T in the range
T1 ~ T ~ Tc the condensation is to the liquid phase, while for temperatures less than T,
condensation of the vapour is directly to the solid phase.
The p-V-Tsurface for a pure substance is sufficiently complex to make its representa-
tion by a reasonably simple analytical equation of state very unlikely. For the gaseous
phase, when the pressure is low and the temperature is high, so that the gas may be
described as 'dilute', the behaviour approximates to that of an ideal gas and the equation
of state

139
gives a good approximation to the behaviour. Here V m is the molar volume and R is the
molar gas constant.
When the pressures are higher this equation needs modification or replacement. One
approach is to use a series expansion, with the behaviour of the fixed mass of gas
represented by the equation

pVm = RT + B(T)p + C(T)p 2 + D(T)p 3 + ...

where, for a given gas, B, C, D, ... are functions of temperature only and of rapidly
decreasing magnitude.
An alternative approach is to apply corrections to the ideal gas equation to allow for
the non-ideality of real gases. A straightforward approach, using ideas from the kinetic
theory of gases (see Chapter 13) leads to van der Waals' equation, which may be written
in the form

[p + (a/V:0)] (Vm- b) = RT

where a and b are constants characteristic of the gas. A large number of equations of this
nature have been proposed, many of them being highly empirical. Examples are:

(a) Berthelot's equation

[p +(a' ITV:O)](Vm- b') = RT

where a' and b' are constants.


(b) Beattie-Bridgeman equation

p = (RT/Vm)([l- (c/VmT 3)]{1 + (B0 /Vm)[l - (b/Vm)]}


- (A 0 1V:0)[1 - (a!V:O)])

where A 0 , B0 , a, b and c are constants.


(c) The equation of Clausius

p = [RTI(Vm- a")]- {c"![T(Vm + b"f]}

where a", b" and c" are constants.

It is sometimes convenient to work with what are known as reduced coordinates, that
is, coordinates measured using the values at the critical temperature as the units. For a
closed hydrostatic system the reduced coordinates are:

the reduced temperature t = T!Tc


the reduced pressure p =pipe
the reduced molar volume Vm = V m1Vc,m

where To Pc and Vc,m are, respectively, the critical temperature, the critical pressure and
the critical molar volume. Using these coordinates, van der Waals' equation becomes

[ft + (3/V:0)](3V m - 1) = st
which is independent of the quantities a and band should, therefore, hold for all van der
Waals gases.

140
9.5 Worked
Examples on
Equations of State
9.8 Derive expressions for the work done on a system consisting of n moles of molecules of a gas at a
pressure p and a temperature T, when its volume V changes from a value Vi to a value Vt by means
of a reversible isothermal process, for the following equations of state:

(a) pVm = RT
(b) p(Vm- b) = RT
(c) [p + (a!V~)JVm = RT
(d) [p + (a!V~)] (Vm- b) = RT

where a and b are constants, R is the molar gas constant and V m is the molar volume.

Solution When a system consisting of a fixed mass of gas undergoes a finite reversible process in which its
volume V changes from Vi to Vt. the work W done on the system by the surroundings is given by

W=-JVfpdV
Vi
where pis the pressure. To evaluate this integral the relationship between p and V must be known.
This relationship is obtained from the equation of state.
(a) When the equation of state is

then

so that

pV = nRT

Then
Vf
W= - J (nRTIV) dV
Vi
At constant temperature T0

(9.17)

(b) When the equation of state is

p (Vm- b)= RT

then

Jl = (RT)I(Vm- b)= (RT)I[(VIn)- b]


= (nRT)I(V- nb)

and, therefore,

W =- JVf [(nRT)I(V- nb)] dV


vi

At constant temperature T0

W = -nRT0 ln[(Vf- nb)/(Vi- nb)] (9.18]

(c) When the equation of state is

141
[p + (a IV;,)] V m = RT

then

p = (RTIVm) - (a IV;,)
= (nRTIV) - (n 2 a/V 2 )

Then, when the process takes place at constant temperature T0

W =- rt [(nRT0 /V) - (n 2 a/V 2 )] dV


vi
=- {nRT0 ln(Vt!Vi) - n 2 a [- (l!Vt) + (1/VJ]}
=- {nRT0 ln(Vt!Vi) + n a [(1/Vt) - (1/Vi)]}
2 (9.19)

(d) When the equation of state is

[p + (a!V;,)](Vm- b)= RT

then

p = (RT)I(Vm- b)- (aJV;,)


= (nRT)!(V- nb)- (an 2 /V 2 )]

and

W = - rt [(nRT)I(V- nb) - (an 2 /V 2 )] dV


vi
and, when the process takes place at constant temperature T0 ,

(9.20)

9.9 The equation of state proposed by Clausius for a system consisting of a fixed mass of a pure
substance, that has a molar volume V m at a temperature T and exerts a pressure p on its
surroundings is

P = [RT/Vm- a")]- [c"IT(Vm + b") 2 ]

where a", b" and c" are constants. Derive expressions for the critical constants for this system.

Solution The critical constants are the values of the pressure, volume and temperature at the critical point.
At the critical point for any closed hydrostatic system, such as a fixed mass of a pure substance,

(9.21)

since the critical point is the point of inflexion for the isotherm at the critical temperature Tc.
For the equation of Clausius

(apliJV)r = -(RT)I(Vm- a") 2 + (2c'')![T(Vm + b") 3 ]


and

Let the molar volume at the critical point be Vmc· Then, using conditions (9.21)

(9.22)

and

(9.23)

142
Dividing Equation (9.23) by Equation (9.22) gives

(Vme - a")/2 = (Vme + b")/3


so that

3Vme- 3a'' = 2Vme + 2b"


or

vme = 3a'' + 2b" (9.24)

Now, substituting for Vmc in Equation (9.22) gives

RTe/(3a" + 2b" - a"? = 2c" fTc (3a'' + 2b" + b") 3

giving

T~ = (8c")/[27(a" + b")R] (9.25)

Next substitute for V me and Tc in the equation of Clausius. For this it is convenient to write the
equation in the form

p~ = {(RTe)I(Vme- a")- c''I[Te(Vme + b"?W


= [(R2 T~)/(Vme- a") 2] - {(2RTc c")/[Te(Vmc- a")(Vme + b'Y]} + {c''2 /[T~(Vmc + b"t]}
Substitution gives

P~ = (c"R)/[216(a" + b") 3 ] (9.26)

9.10 Show that a point on the Boyle curve of a substance that obeys van der Waals' equation of state
must satisfy the equation

(a- bRT) V~- 2abVm + ab 2 = 0

where the symbols have their usual meanings.

Solution The system is 1 mol of molecules of a substance that obeys van der Waals' equation of state:

[p + (a/V~)] (Vm- b)= RT


PVm
L-----
where V m is the molar volume at a temperature T and pressure p, a and b are constants
------
~--~~,~----- ~
characteristic of the substance and R is the molar gas constant.
The Boyle curve is the locus of points on the isotherms in the graph of p V m against p for which

---'-..L__- :11
and is the curve shown by the dashed line in Fig. 9.3, i.e. it is the locus of the minima in the
isotherms.
Write van der Waals' equation in the form
p

Figure 9.3 The Boyle curve


in the graph of p V m against p. Then,

The equation of the Boyle curve is obtained by putting the right-hand side of this equation equal to
zero, that is,

RT{[li(Vm- b)]- Wm!(Vm- b?]} + aV~ =0


143
Therefore,

-a!V;',

giving

(a- bRT) V;',- 2abVm + ab 2 = 0

9.11 A certain closed hydrostatic system has a molar volume V m under a pressure p and at a tempera-
ture T and obeys the equation of state

pVm = RT + B(T)p

where B(T) is a function of temperature T only. Show that the molar heat capacity at constant
pressure cp,m is given by

where (Cp,m)o is the value of Cp,m asp tends to zero.

Solution It has been shown in Example 9.1 that, for a closed hydrostatic system, the dependence of cp,m on
pressure p at constant temperature is given by

When the equation of state of the system is

pVm = RT + B(T)p
or,

Vm = (RT/p) + B(T)
then

and

Therefore,

Integrating at constant temperature, so that T(d 2 BidT 2 ) is a constant, gives

where f(T) is the function of integration. In the limit, asp tends to zero,

which is the value of f(T). Therefore,

9.12 For a closed hydrostatic system the compressibility factor Z is defined by the equation

144
where V m is the molar volume of the system when it is at a temperature T and under a pressure p.
R is the molar gas constant. Derive expressions for the isothermal compressibility and cubic
expansivity of the system in terms of Z.

Solution Consider a closed hydrostatic system consisting of n moles of molecules. Then the isothermal
compressibility KT is defined by the equation

KT = - (1/V) (aV/ap)T

where Vis the volume occupied by the system: V = n Vm· Now,

Vm = RTZ!p

so that

V = nRTZ!p

and

(avtap)T = - (nRTZ!p 2 ) + (nRT!p) (aztap)T

Therefore,

KT =- (l!V) [- (nRTZ!p 2 ) + (nRT!p) ( aztap )T]


= (nRTZ/p 2 V) - [(nRT!pV) (aztap)T]

But,

Z = pVm!RT = pV/nRT

so that

KT = (lip) - (liZ) (aztap)r

For the same system the cubic expansivity ~ is defined by

~ = (l!V) (avtaT)p

Now

V = nRTZ!p
so that

(avtaT)p = (nRZ!p) + (nRT!p) (aztaT)p


Therefore,

~ = (1/V) [(nRZ/p) + (nRT!p) (aztaT)p]


= (liT) + (liZ) (aztaT)p

9.6 Exercises
9.1 Show that, for a closed hydrostatic system that occupies a 9.2 Show that, for a closed hydrostatic system that occupies a
volume V at a temperature T under a pressure p, the heat volume V at a temperature T under a pressure p, the heat
capacity at constant volume Cv is given by capacity at constant volume Cv, the isothermal compressibility
KT and the cubic expansivity ~ are related by the equations
Cv = - T(avtaT)s (aptiJT)v
(avtaT)s = - (Cv KT!Tf3)
where S is the entropy of the system.
(avtaT)s = - (avtaT)p (Cv KT!TV~ 2 )

145
9. 7 For a gas obeying van der Waals' equation of state, find an
expression for the difference between the molar heat capacity at
where CP is the heat capacity at constant pressure. constant pressure and that at constant volume.
Cp,m- Cv,m = R![l- 2a(Vm- b) 2 1RTV;,] ]
9.3 Show that, for a gas that obeys van der Waals' equation of [
state, the change in molar internal energy AUm that occurs when = {p + (a!V;..)}{R![p - (a!V;..)+ (2ab!V;,)]}
the molar volume changes from vmi to vmf and the temperature
changes from Ti to Tt, is given by 9.8 Show that van der Waals' equation may be expressed in the
series form

Z = 1 + [b- (a!RT)]!Vm + W!V;..) + W!V;,) + ...


where Cv, m is the molar heat capacity at constant volume and a
is a constant. where Z = pVm!RT.
Hint: write van der Waals' equation in the form
9.4 Determine the critical constants of a system that consists of
one mole of molecules of a gas that obeys Berthelot's equation of
state
and expand.
[p + (a' ITV;..)](Vm - b') = RT
9.9 For a closed hydrostatic system whose equation of state is
= 3b'; expressed in the series form
Wmc T~ =Sa' 127Rb'; Pc = (1!6b')(a'R/6b') 112 ]

9.5 The equation of state of a certain closed hydrostatic system


is where Z = pVm!RT, show that

where V m is the molar volume of the system when its tempera- B', C' and D' are functions of temperature only, V m is the molar
ture is T and its pressure p, and C is a constant. Derive an volume of the system at a temperature T and pressure p and R is
expression for the difference between the molar heat capacity at the molar gas constant.
constant pressure Cp,m and that at constant volume Cv,m· 2
[Cp,m - Cv,m = 9CT ]

9.6 Show that, for a gas that obeys van der Waals' equation of
state, the Boyle temperature is 27/8 times the critical tempera-
ture.

146
Topic Guide

10 Phase Changes
10.1 The Enthalpy of a Phase
Change (Latent Heat)
10.2 Worked Examples on the
Enthalpy of a Phase
Change
10.3 The Clapeyron-Clausius
Equation
10.4 Worked Examples on the
Clapeyron-C lausi us
Equation
10.5 The Equation of Clausius
10.6 Worked Example on the
Equation of Clausius
10.7 Exercises

10.1 The Enthalpy An examination of the pressure-volume-:-temperature surface for a pure substance (Fig.
of a Phase Change 9.2) shows that the transitions from the solid phase to the liquid phase and from the
liquid phase to the gaseous phase are characterised by
(Latent Heat)
(a) a change in the volume of the system;
(b) no change in the temperature of the system.

In addition, there is an absorption of heat by the system. Phase changes having these
characteristics are known as first-order phase changes. Since the heat absorbed by the
system during a phase change does not produce a change in the temperature of the
system it is often called the latent heat of the phase change. However, as will be seen
later, it is better described as the enthalpy of the phase change.
The latent heat is needed to supply the work done in the change in the volume of the
system against the pressure exerted by the surroundings and also the energy needed to
separate the constituent molecules of the system against their mutual attractions.
The specific latent heat of fusion (specific enthalpy of fusion) is the heat needed to
change unit mass of the solid phase to the liquid phase without change in temperature, at
a constant pressure. For a given substance, this quantity is not strongly dependent on the
pressure. If, on changing from solid to liquid at constant temperature under a given
pressure, a system of mass m absorbs a quantity of heat Q, the specific enthalpy of
fusion, ht, is given by

ht= Q!m (10.1)

The specific enthalpy of vaporisation hv is the heat needed to change unit mass of the
system from the liquid phase to the gaseous phase without change in temperature at
constant pressure. If, on changing from the liquid phase to the gaseous phase at constant
temperature under a given pressure, a system of mass m absorbs a quantity of heat Q, hv
is given by

hv = Q/m (10.2)

hv may be strongly dependent on the pressure.

147
10.2 Worked
Examples on the
Enthalpy of a Phase
Change 10.1 The temperature of liquid helium may be lowered by causing it to boil away under reduced
pressure by pumping away the vapour. The specific enthalpy of vaporisation of liquid helium may
be taken as a constant equal to 23 x 103 J kg - l and the specific heat capacity of the liquid is given
in the table.
Calculate the percentage of the liquid that must be boiled away in reducing the temperature of
an isolated mass of liquid helium from the normal boiling temperature of 4.2 K to 3.0 K.

4.5 4.0 3.5 3.0 2.5


4.9 3.8 3.0 2.5 2.2

Solution The system here is the total mass of helium, which is isolated from its surroundings. However,
since the process is the transfer of helium from the liquid phase to the gaseous phase, it is
advantageous to concentrate on the liquid phase which, of course, is not of constant mass.
At any stage in the evaporation process let the mass of liquid helium be m. To evaporate a mass
dm of the liquid in a subsequent infinitesimal (isothermal) reversible process requires an amount of
energy equal to hv dm, where hv is the specific enthalpy of vaporisation at the given pressure p and
temperature T. When the liquid is isolated this energy comes from the internal energy U of the
mass m of the liquid. The energy transfer from the liquid to the vapour in the infinitesimal process
is dQ, given by

Here cP is the specific heat capacity at constant pressure of the liquid at a temperature T.
Therefore, for the infinitesimal reversible process

or,

In a finite reversible process, in which the temperature of the liquid helium is changed from Ti to Tf
as the mass of liquid changes from mi to mf,

The left-hand side of the above equation is

while the right-hand side is the area under the cP-T graph, the sign depending on the limits. The
graph of cP against Tis shown in Fig. 10.1.
By counting squares (or any other suitable technique) the area under the curve has a value of
3.90 x 103 J. Since the limits go from 4.2 K to 3.0 K the sign is negative.
Therefore,

giving

ln(mf/mi) = - 0.1696
or,

mtlmi = exp(-0.1696)
= 0.844

148
3
I
I""
_,."'
~
..,o.
2

Figure 10.1 Grap~of :_p


agamst 1.

Therefore, the fraction of the liquid that is evaporated is

10.2 An inventor claims that his refrigeration device, operating in an ambient temperature of 300 K,
produces 24.0 kg of ice at its normal melting point (273 K) per hour from water at the same
temperature, with a power consumption of 100 W. Comment on this claim.
For water at 273 Kanda pressure of one atmosphere, ht = 3.335 x 105 J kg- 1 .

Solution The inventor's claim may be examined through the first and second laws of thermodynamics. In
particular, there should be an energy balance, as specified by the first law for a cyclic process, and
the device should not be more efficient than a reversible device operating between the same
temperature limits.
The system is the working substance of the refrigerator and interacts thermally with surround-
ings at a temperature Ts and with the mass of water that is being transformed into ice at a
temperature T0 • There is also a work interaction with the surroundings. Let the refrigerator
perform n cycles per hour and, in each cycle, absorb a quantity of heat Qs from the surroundings
and a quantity of heat Q0 from the water. Let the work done on the system in each cycle be W.
Then, applying the first law gives

nQs + nQ0 + n W = 0

since there is no change in the internal energy of the system in a complete cycle.
If the inventor's claim is valid,

where m is the mass of ice produced per hour. Then

This is the energy balance that must hold in the operation of the refrigerator. Now

nW = 100 (W) X 3600 (s hr- 1 ) = 3.6 x 105 J hr- 1

so that

149
giving

nQs = -8.36 x 106 J hr- 1

The refrigeration device cannot be more efficient than a reversible device performing the same
cycle of operations, i.e. than a Carnot refrigerator. For the inventor's device, the change in the
entropy of the working substance per cycle is zero, since the entropy is a state function. The change
per hour in the entropy of the surroundings !l.Ss is given by

assuming that the interactions with the surroundings and with the water may be treated as
reversible, being interactions with bodies that essentially behave as reservoirs. (Remember that
the sign convention applies to the working substance.)
Then,

!l.Ss = -((8.00 x 106 (J hr- 1 )/273 (K)) + (-8.36 X 106 (J hr- 1 )/300 (K))]
= -2.93 X 104 + 2.79 X 104 J hr- 1 K- 1
<0

This result violates the law of the increase of entropy and, therefore, the inventor's claim is invalid.
The extraction of heat of amount Q0 from the water being transformed to ice will involve the
rejection of a minimum amount of heat Qs(min) to the surroundings when the process is reversible.
This amount of heat may be obtained from the equation defining thermodynamic temperature

Therefore,

and the minimum amount of work W(min) that must be done on the system per cycle to produce
the transformation of water to ice is given by

W(min) + Q0 + Qs(min) = 0

or,

so that

W(min) = - Q0 [1 - (Ts/T0 )]

Therefore, the minimum amount of work that must be done on the system in one hour is

-(8.00 X 106 (J hr- 1)) (1- (300 (K)/273 (K))]

corresponding to a power of +791 210 J hr- 1 or 220 W.

10.3 A piece of ice of mass 2.0 kg at a temperature of 263 K and at atmospheric pressure is placed in
6.0 kg of liquid water at a temperature of 290 K at the same pressure. Calculate the final
temperature of the water substance.
Under a pressure of one atmosphere ice melts at a temperature of 273 K. The specific heat
capacity at constant pressure for ice is 2200 J kg- 1 K- 1 while that for water is 4200 J kg- 1 K- 1 .
The specific enthalpy of fusion of ice at a pressure of one atmosphere is 334 x 103 J kg- 1 •

Solution The system is the mass of ice and water and may be assumed to have no thermal interaction with

150
the surroundings and negligible work interaction arising from the change in volume of the system
as a result of the ice melting. When the ice and water are first brought into thermal contact there
will be a heat transfer from the water to the ice. This will cause the temperature of the ice to rise
and that of the water to fall. If the heat transfer is sufficient, the temperature of the ice will rise to
its melting point and melting will start. If not, the water will be turned into ice. If the heat transfer
is sufficient, all the ice will melt and the melted ice and the original water will reach a final common
equilibrium temperature.
-Consider first the question of whether all the ice will melt. The heat that must be absorbed by the
ice to raise its temperature from 263 K to 273 K without melting is Q1 , given by

where m is the mass of ice, cP is the specific heat capacity of ice at constant pressure and ~Tis the
rise in temperature. Then,

Q 1 = 2.0 (kg) X 2200 (J kg- 1 K- 1) X (273 (K)- 263 (K))


=440001

The heat absorbed by the original water as its temperature falls from 290 K to 273 K is Q2 , given
by

Q2 = m(water) X cp(water) X ~T(water)

= 6.0 (kg) X 4200 (J kg- 1 K- 1 ) X (273 (K)- 290 (K))


=- 428 400 J

This amount of energy is more than enough to raise the temperature of all the ice initially present
to the melting point.
To melt a mass m of ice at constant temperature requires the absorption of an amount of heat
Q3 , given by

where ht is the specific enthalpy of fusion.

Q3 = 2.0 (kg) X 334 X 103 (J kg- 1)


= 668 000 J

Therefore, the total amount of heat that the ice must absorb for it just to melt completely is

Q1 + Q3 = 44 000 + 668 000 = 712 000 J


Since the magnitude of Q1 + Q3 is larger than that of Q2 , the water is unable to supply enough
energy to melt the ice completely. To raise the temperature of the ice requires 44 kJ so that
384.4 kJ of energy are available to melt the ice. Therefore, the mass of ice melted M will be given
by

384.4 (kJ) = M X ht
= M X 334 (kJ kg- 1)

or,

M = 384.4/334 = 1.15 kg

10.4 Calculate the minimum amount of work that must be supplied to a refrigerator to convert 1.00 kg
of water, initially at a temperature of 293 K and under a pressure of 1.1 x 105 Pa, into ice at a
temperature of 273 K at the same pressure. The temperature of the surroundings is 293 K. The
specific heat capacity at constant pressure for water may be taken as a constant equal to
4200 J kg - 1 K - 1 ; the specific enthalpy of fusion of ice at a temperature of 373 K and under a
pressure of 1.1 X 105 Pa is 34 x 104 J kg - 1 •

151
Solution The refrigerator will require the minimum amount of work done on its working substance when all
operations are carried out reversibly, that is, when it is a Carnot refrigerator. Take the working
substance of the Carnot refrigerator as the system. It will interact thermally with both the mass of
water and with the surroundings and there will also be a work interaction with the surroundings.
Because the mass of the water is finite, extracting heat from the water will produce a fall in
temperature. Therefore, if the refrigeration cycle is to be carried out reversibly, only an infini-
tesimal amount of heat must be extracted from the water in each cycle.
The production of ice from the water may be considered in two stages:

(a) the reduction of the temperature of the water from 293 K to 273 K;
(b) the conversion of the water at a temperature of 273 K into ice at the same temperature.

In any infinitesimal cyclic process the absorption of heat takes place at effectively constant
temperature so that, in the infinitesimal reversible cyclic process considered, the heat absorbed in
each cycle from the water dQ, and that absorbed from the surroundings dQ 0 are related by the
defining equation for thermodynamic temperature:

dQfdQ0 = - TfT0

where Tis the temperature of the water and T0 is that of the surroundings. Then,

dQ 0 = - ( T0 fT) dQ
(a) Consider first the reduction of the temperature of the water from 293 K to 273 K. The value
of dQ is given by

dQ = - mcP dT

where m is the mass of the water, cP is its specific heat capacity at constant pressure and d Tis the
change in the temperature that results from the extraction of the heat from the water, even though,
in the discussion of this extraction as part of a cyclic process, it was considered to take place at
effectively constant temperature. The negative sign appears because the sign convention applies to
the system and not to the surroundings.
In one complete infinitesimal cycle of operation there is no change in the internal energy of the
system, so that the first law gives

dQ + dQ0 + dW = 0
where dW is the work done on the system in each cycle. Therefore,

- mcP dT + [- (T0fT) dQ] + dW =0


or

- mcP dT + mcP (T0fT) dT + dW = 0


and

dW = mcP dT- (T0fT) mcP dT

In a finite process, composed of an infinite number of elementary cycles of the Carnot refrigerator,
in which the temperature of the water changes from Ti to Tr, the work done on the working
substance is W, given by

W = mcP (Tr- Ti) - T0 mcP ln(TrfTi)


= mcP [Tr- Ti - T0 ln(TtfTi)]
= 1.00 (kg) X 4200 (J kg- 1 K- 1 ) X [273 (K)- 293 (K)- 293 (K) x ln(273 (K)f293 (K))]
= 3004 J
(b) The phase change takes place at constant temperature, so that it is not necessary to treat the
behaviour of the Carnot refrigerator in detail. Let the working substance complete a whole

152
number of cycles during which the water at a temperature of 273 K is converted to ice at the same
temperature. Then, during this process there is no change in the internal energy of the working
substance and the first law may be written

Q + Q0 + W =0
where Q is the heat absorbed by the working substance from the water and Q0 is that absorbed
from the surroundings. Q and Q 0 must satisfy the following conditions:

Q! Q0 = - T(ice)/ T0

where T(ice) is the temperature of the ice, and

Q = mht
Therefore,

so that,

W = - (mht - mht T0 /T(ice))


= - mhf (1 - T0 /T(ice))
= - 1.00 (kg) X 340 000 (J kg- 1) X (1 - 293 (K)/273 (K))
= 2.49 X 10 J
4

The total amount of work that must be done on the system to reduce the temperature of the water
and then convert it into ice is, therefore,

0.3004 X 104 + 2.49 X 104 = 2. 79 X 104 J

10.3 The A more general approach to the description of phase changes may be made as follows.
Clapeyron-Clausius Consider a closed hydrostatic system consisting of a pure substance (a one-component
system) in a state in which two phases coexist in equilibrium. If surface effects across the
Equation phase boundary are negligible, the temperature T and pressure p are both uniform
throughout the system. In the p-T diagram (Fig. 10.2) this equilibrium state is rep-
resented by a point on the appropriate phase boundary, such as the point marked a in
Fig. 10.2. For example, if the two phases are the liquid and solid phases, the equilibrium
state is represented by a point on the fusion curve.

Phase 1

Phase 2
Figure 10.2 A phase
boundary, showing states in
which two phases coexist in
equilibrium. T

Let the phases be designated 1 and 2, and let an infinitesimal mass of substance pass
from phase 1 to phase 2 by a reversible process, which must take place at constant
temperature and pressure. Since the change dG in the Gibbs function for the complete
system (consisting of both phases) is given by

dG = Vdp- SdT

153
there can be no change in the Gibbs function of the system as it undergoes a reversible
phase change. Here Vis the volume of the system and Sis its entropy. From this it is
readily shown that the condition for the two phases to coexist in equilibrium is

where g1 and g 2 are, respectively, the specific Gibbs functions for phases 1 and 2.
Consider now how the temperature at which two phases coexist in equilibrium de-
pends on pressure. When the system is in the equilibrium state represented by point a in
Fig. 10.2 the equilibrium condition is

If now the pressure is increased by the amount dp and the two phases continue to coexist
in equilibrium, the temperature must increase by the amount dT to ensure that the new
equilibrium state, represented by the point b in Fig. 10.2, remains on the appropriate
phase boundary. It may be shown that the dependence of temperature on pressure is
given by

dpldT = (h 2 - h 1 )1[T(v 2 - v1 )]
= h 12 1[T(v2 - vl) (10.3)

where v is the specific volume and h is the specific enthalpy of the appropriate phase and
h 12 is the specific enthalpy of the phase change for phases 1 and 2.
Equation (10.3) gives the equation of the phase boundary in differential form and is
known as the Clapeyron-Clausius equation.

10.4 Worked
Examples on the
Clapeyron-Clausius
Equation to.s Derive the form of the approximate Clapeyron-Clausius equation appropriate to sublimation and
vaporisation processes at low vapour pressures.

Solution Take as the system one mole of molecules of any pure substance. For such a system the Clapeyron-
Clausius equation may be written

where Hm is the molar enthalpy of the phase change from phase 1 to phase 2, V ml is the molar
volume of phase 1, V m 2 is that for phase 2 and T is the temperature at which the phase change
occurs when the pressure is p.
When the pressure is low and all the substance is in the gaseous (vapour) phase, the mass of
vapour remains constant as the volume is varied and the pressure exerted by the vapour is given
approximately by the ideal gas equation. Then, to a good approximation,

pVm2 = RT

or,

Vmz = RT!p

Assume also that V m 2 is much greater than V ml· Then, neglecting V ml in comparison with V ml• and
substituting for V mz• the Clapeyron-Clausius equation becomes

154
Separating the variables gives

and integrating, assuming that Hm is a constant, gives

In p =- Hml RT + constant

When T has the value T0 , let p have the value p 0 • Then,

In Po = - Hml RT0 + constant

and

In(plp 0 ) = - (Hm!R)[(l!T)- (l/T0)]

10.6 For ammonia, the equation of the sublimation curve is

ln(p/Pa) = 27.92 - 3754/( TIK)

while the equation of the vaporisation curve is

ln(p/Pa) = 24.38 - 3063/(T/K)

Calculate the temperature of the triple point of ammonia and the molar enthalpies of sublimation,
vaporisation and fusion at the triple point.

Solution Consider a system consisting of one mole of molecules of ammonia. The triple point is the point of
intersection in the pressure p-temperature T graph of the sublimation, vaporisation and fusion
curves. Therefore, on the sublimation curve, the triple point is given by

In p 3 = 27.92- 3754/T3

and, using the vaporisation curve, by

In p 3 = 24.38 - 3063/ T3

where p 3 and T3 are, respectively, the pressure and temperature at the triple point.
Eliminating p 3 gives

27.92- 3754 (K)IT3 = 24.38- 3063 (K)'T3

so that

T3 = 195.2 K

The values of the molar enthalpy of sublimation Hsm and the molar enthalpy of vaporisation Hvm
may be obtained using the approximate form of the Clapeyron-Clausius equation obtained in
Example 10.5, that is,

where Hm is the enthalpy of the appropriate phase change. For the sublimation process

lnp = 27.92- 3754/T


so that

(lip) (dpldT) = 3754/T2

155
or,

(dp/dT) = 3754 (p/T 2 )

Therefore, using the approximate Clapeyron-Clausius equation,

or,

Hsm = 3754 X R
= 3754 (K) X 8.315 (J K-1 mol- 1 )

= 31.2 kJ mol- 1

Similarly, for the vaporisation process,

Hvm = 25.5 kJ mol-1


The molar enthalpy of fusion at the triple point may be obtained by considering the change from
the solid phase to the liquid phase to occur in two stages. When 1 mol of ammonia sublimes from
the solid phase directly to the vapour phase the energy that must be supplied to the system at the
conditions of the triple point is 31.2 kJ mol- 1 . When this vapour condenses to the liquid phase
under the same conditions the energy transfer to the surroundings is 25.5 kJ mol- 1 • Since the
enthalpy is a state function, the change in the enthalpy needed to produce the change from the
solid phase to the liquid phase is independent of the way the change is brought about. Therefore,

Htm = 31.2 (kJ mol-1 ) - 25.5 (kJ mol-1 )


= 5.7 kJ mol- 1

where Htm is the molar enthalpy of fusion of ammonia at the triple point.

10.7 The liquid-vapour equilibrium curve (vaporisation curve) for carbon tetrachloride over the
temperature range 273 K to 313 K is given in the table below, where p is the saturation vapour
pressure at a temperature T.

TIK 273 283 293 303 313


PI X 104 Pa 0.439 89 0.746 48 1.2130 1.8662 2.8660

Estimate the molar enthalpy of vaporisation of carbon tetrachloride at 293 K.

Solution Take as the system one mole of molecules of carbon tetrachloride. Let the system be initially in the
liquid phase at a temperature of 293 K and then undergo a reversible process in which it changes to
the vapour phase at the same temperature. Assume that the approximate integral form of the
Clapeyron-Clausius equation holds for the conversion of the system from the liquid phase to the
vapour phase. Then, the dependence of the saturation vapour pressure p on temperature Tis given
by

In p = In A - Hvml RT

where A is a constant, Hvm is the molar enthalpy of vaporisation and R is the molar gas constant. A
graph of In p against (liT) should, therefore, be a straight line of slope- (Hvm/R) and intercept In A.
This graph is shown in Fig. 10.3. The graph is a straight line with a slope of -3940 K. The shape of the
graph indicates that the equation is valid over the temperature range considered and that

slope of graph = - 3940 K = - (Hvm!R)

or,

Hvm =R X 3940 (K)


= 8.315 (J K- 1 mol-1 ) X 3940 (K)
= 33 kJ mol- 1

156
In p
10.5

10.0

9.5

9.0

8.5

Figure 10.3 The graph of


In p against liT for the
liquid-vapour equilibrium of
carbon tetrachloride. 3.1 3.3 3.5

10.8 Using the following information, determine the specific enthalpy of vaporisation of water at a
temperature of 373.15 K.

TIK 368.15 373.15 383.15


p!M Pa 0.084 53 0.101 35 0.1433
v1 /X 10- 3 m3 kg- 1 1.0435
v)x 10- 3 m 3 kg- 1 1672.9

Here p is the saturation vapour pressure at a temperature T, v 1 is the specific volume of the liquid
phase and vv is the specific volume of the vapour phase.

Solution Take as the system one kilogram of water. The specific enthalpy of vaporisation hv is given by the
Clapeyron-Clausius equation

dp/dT = hvf[T(vv - Vt)]

where vv is the specific volume of the vapour and v1 is that of the liquid when the transition takes
place at a temperature T and a pressure p. To use this equation dpldT must be obtained. This can
be done by plotting a graph of the saturation vapour pressure p against T and measuring the slope
when T = 373.15 K.
The graph is shown as Fig. 10.4.
When T = 373.15 K the slope is 0.0035 MPa K- 1 • Therefore,

hv = (dpldT) X T X (vv- v1)


= 0.0035 (MPa K-1) x 373.15 (K)
X [1627.9 X 10-3 (m 3 kg- 1 ) - 1.0435 X 10-\m3 kg- 1 )]
= 2184 kJ kg-!

10.5 The Equation In any phase change in a closed hydrostatic system consisting of a pure substance, let a
of Clausius mass m change from phase 1 to phase 2 by a reversible process at the appropriate
temperature T and pressure p. The specific enthalpy of the process h12 is given by

157
p(MPa)
0.15

0.14

0.13
I
I
'/
0.12 y

0.11

0.10

0.09

Figure 10.4 The graph of p


0.08
against T for the equilibrium
of liquid and gaseous water. 360 370 380 390 TIK

where h 1 and h 2 are the specific enthalpies of the system in phases 1 and 2, respectively,
and s 1 and s2 are the corresponding specific entropies. When the pressure on the system
changes, the temperature of the system must also change to keep the system on the
appropriate phase boundary (equilibrium curve), as shown in Fig. 10.2. This condition
removes one degree of freedom of the system so that h and s may be treated as functions
of one coordinate only; for example, they may be treated as functions of temperature
only, subject to the condition that the two phases coexist in equilibrium. The dependence
of h 12 on T may be found by differentiating the above equation with respect to T. Then,

d(h 21T)/dT - d(h/T)/dT = dszfdT- ds 1/dT

or,

(liT)(dhzfdT) - (h 2 /T 2 ) - (1/T)(dh/dT) + (h/T 2 )


= (ds2 /dT) - (ds 1/dT)

Therefore,

or,

(10.4)

Equation (10.4) is the equation of Clausius. The quantities c1 and c2 may be treated as
specific heat capacities, subject to the condition that the two phases remain in mutual
equilibrium as the temperature of the system changes. For example, c1 might be the
specific heat capacity of the saturated liquid and c2 that of the saturated vapour.

10.6 Worked
Example on the
Equation of Clausius
10.9 The specific enthalpy of vaporisation hv of water varies with temperature T as shown. Calculate the
specific heat capacity of saturated steam at a temperature of 373 K if the specific heat capacity of
saturated water at that temperature is 4229 J kg-1 K - 1 .

h)kJ kg- 1 2283 2270 2257 2244 2230


TIK 363 368 373 378 383

158
Solution Take as the system one kilogram of saturated water at a temperature of 373 K. The process that
this system undergoes is the conversion of this saturated water to saturated vapour (steam) without
change in temperature. If this process is assumed to be reversible, the specific heat capacity of the
saturated steam c(vap) is related to that of the saturated liquid c(liq) by the equation of Clausius:

dhvldT- (hv!T) = c(vap) - c(liq)

where hv is the specific enthalpy of vaporisation at a temperature T and a pressure p. To use this
equation the value of dhjdT must be obtained. This quantity is the slope of the graph of hv against
T, shown in Fig. 10.5. From this graph the value of dhjdT is - 2.6 kJ kg- 1 K- 1 • Therefore,
substituting the values in the above equation,

- 2.6 X 103 (J kg-1 K-1 ) - (2257 X 103 (J kg- 1 )/373 (K)) = c(vap) - 4229 (J kg- 1 K- 1 )

giving

c(vap) = - 4.42 x 103 J kg-1 K-1

hv (kJ kg- 1 )
2290

2280

2270

2260

2250

2240
Figure 10.5 The graph of hv
against T for water. 360 370 380 390 T/K

The specific heat capacity of saturated steam at a temperature of 373 K is negative: heat must be
extracted from the system as the temperature is raised under conditions in which the vapour
remains saturated.

10.7 Exercises
10.1 Calculate the minimum amount of work that is needed to change of the boiling temperature with altitude near sea level,
produce 15.0 kg of ice at a temperature of 273.15 K from water assuming that the air temperature is 300 K. [3.1 K km-1)
at the same temperature, both at a pressure of one atmosphere,
when the temperature of the surroundings is 298.0 K. At a
temperature of 273.15 Kanda pressure of one atmosphere the 10.4 Estimate the change in the melting temperature of ice,
initially at s.t.p., when the pressure is increased by one atmos-
specific enthalpy of fusion for ice is 338.7 J kg- 1 • [462 kJ) phere (1.01 X 105 Pa). The density of water at s.t.p. is 1.00 x 103
kg m-3 , that of ice at s.t.p. is 917 kg m-3 and the specific en-
10.2 The saturation vapour pressure p of a certain liquid at a thalpy of fusion of ice at s. t. p. is 3.335 x 105 J kg-1 •
temperature T is given by
[0.007 K)
p =A+ (BIT)
10.5 A Dewar vessel of negligible heat capacity contains liquid
where A and B are constants. Show that, for the process of nitrogen at its normal boiling point of 77.4 K. The pressure over
vaporisation, the change in the molar entropy Sm of the material the nitrogen is then reduced by pumping the vapour so that the
is given by liquid boils. Estimate the fraction of the liquid that must be
boiled away before solid nitrogen starts to appear.
For liquid nitrogen, specific enthalpy of vaporisation = 199 kJ
kg-1 , specific beat capacity at approximately constant pressure =
where A Vm is the change in volume of one mole of the substance 2.0 kJ kg-1 K-I, and freezing temperature = 63.3 K. (0.13)
for the phase transition from liquid to vapour.
10.6 A piece of lead of mass 11.4 x 10-2 kg and at a tempera-
10.3 The specific enthalpy of vaporisation of water at a press- ture of 289 K is placed in liquid nitrogen at a temperature of
ure of one atmosphere is 2.44 MJ kg-1 and the vapour density at 77.4 K under a pressure of one atmosphere. When equilibrium
the normal boiling point is 0.598 kg m-3 • Calculate the rate of bas been established it is found that 14.2 X 10-3 kg of liquid bas

159
boiled away. Calculate the mean specific heat capacity of lead Determine the molar enthalpy of vaporisation of carbon tetra-
over this temperature range. chloride in this temperature range using a suitable graph.
For liquid nitrogen the specific enthalpy of vaporisation is [33 kJ mol-1]
199 kJ kg- 1 • [117 J kg- 1 K- 1 ]
10.8 A certain substance sublimes at temperatures below
10.7 The saturation vapour pressure p of carbon tetrachloride 650 K. At temperatures between 550 K and 650 K the equation
varies with temperature T as shown. of the sublimation curve is

ln(p (mm Hg)) = - 1560 (T/K) + 207


TIK 273 283 293 303 313
p!Pa 4402 7470 12 139 18 676 28 681 where p is the vapour pressure of the solid at a temperature T.
Estimate the mean molar enthalpy of sublimation over this
temperature range. [130 kJ mol-1]

160
Topic Guide

11 The Third Law of


11.1 The Third Law of
Thermodynamics
11.2 Worked Examples on the
Third Law
11.3 Planck's and Simon's
Versions of the Third Law
Thermodynamics
11.4 Worked Example on Absolute
Entropy
11.5 Allotropic Transformations
11.6 Worked Example on Allotropic
Transformations
11.7 Exercises

11.1 The Third Law This law is concerned with the behaviour of systems as the temperature approaches
of Thermodynamics absolute zero. The third law of thermodynamics was proposed by Nernst as his new heat
theorem in 1906, the earliest statement of the law being that, for any closed hydrostatic
system,

lim
T--+ 0
d(~F)/dT = lim
T--+ 0
d(~U)/dT = 0 (11.1)

where ~F is the change in the Helmholtz function and ~ U the change in the internal
energy of the system in a finite process between two equilibrium states, and T is the
temperature of the system.
Since

F = U- TS

where S is the entropy of the system, the change in F that occurs in a process at constant
temperature is given by

~F = ~U- T~S

Therefore,

d(~F)/dT = d(~U)/dT - T d(~S)IdT- ~S

Nernst's postulate may then be put in the form

lim
T-+ 0
~s = o (11.2)

that is, the entropy change associated with any reversible process of a system approaches
zero as the temperature approaches absolute zero.

11.2 Worked
Examples on the
Third Law 11.1 Show that, for a closed hydrostatic system, the heat capacity at constant volume Cv satisfies the
condition

Cv~ 0 as T~ 0
or,

lim Cv
T-+ 0
=0

161
Solution Consider a closed hydrostatic system. The simplest approach to this problem is to start with the
expression for the heat capacity that involves the entropy of the system. When such a system
undergoes an infinitesimal reversible process at constant volume, the heat absorbed by the system
is given by Cv dT, where dT is the change in the temperature of the system, and the change in the
entropy dS is given by

dS = (CviT) dT

The expression for Cv may be written

Cv = T (as;aT)v = [as!a(InT)]v

As T approaches zero the value of lnT approaches minus infinity and, from the third law, dS tends
to zero. Therefore, the partial derivative tends to zero so that

lim Cv = 0
T~o

11.2 Show that, for a closed hydrostatic system,

where KT is the isothermal compressibility and Tis the temperature.

Solution The isothermal compressibility KT of a closed hydrostatic system is defined by the equation

where Vis the volume of the system when the pressure is p. Then,

The partial derivative (i:Jp!a'v)T cannot be transformed using a Maxwell relation and so the third
law cannot predict the value of KT. However, it is possible to examine the temperature dependence
of Kr- From the expression for KT

[a(liKT)!aTJv = - v {a[(ap/aV)T]IaT}v
= - v {a[ap!CJT)v]laV}r

Now,

(M3)

so that

According to the third law, the change in the entropy of a system in a reversible isothermal
process tends to zero as the temperature tends to zero, that is,

(aS!aV)T ~ o as T ~ o
Therefore,

As the absolute zero of temperature is approached, the value of the isothermal compressibility of a
closed hydrostatic system becomes independent of temperature.

11.3 4He remains liquid under its own vapour pressure down to the lowest temperatures and a
considerable pressure has to be applied to produce solid helium. Show that the slope of the fusion

162
curve (in the pressure-temperature graph) for 4 He must approach zero as the temperature
approaches absolute zero.

Solution For a system of one mole of 4 He the slope of the fusion curve is given by the Clapeyron-Clausius
equation

where Hrnf is the molar enthalpy of fusion, Vrni is the molar volume of the liquid phase and V ms is
that for the solid phase. The phase change takes place at a temperature Tunder a pressure p. For a
reversible phase change

where L1Srnf is the change in the molar entropy resulting from the phase change. Therefore, the
Clapeyron-Clausius equation may be written

where L1 V rnt = V rn1 - V rns· From the third law

so that

dpldT~ 0 as T~ 0

Therefore, the slope of the fusion curve of 4 He tends to zero as the temperature at which the phase
change occurs approaches absolute zero. Also, in the limit as T tends to absolute zero the value of
T L1Smt ( = Hmt) tends to zero so that, in the limit, melting of the solid may be achieved by an
almost adiabatic expansion.
Experiment shows that the fusion curve for 4 He (Fig. 11.1) has almost zero slope at all
temperatures below about 1 K.

p
Fusion curve

Solid

25atm. - - - -
Liquid

Figure 11.1 The fusion curve


for 4He at low temperatures. 0 T

11.4 Show that it follows from the third law of thermodynamics that the absolute zero of temperature is
unattainable.

Solution Any system that is to have its temperature lowered as much as possible must undergo a sequence of
consecutive reversible isothermal and reversible adiabatic processes. Two such processes are
shown on the entropy S against temperature T diagram shown in Fig. 11.2.
It is essential that S depends on another parameter in addition to the temperature. For a closed
hydrostatic system the entropy could depend on pressure p. Figure 11.2 shows how the entropy of
the system varies with temperature when the value of the pressure is maintained constant at Ihe
values p 1 and p 2 , respectively. The system starts in the equilibrium state in which the temperature
is Ti and the value of pis PH so that the value of Sis Si. Then Sis reduced reversibly at constant
temperature by changing the value of p to p 2 • The value of S falls to the value Sf. This process will
require the rejection of heat to the surroundings. The system is then thermally isolated and the

163
s

Figure 11.2 The entropy-


temperature diagram for a
closed hydrostatic system. T, T, T

p,
s

Figure 11.3 The entropy-


temperature diagram for a
closed hydrostatic system close
to absolute zero when the
third law is obeyed. 0 T

value of p is restored reversibly to PI, a process that takes place at constant entropy Sf and is
accompanied by a fall in the temperature of the system to Tf.
According to the third law, the entropy curves for the equilibrium states of the system corre-
sponding to PI and p 2 run together as absolute zero is approached, as shown in Fig. 11.3.
A number of consecutive processes are indicated and it can be seen that as the temperature
becomes lower the reduction in temperature produced by the isentropic process becomes less.
Therefore, absolute zero is unattainable, except by the performance of an infinite number of
consecutive reversible isothermal and isentropic processes.

11.3 Planck's and In 1911 Max Planck extended the Nernst heat theorem by postulating that, as the
Simon's Versions of temperature of a system approaches absolute zero, the absolute entropy of a simple
substance tends to zero, that is,
the Third Law
lim S = 0
r~o
(11.3)

With this postulate, the vanishing of the specific heat capacities at the absolute zero
allows absolute zero to be used as a reference level for thermodynamical calculations.
The entropy S 1 of a system at a temperature TI is given by

51 - So= r 0
1
(CIT) dT

where 5 0 is the entropy at absolute zero and C is the heat capacity appropriate to the
process linking the initial and final equilibrium states at temperatures zero and T1 ,
respectively. Since C tends to zero as T tends to zero, the integral is finite and, by putting
S0 to zero for all simple substances in thermodynamic equilibrium, a unique value is
allotted to S1 :

S1 = r0
1
(CIT) dT

The third law was further refined by F. E. Simon who noted that, for some systems,
different parts of the system are effectively independent of each other when the total

164
entropy of the system is calculated. For example, in a paramagnetic crystal the magnetic
spins and the thermal motions of the lattice make essentially independent contributions
to the entropy. Parts of a system, or the processes taking place within them, that
contribute independently to the entropy of the system are called aspects of the system.
Simon's formulation of the third law then states that:

the contribution to the entropy of a system by each aspect of that system that is in
thermodynamic equilibrium tends to zero as the temperature tends to the absolute
zero.

11.4 Worked
Example on
Absolute Entropy u.s The absolute molar entropy of dilute methane gas at a temperature of 298.2 K is 183.3 J K - l mol- 1 .
Calculate the absolute molar entropy of dilute methane at 450.0 K, assuming that the molar heat
capacity at constant pressure cp,m is given by

Cp, m (J K - l mol- 1) = 23.28 + 47.13 X 10- 3 (T (K)) - 1.90 x 105 (r- 2 (K - 2 ))

Solution The system is one mole of molecules of methane gas that is sufficiently dilute for it to be treated as
an ideal gas so that Cp,m is independent of pressure. To change the temperature of the gas from
298.2 K to 450.0 K, keeping the pressure constant, requires the absorption of heat by the system
and results in a change in the entropy of the system. This change in the entropy depends only on
the initial and final equilibrium states of the system as the entropy is a state function. However, to
calculate the change in the entropy the real, irreversible process must be replaced by a notional
reversible process linking the initial and final equilibrium states. In an infinitesimal part of this
notional reversible process let the change in the entropy of the system be dS and the temperature
of the system be T. Then, dS is given by

dS = (Cp, miT) dT

Since the absolute molar entropy is known at 298.2 K, the equation must be integrated over the
temperature range 298.2 K to 450.0 K to obtain the entropy at 450.0 K. Then, denoting the initial
and final equilibrium states by i and f, respectively,

Sf- Si = f (Cp, miT) dT


Tf

=roo [(23.28 (J K-
Tj

1 mol- 1 )1T) + (47.13 x 10- 3 (J K- 1 mol- 1 )(TIT))


298.2

- (1.90 X w-s (J K-l mol- 1 )IT 3 )] dT


= 16.44 J K - l mol- 1
But Si = 183.3 J K - l mol- 1 so that

Sf = 16.44 + 183.3 = 199.7 J K - l mol- 1

11.5 Allotropic Some substances may exist in different equilibrium solid forms under different conditions
Transformations of temperature and pressure. For example, at a pressure of one atmosphere sulphur
crystallises in the rhombic system at temperatures below 368 K and in the monoclinic
system over the temperature range 368 K to 393 K, at which latter temperature melting
occurs. A solid phase transition from one stable phase to another as a result of changes in
temperature or pressure is called an allotropic transformation.
When the transformation from a high-temperature solid phase to a low-temperature
solid phase takes place slowly there is a complete transformation of the material from the
high-temperature phase to the low-temperature phase, but, when the cooling through
the transition temperature is rapid, the high-temperature phase may be retained as a
well-defined metastable phase. It is then possible to produce two specimens at absolute

165
zero: a slowly-cooled one, containing only the low-temperature phase, and a rapidly-
cooled one, containing the high-temperature phase in a metastable equilibrium.
According to the third law, both stable and metastable phases must have the same
value of the entropy at the absolute zero, equal to zero on Planck's hypothesis. There-
fore, the entropy of the high-temperature phase at the transition temperature may be
calculated in two different ways which must give the same result because the entropy is a
state function. These ways are:

(a) by integrating CP!T for the metastable allotrope from absolute zero up to the
transition temperature, and
(b) by integrating CP!T for the stable allotrope (low-temperature phase) from absolute
zero to the transition temperature and adding the entropy of the solid phase transi-
tion.

When the final temperature is the transition temperature, these two calculations give
the same result.

11.6 Worked
Example on
Allotropic
Transformations 11.6 As solid methylammonium chloride has its temperature raised under a pressure of one atmosphere
it undergoes a solid phase transition from a phase denoted ~ to one denoted y at a temperature of
220.4 K. Calculate the molar enthalpy of this transformation using the values of the molar entropy
changes ~Sm given below.

~Sm (~ solid, from absolute zero to 220.4 K) = 92.26 J K - l mol- 1

~Sm (y solid, from absolute zero to 264.5 K) = 115.57 J K - l mol- 1

~Sm (y solid, from 220.4 K to 264.5 K) = 15.20 J K - l mol- 1

Solution The changes in the entropy of a system consisting of 1 mol of molecules of methylammonium
chloride may be determined by allowing the temperature of the system to vary slowly at (constant)
atmospheric pressure. The molar entropy of either phase may be taken as zero at absolute zero.
Then, the molar entropy of a system in they phase at the transition temperature may be calculated
using the two following paths.
(a) Start with the system in they phase at absolute zero and raise the temperature slowly to that
of the phase transition.
The change in the molar entropy is

~Sm(Y solid, 0 K ~ 220.4 K)

(b) Start with the system in the ~ phase at absolute zero and raise the temperature of the system
to that of the ~ ~ y transition, but keeping the material in the ~ phase. The change in the molar
entropy of the system is

~Sm(~ solid, 0 K ~ 220.4 K)

Then add the molar entropy change of the transition, which is given by

where Hm is the molar enthalpy of the allotropic transition and There is the temperature at which
the transition takes place under a pressure of one atmosphere. Then,

~Sm(Y solid, 0 K ~ 220.4 K) = ~Sm(~ solid, 0 K ~ 220.4 K) + Hm/T

166
However, the information given is of dSm(Y solid) for the temperature ranges 0 K to 264.5 K and
220.4 K to 264.5 K. Therefore,

dSm(Y solid, 0 K ~ 220.4 K) = dSm(Y solid, 0 K ~ 264.5 K)


- dSm(Y solid, 220.4 K ~ 264.5 K)

so that

dSm(Y solid, 0 K ~ 264.5 K) - dSm(Y solid, 220.4 K ~ 264.5 K)


= dSm(P solid, 0 K ~ 220.4 K) + Hm/T
Substitution gives

so that

Hm = 1790 J mol- 1

11.7 Exercises
11.1 Show that for a closed hydrostatic system, the heat capa- where a is the surface tension (specific surface free energy) and
city at constant pressure, CP, satisfies the condition A is the surface area of the film.

lim cp = 0 11.3 Show that, for a closed hydrostatic system, the cubic
r~o
expansivity tends to zero as the temperature tends to zero.
11.2 Show that, for a surface film of liquid helium, 11.4 Show that the third law predicts that the ideal gas equa-
tion cannot hold at the absolute zero of temperature.
lim (iJaliJT)A
r~o
=0

167
Topic Guide
12.1 Irreversible Processes and
Thermodynamics
12.2 The Joule Process
12 Irreversible Processes
12.3 Worked Examples on the
Joule Process
12.4 The Joule-Thomson
Process
12.5 Worked Examples on the
Joule-Thomson Process
12.6 Exercises

12.1 Irreversible Each equilibrium state of a system is characterised by a set of values of the thermodyn"
Processes and amic coordinates that does not depend on the way in which the equilibrium state was
reached. Processes that are, effectively, a succession of equilibrium states can also be
Thermodynamics described in terms of the thermodynamic coordinates, but this is not true for processes in
which there are finite gradients in one or more of the intensive coordinates. Such
processes are irreversible. However, provided that a given process takes place between
two equilibrium states of the system, thermodynamics can be used to relate the values of
the coordinates in these two equilibrium states, even though no information can be
gained about the process itself. The technique, which has been described already, is to
find a notional reversible process linking the required initial and final equilibrium states
and then use that process to determine the relation between the coordinates. This
approach will now be applied to two important irreversible processes: the Joule process
(free expansion) in which the initial and final states of the system are true equilibrium
states and the Joule-Thomson process, which is a continuous flow process.

12.2 The Joule In a free expansion a gas is allowed to expand in such a way that there is no work
Process interaction and no thermal interaction with the surroundings. The apparatus used by
Joule is shown in Fig. 12.1. In this, two identical copper vessels are connected by a short
length of tube fitted with a stopcock. One vessel contains the gas and the other is
evacuated. When equilibrium is achieved the stopcock is opened and an irreversible gas
flow takes place until a new equilibrium state is achieved.
From the first law it follows that the internal energy U of the gas is the same in the
R E initial and final equilibrium states. The process is highly irreversible, but a relationship
between the coordinates in the initial and final equilibrium states may be obtained by
replacing the real process with a notional reversible process that takes place at constant
internal energy.
Let the initial equilibrium state of the gas be characterised by a pressure pi, a volume
Vi and a temperature Ti, and let the respective values for the final equilibrium state be Pt.
Figure 12.1 Joule's free Vt and Tt. The change in temperature which occurs in an infinitesimal part of the notional
expansion apparatus. reversible process will be determined by the value of (aT!aV)u, a quantity known as the
Joule coefficient. An expression for the Joule coefficient may be developed as follows.
Using a reciprocity relation, the Joule coefficient may be written

(aT!aV)u = - (awav)T (aT!aU)v

Now, for a reversible process,

(aUJaT)v = Cv

the heat capacity at constant volume. The entropy form of the first law is

dU = TdS- pdV

168
where S is the entropy of the system. Differentiating with respect to V, keeping T
constant gives

But,

(M3)

so that

(iJT!iJV)u = -(l!Cv) [T(iJp!iJT)v- p] (12.1)

In a finite process, in which the volume of the system changes from V; to V1, the change
in temperature ~ T is given by

(12.2)

12.3 Worked
Examples on the
Joule Process 12.1 Calculate the change in temperature of one mole of a monatomic van der Waals gas when it
undergoes a free expansion from a volume of 1.0 x 10- 1 m3 to one of 2.0 x 10- 1 m3 .
Take the constant a in van der Waals' equation to be 0.14 N m4 mol- 2 and the molar heat
capacity at constant volume to be a constant equal to 12.47 J K- 1 mol- 1 .

Solution The system is 1 mol of molecules of a monatomic gas that obeys van der Waals' equation and the
process that it undergoes is a free expansion from an initial equilibrium volume V mi to a final
equilibrium volume V mt·
For a closed system consisting of 1 mol of molecules the change in temperature t-.T produced by
a free expansion is given by

I'!.T = -
vmf
f
(1/Cv.m) [T (Bp/BT)vm- p] dVm
vmi
where p is the pressure, T is the temperature and Cv,m is the molar heat capacity at constant
volume.
To evaluate the integral, the relationship between p, V and Tis needed. For 1 mol of a gas that
obeys van der Waals' equation

[p + (a/V~)J(Vm- b)= RT

or,

p = [RTI(Vm- b)]- [a/V~]

where a and b are constants characteristic of the gas. Therefore,

(Bp/BT)vm = RI(Vm- b)

and I'!.T = - f Vmf


(1/Cv,m) {[RTI(Vm- b)]- p} dVm
Vmi
Substituting for p from the equation of state gives

t-.T =- f Vmf
(1/Cv,m) {[RT/(Vm- b)]- [RTI(Vm- b)]+ [a/V~]} dVm
vmi

169
=- (0.14 (N m 4 mol- 1 )/12.47 (J K- 1 mol- 1 )) x [(111.0 x 10- 3 (m 3 )) - (112.0
X 10- 3 (m 3 ))]
= - 5.6 K

12.2 One mole of helium gas, occupying a volume of 0.12 x 10- 3 m 3 at a temperature of 293 K,
undergoes a free expansion to a final pressure of 1.1 x 105 Pa. Calculate the change in the
temperature of the gas produced by this process, assuming that helium obeys van der Waals'
equation of state with a = 0.0034 N m 4 mol- 2 and b = 23.70 x 10- 6 m 3 mol- 1 and that the molar
heat capacity at constant volume is 12.52 J K- 1 mol- 1 .

Solution The system is one mole of helium gas that is assumed to obey van der Waals' equation of state. The
system undergoes a free expansion.
Let the initial equilibrium state be characterised by a pressure Pi, molar volume V mi and
temperature Ti, and let the corresponding values for the final equilibrium state be Pt, V mf and Tf.
In Example 12.1 it was shown that, when a system consisting of one mole of a van der Waals gas
undergoes a free expansion, the change in temperature !:!..Tis given by

Here V mi, Ti and Pt are known but, to determine !:!.. T, V mf must be determined. Applying van der
Waals' equation to the initial and final equilibrium states gives

To determine V mf using this equation Pi and Tf must be known. Pi is readily determined from the
initial conditions using van der Waals' equation in the form

Pi= [RTI(Vmi- b)]- (a!V~i)


= [8.315 (J K- 1 mol- 1 ) X 293 (K)/(0.12 X 10- 3 (m 3 ) - 23.70 X 10- 6 (m 3 mol- 1 ))]
- [0.0034 (N m4 mol- 2 )/(0.12 x 10- 3 (m3 )) 2 ]
= 250.6 x 105 Pa

Now, assuming that Tf = Ti, the value of Vmf may be obtained as follows. Expanding van der
Waals' equation gives

p v;,- v;,(pb + RT) +a Vm- ab = 0

which is a cubic equation in Vm and is most easily solved by an iterative process. First, an
approximate value for V mf is obtained using the ideal gas equation and assuming that the
temperature change is negligible. Then,

or,

250.6 X 105 (Pa) X 0.12 X 10- 3 (m3 ) = 1.1 X 105 (Pa) X Vmf

Therefore, as a first approximation, take Vmf as 27.338 X 10- 3 m3 •


In the first iteration this value of V mf is used in van der Waals' equation in the form

pVm/RT = 1 + pb/RT- a!(RTVm) + abi(RTV~) (12.3)

so that

PtVtiRTt = 1 + [(1.1 X 105 (Pa) X 23.70 X 10- 6 (m 3 mol- 1 ))/(8.315 (J K- 1 mol- 1 )


X 293 (K))] - [0.0034 (N m4 mol- 2 )/(8.315 (J K- 1 mol- 1 ) X 293 (K) x 27.338
x 10- 3 (m 3 ))] + {(0.0034 (N m4 mol- 2 ) x 23.70 x 10- 6 (m 3 mol- 1 ))/
[8.315 (J K- 1 mol- 1) X 293 (K) X (27.334 X 10- 3 (m3 )) 2 ]}
= 1.001 019

170
Therefore,

= 22.17 X 10- 3 m 3

In the second iteration this new value of V mf is substituted in Equation (12.3) to give a better value
of vmf· Then,

PrVriRTr = 1 + [(1.1 x 105 (Pa) x 23.70 x 10- 6 (m 3 mol- 1))/(8.315 (J K- 1 mol- 1)


x 293 (K))] - [0.0034 (N m4 mol- 2 )/(8.315 (J K- 1 mol- 1) X 293 (K)
x 22.17 x 10- 3 (m3 ))] + {(0.0034 (N m4 mol- 2 ) x 23.70 x 10- 6 (m 3 mol- 1))/
[8.315 (J K- 1 mol- 1 ) X 293 (K) X (22.17 X 10- 3 (m3 )) 2 ]}
= 1.000 010 399
Therefore,

Vmf = 22.17 X 10- 3 m 3

This value differs by less than 1% from that obtained in the previous iteration and may, therefore,
be considered as satisfactory.
Therefore,

11T =- (a!Cv, m)[(1/Vmi) - (1/Vmr)J


= -(0.0034 (N m4 mol- 2 )/12.52 (J K- 1 mol- 1 )) x [(103 /0.12 (m3 ) ) - (103 /22.17 (m 3 ))]
= -2.3 K

12.3 Derive an expression for the Joule coefficient of a gas that obeys Berthelot's equation of state

[p + (a'ITV~)J(Vm- b') = RT

where V m is the molar volume under a pressure p and at a temperature T, R is the molar gas
constant and a' and b' are constants.

Solution Take as the system one mole of molecules of a gas that obeys Berthelot's equation. For any molar
closed hydrostatic system the Joule coefficient (aT/aV)um is given by

(aT!aV)um = - (l!Cv,m) [T(ap!aT)v m - p]

where Cv,m is the molar heat capacity at constant volume. The equation of state is needed to
determine (ap!aT)v,m· Berthelot's equation may be written

p = [RT!(Vm - b')] - [a' lTV~)]

which gives

so that

(aT!aV)um =- (l!Cv,m) [RTI(Vm - b') + a'TI(T2 V~) - p]


= -(l!Cv,m) [RTI(Vm- b') + a'I(TV~)- RTI(Vm- b') + a'I(TV~)]
= -(l!Cv,m) [2a'/(TV~)]

12.4 Many gases at low pressures obey approximately the equation of state

171
where V m is the molar volume at a temperature T and pressure p, R is the molar gas constant and
B'(T) is a function of temperature only.
Nitrogen at low pressures obeys the above equation of state with B'(T) varying with tempera-
ture as shown in the table. If the molar heat capacity at constant volume for nitrogen is treated as a
constant of value 20.79 J K- 1 mol-\ determine the change in temperature of 1 mol of nitrogen
when it undergoes a free expansion from an initial equilibrium state with a temperature of 240 K
and a volume of 20.0 x 10- 3 m3 to a final equilibrium state with a volume of 40.0 x 10- 3 m 3 •

TIK 120 150 200 273


B'/10- 6 m3 mol- 1 -114.6 -71.2 -34.3 -9.5

Solution The system is one mole of molecules of a gas that obeys the equation of state

and the process that it undergoes is a free expansion. For any closed hydrostatic system the change
in temperature produced by a free expansion is obtained from the Joule coefficient and the
equation of state.
For a system consisting of 1 mol the Joule coefficient is given by (see Example 12.1)

(aTJaV)um = - (1/Cv,m) [T(apJaT)vm - p]

By writing the equation of state in the form

p = RT/Vm + RTB'(T)IV;..

it is readily seen that

(ap!aT)vm = (R!Vm) + RB'(T)JV;.. + (RTIV;..)(dB'IdT)

B' is a function of Tonly so that the total derivative is used. Substituting for (aplaT)vm and for pin
the equation for the Joule coefficient gives

(aTJaV)um = - (liCv,mH (RTIVm) + (RTB' IV;..) + [(RT 21V;..)( dB' /dT)]


- (RTIVm) - (RTB' IV;..)}
= - (1/Cv,m) [(RT 2 1V;..) (dB'IdT)]

In a finite process, in which the molar volume changes from V mi to V mf, the change in temperature
d T is given by

assuming that Cv,m is a constant. If it is further assumed that d Tis small, both T and dB' /dT may
be treated as constants. Then

For nitrogen the variation of B' with Tis shown in Fig. 12.2. From this figure, the value of dB' /d T
when Tis 240 K is 0.34 X 10- 6 m 3 K- 1 mol- 1 • The value of Cv.m is 20.79 J K- 1 mol- 1 and so

dT = [8.315 (J K- 1 mol- 1 ) X (240f (K2 ) x 0.34 x 10- 6 (m 3 K- 1 mol- 1))/


[20.79 (J K- 1 mol- 1)] x [(1/40.0 x 10- 3 (m 3 mol- 1) ) - (1/20.0 x
10- 3 (m 3 mol- 1))]
= -0.20 K

172
0 100 300 T/K
-10
-20
~ -30
I
0 -40
..,E -50
..E -60
I
~ -70
~ -80
ill -90
-100
-110
Figure 12.2 The dependence -120
of B' on T for nitrogen at low
pressures.

12.4 The Joule- The Joule-Thomson process is an example of a continuous flow process in which a gas is
Thomson Process made to expand continuously from a high pressure to a lower pressure through a plug of
porous material known as a throttle. The essential features of an arrangement for
examining the Joule-Thomson process are shown in Fig. 12.3.

Figure 12.3 The essential


features of a Joule-Thomson Thermal Throttle
arrangement. insulation

Since the process is one of continuous flow, pressure gradients are present in the gas so
that any convenient thermodynamic system chosen in the arrangement is not in thermo-
dynamic equilibrium. However, when steady conditions are achieved, it is possible to
make measurements of the pressure p and temperature T at a particular location in the
arrangement that characterise the condition of the gas in that region. Therefore, the gas
just about to pass through the throttle and the gas that has just passed through the
throttle may be treated as being in states of quasi-equilibrium. The gas about to pass
through the throttle is characterised by a pressure Pi and temperature Ti , while the
corresponding values for the gas that has just passed through the throttle are Pt and Tt .
To obtain a relationship between the values of the temperature and pressure, the
throttling process may be modelled as shown in Fig. 12.4.

Throttle

Figure 12.4 A model for the %1Wff$~#'($//ff//W'ff.@'/2


throttling process. (b)

173
The system is a fixed mass of gas at a temperature Ti and pressure pi, maintained on
the high-pressure side of the throttle by two pistons A and B, as in Fig. 12.4(a). These
pistons reproduce the effects of the remainder of the flowing gas in the real arrangement.
The pistons A and B then move slowly and simultaneously at the appropriate, but
different, speeds so the pressure of the gas between piston A and the throttle is
maintained at the constant value pi, while that of the gas between the throttle and the
piston B is maintained at the constant value Pf· Both pistons are stopped when all the gas
has just passed through the throttle and occupies a volume Vf at a pressure Pro as in Fig.
12.4(b). It is readily shown that, for the process considered,

that is, the enthalpy of the mass of gas chosen as the system is the same before and after
the throttling process.

12.5 Worked
Examples on the
Joule-Thomson
Process 12.s Derive an expression for the rate of change of temperature with pressure for a gas undergoing a
throttling process and draw a graph in the pressure-volume plane showing the results of ex-
perimental measurements.

Solution Take as the system a fixed mass of gas that passes through the throttle. The process may be
modelled as shown in Fig. 12.4. Therefore, to obtain a relationship between the values of pressure
and temperature in the initial and final equilibrium states of the process modelling the real
throttling process, the irreversible process must be replaced by a notional reversible process linking
the same equilibrium states as the irreversible process, and it is convenient to choose a reversible
process that is carried out at constant enthalpy.
Consider a fixed mass of gas undergoing a reversible process at constant enthalpy. The change in
temperature that is produced is specified by the Joule-Thomson coefficient 11, defined by

(12.4)

Using the reciprocity relation

gives

Now

where CP is the heat capacity at constant pressure. Also, since

H= U + pV

dH = dU + pdV + Vdp
From the entropy form of the first law

dU = TdS- p dV

so that

dH=TdS+Vdp

174
Then

(oH!op)r = T(oSJap)r + v

which, using Maxwell relation (M1), becomes

(oH!op)r = - T (oV!oT)p + V

Therefore,

l..l = (1/Cp)[T(oV/oT)p - V (12.5)


= (VICP)[PT- 1] (12.6)

where p is the cubic expansivity.


The results of experimental measurements on the Joule-Thomson effect are conveniently
plotted as lines of constant enthalpy (isenthalps) in the pressure-temperature plane. The results for
a typical common gas are shown in Fig. 12.5.

Lines of constant
enthalpy

Figure 12.5 Isenthalpic f----Vapor pressure


curves for a throttling process. curve
C is the critical point. p

The slope of an isenthalpic curve is, of course, the Joule-Thomson coefficient l..l· The locus of the
points for which l..l is zero is shown as a dashed line in Fig. 12.5 and is known as the inversion curve;
the corresponding temperature is the inversion temperature. Figure 12.5 shows that there is a
maximum value for the inversion temperature. This value is a characteristic parameter of the
system. From Equations (12.5) and (12.6), the equation of the inversion curve is

or Ti (oV/oT)p = V (12. 7)

where Ti is the inversion temperature.

12.6 Show that, when a gas undergoes a Joule-Thomson expansion, the Joule-Thomson coefficient l..l is
given by

where V m is the molar volume when the pressure is p and the temperature is T, Cp,m is the molar
heat capacity at constant pressure and Um is the molar internal energy.

Solution Take as the system one mole of molecules of the gas that is passing through the throttle. For this
system the Joule-Thomson coefficient l..l is defined by

l..l = (oT!op)nm

where Hm is the molar enthalpy. Using the reciprocity relation

175
the expression for !.t may be written

Now,

so that

Further,

so that

This expression shows that !.t can be expressed as the sum of terms representing the deviation of the
behaviour from Boyle's law and from Joule's law.

12.7 An ideal gas undergoes a throttling process in which the pressure changes from 4.4 x 105 Pa to 1.1
x 105 Pa on passage through the throttle. If the temperature on the high-pressure side of the
throttle is 323 K, calculate

(a) the temperature change of the gas;


(b) the change in the molar entropy of the gas.

Solution Take a fixed mass of gas as the system. The change in temperature of the system as a result of
passing through the throttle is determined by the value of the Joule-Thomson coefficient !.t· For
any closed hydrostatic system !.t is given by

!.t = (1/Cp) [T(aVJaT)p - V]

where V is the volume occupied by the system at a temperature T and pressure p, and CP is the
heat capacity at constant pressure. For an ideal gas

where V m is the molar volume and R is the molar gas constant. Therefore, for one mole of an ideal
gas,

and

But,

Vm = RT/p

so that

that is, an ideal gas suffers no change of temperature on passing through a throttle, whatever the
initial conditions.
The entropy change brought about by the passage of the gas through the throttle must be

176
calculated using a notional reversible process linking the two (quasi-) equilibrium states of the gas
on either side of the throttle. This notional process should be at constant enthalpy for any system
and, in addition, for an ideal gas should be at constant temperature.
Start by treating the entropy S of the system as a function ofT and p, that is, by writing

S = S(T, p)

Then

dS = (as;aT)p dT + (as;ap)T dp
For a process taking place at constant temperature dT is zero and

Using Maxwell relation (Ml) this equation becomes

dS = - (av!aT)p dp

For a system consisting of one mole of molecules of ideal gas

so that

Therefore, for a throttling process involving an infinitesimal pressure drop across the throttle,

dSm = -(Rip) dp

and, in a throttling process involving a finite pressure difference across the throttle, the change in
the molar entropy is ~Sm, given by

~Sm = - R ln(pf/pi)
= -8.315 (J K- 1 mol- 1) X ln(l.l X 105 (Pa)/4.4 X 105 (Pa))
= 11.5 J K- 1 mol- 1

12.8 Show that, for a gas that obeys van der Waals' equation of state

[p + (a!V~)J[Vm- b] = RT

the Joule-Thomson coefficient f.t is given by

and that the equation of the inversion curve is

Ti = (2a!Rb)[l- (b!VmW

Here, V m is the molar volume when the temperature is T and the pressure is p, R is the molar gas
constant and a and b are constants.

Solution Take as the system one mole of molecules of a gas that obeys van der Waals' equation. The
Joule-Thomsom coefficient f.t is defined by

and it has been shown that, for any molar closed hydrostatic system, f.t is given by

177
where Cp,m is the molar heat capacity at constant pressure.
Writing van der Waals' equation in the form

pVm- pb + (a/Vm)- (ab/V;,) = RT

and differentiating with respect to T, keeping p constant, gives

or,

Therefore,

1.t = (1/Cp,m){RT/[p- (a/V;,) + (2ab/V~)]- Vm}

= (1/Cp,m)[RTV~/(pV~- aVm + 2ab)- Vm]


Now,

p = [RT/(Vm - b)] - (aiV;,)

so that

which, when expanded and collected up, gives

The inversion curve is the locus of those points in the p-Tplane for which l.t is zero. Therefore,
the inversion temperature T; is obtained by putting the numerator in the equation for l.t equal to
zero. Then

or,

T; = [2a(Vm- b) 2 ]/RbV;,
= [2a(v;,- 2Vmb + b2 )]1Rbv;,
= (2a/Rb)[1- (b!VmW

12.9 The behaviour of many gases at low pressure is represented approximately by the equation

pVm = RT+ B(T)p


where Vm is the molar volume at a pressure p and temperature T, R is the molar gas constant and
B(T) is a function of temperature only. For hydrogen the value of B varies with T as shown in the
table. Obtain a value for the Boyle temperature and for the inversion temperature of hydrogen.

-10 20 40 55 71 82 88 92 93
98 123 143 173 223 273 323 373 423

Solution The Boyle temperature TB is the temperature at which the gas closely obeys Boyle's law over a
fairly wide range of temperatures. Take as the system one mole of molecules of hydrogen. This
system satisfies the equation of state

pVm = RT + B(T)p

178
while the equation of state of one mole of ideal gas is

Therefore, at low but finite pressures, hydrogen will behave like an ideal gas at the temperature at
which B(1) is zero.
The inversion temperature T; is the temperature at which the Joule-Thomson coefficient !l is
equal to zero. It has been shown that, for any closed hydrostatic system, T; is given by

For hydrogen at low pressures

Vm = RTip + B(T)
so that

The total derivative is used as B depends only on T. Therefore, the condition for T; is

RT;Ip + T;(dBidT) = RT;Ip +B


or,

(dBidT) =BIT;

Both TB and T; may be obtained from the graph of B against T, shown as Fig. 12.6. The Boyle
temperature is readily obtained as the temperature at which B is equal to zero. From Fig. 12.6 the
result is

TB = 103 K

The inversion temperature is obtained by drawing the tangent to the B-T curve from the origin of
coordinates. At the point of contact of this tangent, shown in Fig. 12.6,

dBidT =BIT

100

-20
Figure 12.6 The dependence
of B on T for hydrogen.

Therefore,

T; = 196 K
Note that this construction gives only a single value of the inversion temperature, and gives no
indication of a range of values as shown in Fig. 12.5. This is a consequence of the particular
equation of state that has been used. This equation is valid only at low gas pressures where the

179
inversion curve is practically horizontal, giving a unique value of the inversion temperature that is,
in fact, equal to the maximum inversion temperature of the gas.

12.10 A certain gas obeys the equation of state


p(Vm- b)= RT

where V m is the molar volume when the pressure is p and the temperature is T and R is the molar
gas constant. For the gas in question, b is equal to 20 x 10- 6 m 3 mol- 1 . Calculate the change in the
temperature of the gas when it undergoes a Joule-Thomson expansion from a pressure of 1.1 X 107
Pa to 1.1 x 105 Pa. The molar heat capacity at constant pressure for the gas may be taken as a
constant equal to 20.79 J K- 1 mol- 1 .

Solution Take as the system one mole of molecules of the gas. The temperature change produced by a
throttling process may be obtained using the Joule-Thomson coefficient !land the equation of
state. For any closed hydrostatic system consisting of one mole of molecules !l is given by

The partial derivative is obtained from the equation of state which, for the gas under considera-
tion, is

p(Vm- b)= RT

or,

Vm = (RT/p) + b
Therefore,

and

!l = (1/Cp,m)[(TR/p)- Vm]
= (1/Cp,m)[(RT/p) - (RT/p) - b]
= -b/Cp,m

Therefore, in a throttling process in which there is a finite pressure drop across the throttle from Pi
to Pt, the change in temperature ~ T is given by

~T = !l (Pr - Pi)
= - (b/Cp,m)(Pr - Pi)

= -[20 x 10- 6 (m 3 mol- 1)/20.79 (J K- 1 mol- 1 )] x [1.1 x 105 (Pa)- 1.1 x 107 (Pa)]
= 10.5 K

The temperature of the gas rises as a result of passing through the throttle.

12.11 The behaviour of a certain gas at a temperature Tis represented approximately by the equation of
state

pVm=RT+B(T)p

where Vm is the molar volume, pis the pressure, R is the molar gas constant and B(T) is a function
of temperature only.
If B(T) is given by

180
B (m 3 mol- 1 ) = {4.7 - [15201(T (K))]} x 10- 6

calculate the maximum inversion temperature and the Boyle temperature of the gas.

Solution Take as the system one mole of molecules of the gas. For the given equation of state the Boyle
temperature Ta is the temperature at which the value of B is zero (see Example 12.9). Therefore,

4.7- 1520 (K)ITa = 0

or,

TB = 152014.7 = 323 K

The equation of the inversion curve is l..t = 0, where l..t is the Joule-Thomson coefficient. For a
system consisting of one mole of any gas this condition may be written (see Example 12.8)

Now, for the gas considered

Vm = RTip + B

so that

Then

Ti [(Rip)+ dBidT] = Vm

From the equation of state

Vm=RTip+B

so that

Ti [(Rip) + dBidT] = RT/p + B

and

Ti = Bl(dBidT)

Now

B = [4.7 - (15201T)] x 10- 6

so that

dBidT = [1520 x w- 6 JIT 2

Therefore,

Ti = {[4.7 - (15201Ti)] X 10- 6 }1[(1520 X 10- 6 )1Tn

Expanding gives

Ti = (4.7 Tf - 1520 Ti)11520

or

4.7 n- 3o4o ri = o
181
Solving this quadratic equation gives

T; = 0 or 647 K

T; is not accessible and so

T; = 647 K

12.6 Exercises
12.1 One mole of argon undergoes a free expansion from an where Ta is the Boyle temperature of the gas.
initial volume of 1.0 x 10- 3 m3 to a final volume of 2.0 x Berthelot's equation of state is
10- 3 m3 • Calculate the change in temperature of the gas pro-
duced by the expansion, assuming that argon obeys van der [p + a'/(TV~)](Vm- b') = RT
Waals' equation of state with a = 0.136 N m4 mol- 1 •
The molar heat capacity at constant volume for argon may be where V m is the molar volume at a pressure p and temperature
taken as a constant equal to 12.6 J K- 1 mol- 1 • [ - 5.4 K] T, R is the molar gas constant and a' and b' are constants.

12.2 Determine the ratio of the inversion temperature to the 12.5 In a particular Joule-Thomson arrangement in which oxy-
critical temperature for a gas that obeys van der Waals' equation gen is expanded through a throttle, the density of the gas
of state. [6.75] changes from 0.25 kg mol m- 3 on the high-pressure side of the
throttle to 0.08 kg mol m- 3 on the low-pressure side. Calculate
12.3 At low pressures the behaviour of argon is well rep- the temperature change of the gas as a result of passing through
resented by the equation of state the throttle, assuming that oxygen obeys van der Waals' equcv
tion of state and the parameters have the following values:
pVm = RT + B(T)p
a = 138 x 103 N m4 kg- 2 mol- 2
where V m is the molar volume when the pressure is p and the
temperature is T, R is the molar gas constant and B is a function b = 31.83 X 10- 3 m 3 kg- 1 mol- 1
of temperature only. At a temperature of 273 K the value of Cp,m = 20.8 J K- 1 mol- 1
B( T) for argon at low pressure is - 22 x w- 6 m- 1 and the value [ -1.6 K]
of dB/dT is 0.25 X 10- 6 m3 K-l mol- 1 • Calculate the value of
the Joule-Thomson coefficient for low-pressure argon at a 12.6 For a gas at low pressures one form of series equation of
temperature of 273 K. [0.4 x 10- 5 K Pa- 1] state is

12.4 For a gas obeying Berthelot's equation of state, show that pVm!RT = 1 + B'(T) p + C'(T) p 2
the inversion temperature T1 is given by
where V m is the molar volume at a temperature T and pressure
p, R is the molar gas constant and B' and C' are functions of
temperature only.
Show that, in the limit of vanishingly small pressures,
where TiD is the value of the inversion temperature in the limit of
vanishingly small gas pressure and is given by lim 11 Cp,m = RT 2 (dB'IdT)

T; 0 = (3a'!Rb')vz where 11 is _the Joule-Thomson coefficient and Cp,m is the molar


heat capactty at constant pressure. Show also that the equation
Show also that of the inversion curve is

p =- (dB' /dT)I( dC' ldT)

182
Topic Guide

13 A Simple Kinetic Theory


13.1 A Simple Kinetic Theory of
Gases
13.2 Worked Examples on a
Kinetic Theory of Gases
13.3 Exercises of Gases

13.1 A Simple The kinetic theory of matter seeks to show how the properties of matter in bulk may be
Kinetic Theory of explained in terms of the behaviour of the basic particles composing matter. The success of
this approach will depend on attributing suitable properties to these constituent particles.
Gases Gases at low pressures (dilute gases) are among the simplest systems to treat from a
kinetic theory standpoint. This is reflected in the simple form of the macroscopic
equation of state: for a system consisting of one mole of molecules of an ideal gas, the
pressure p, molar volume V m and temperature Tare related by the equation

pVm = RT (13.1)

where R is the molar gas constant. The expression for the molar internal energy Um of an
ideal gas may be written
Um = F(T) (13.2)

where F(T) is a universal function of temperature.


To model, at the microscopic level, the behaviour described by Equations (13.1) and
(13.2), something must be known of the way in which gas molecules interact with each
other and with the walls of the containing vessel. Simple experiments suggest that the
molecules of a gas travel in random directions between collisions and that, when a
molecule strikes the wall of the container, it sticks there for a short time (the residence
time) before being released from the wall by a thermal fluctuation in the wall or by being
struck by another gas molecule. A molecule leaves the wall in a random direction bearing
no relation to its incident direction.
When the effect of a large number of molecules moving in random directions is consi-
dered, the net effect of their striking the walls of the container from random directions,
sticking to the wall and then being ejected in random directions, is as if the molecules were,
in fact, regularly reflected from the walls of the container. This will be true irrespective of the
nature of the gas and, in particular, of whether it is a monatomic, diatomic or more complex
molecule. Therefore, the molecules may be treated as spheres.
The observation that, in equilibrium, there is no net gas flow shows that gas molecules
leaving a given area of the wall surface in a given time carry away as much momentum as
is brought up to that area of surface in the same time. Therefore, the molecules may be
treated as elastic spheres.
It is further assumed that the molecules have a very small diameter compared with
their average separation (i.e. the repulsive interaction is very short-range) and that the
attractive forces between them are negligible.
On the basis of the above assumptions it is a straightforward matter to show that the
pressure p exerted by the model gas is given by

(13.3)

where pis the density of the gas, m is the mass of each molecule, n is the number of
molecules in unit volume and (c 2 ) is the mean square speed of the molecules, defined by

(13.4)

183
where n1 is the number of molecules per unit volume with a speed c~> and so on.
Equation (13.3) may be written
(13.5)

showing that

p = ~ x (total kinetic energy of the molecules in unit volume) (13.6)

From Equation (13.3)

pVm = 1mnVm (c 2)

1mNA (c 2) (13.7)

where NA (= nVm) is the number of molecules in 1 mol (Avogadro's constant), and,


since the molar mass Mm is equal to mNA>

(13.8)

The concept of temperature may be introduced into kinetic theory if it is assumed that
Equations (13.1) and (13.8) both describe the behaviour of an ideal gas. Then

RT=1Mm(c 2 )

or,

(13.9)

Further, the average kinetic energy per molecule, which is equal to ~m (c 2 ) , is given by

~m (c 2 ) = ~ mRT!Mm = ~ RTINA
= ~kT (13.10)

where k is Boltzmann's constant.


For a gas composed of monatomic molecules, it may be further assumed that all the
energy of the gas resides in the kinetic energy of translation, since there can be no energy
of vibration or rotation. Then, the molar internal energy may be written

Consequently, the molar heat capacity at constant volume Cv,m for a monatomic gas is
given by

Cv,m = ~R (13.11)
The actual speed distribution for a gas of point molecules is given by the Maxwell-
Boltzmann distribution: the number of molecules in unit volume with speeds that lie in
the range c to c + de is N(c), given by

N(c) = 4:rt c 2 n(m/2nkT) 312 exp{ -[(mc 2 )/(2kT)]} (13.12)

Real molecules have finite diameters and undergo frequent collisions with each other.
The average distance travelled between such collisions is known as the mean free path of
a molecule. If it is assumed that a particular molecule is moving through a region of the
gas where the neighbouring molecules are travelling so slowly that they may be con-
sidered to be at rest (the fast molecule approximation), the mean free path If is given by

If= 1/(:rt~n) (13.13)

184
where a is the molecular diameter. This result is not much changed if it is assumed that
the molecular speeds are distributed according to the Maxwell-Boltzmann law. The
result is then

(13.14)

13.2 Worked
Examples on a
Kinetic Theory of
Gases 13.1 Determine the value of the molar gas constant R, given that the volume of one kilomole of all gases
at s.t.p. (a pressure of one atmosphere and a temperature of 273.15 K) is 22.4 m3 •
From this value obtain the root mean square speed of (diatomic) hydrogen gas molecules at
s.t.p. The relative molecular mass of hydrogen is 2.016.

Solution The equation of state for an ideal gas is

where V m is the molar volume at a pressure p and a temperature T. Therefore,

At s. t. p. the pressure of the gas is one atmosphere (101 325 Pa), the temperature is 273.15 K and
the volume occupied by 1 kmol is 22.4 m3 . Therefore

Vm = 22.4 X 10- 3 m3

and
R = (101 325 (Pa) x 22.4 x 10- 3 (m 3 mol- 1 ))/273.15 (K)
= 8.31 J K- 1 mol- 1

The mean square speed of the molecules (c 2 ) may be obtained from Equation (14.3) once the
density p of the gas has been determined.
Now

density = mass/volume

and the relative molecular mass of an element is the ratio of the average mass per molecule of the
natural isotopic composition of the element to 1/12 of the mass of an atom of the nuclide 12C.
Further, the mole is the amount of substance which contains as many elementary entities (mole-
cules in this example) as there are atoms in 0. 012 kg of 12C. Therefore, the mass of 1 mol of
hydrogen molecules is 2.016 X 10- 3 kg. Since the molar volume at s. t. p. is 22.4 X 10- 3 m 3 , the
density of hydrogen at s.t.p. is given by
p = (2.016 x 10- 3 (kg))/(22.4 x 10- 3 (m 3 ))
= 0.090 kg m- 3

But,

so that, at s. t. p.,
(c 2 ) = (3 x 101 325 (Pa))/(0.090 (kg m- 3 ))
= 3 377 500 m 2 s- 2

and

185
13.2 Calculate the total kinetic energy of the molecules in 2.0 m3 of (monatomic) argon gas at a
temperature of 273 K

(a) when the gas pressure is atmospheric, and


(b) when the gas pressure is 10- 2 torr.

Assume that argon behaves as an ideal gas.

Solution From Equation (13.6) the total kinetic energy Ek of the molecules in a volume V of an ideal gas is
given by

where pis the gas pressure.


(a) When the pressure is atmospheric (101 325 Pa) the corresponding value of Ek is

Ek(atmospheric pressure) = ~ x 101 325 (Pa) X 2.0 (m3 )


= 3.04 X 105 J

(b) A pressure of 1 torr is defined as (101 325/760) Pa so that, when the temperature is 273 K,
the molecules of an ideal gas exerting a pressure of 10- 2 torr have a total kinetic energy equal to

Ek(10- 2 torr) = ~ x (101 325/760) x 10- 2 (Pa) x 2.0 (m3 )


= 4.0 J

Notice that, from Equation (13.10), the average kinetic energy per molecule of an ideal gas at a
temperature T is ~ kT, a function of the temperature of the gas only and independent of the
pressure.

13.3 The escape speed Ve for a particle on the surface of the Earth is given by

where g is the acceleration of free fall at the surface of the Earth (g = 9.81 m s- 2 ) and R is the
radius of the Earth (R = 6360 km). Calculate the temperature of the atmosphere at which the root
mean square speed of an ideal gas is equal to ve

(a) for oxygen (relative molecular mass = 32.0);


(b) for hydrogen (relative molecular mass = 2.016).

On the surface of the Moon the acceleration of free fall is 1.62 m s- 2 and the radius of the Moon
is 1738 km. Repeat the calculations carried out for the Earth.

Solution From Equation (13.10) the mean square speed (c 2 ) is given by

where m is the mass of a molecule, k is Boltzmann's constant and Tis the temperature of the gas.
Therefore, the condition for the root mean square speed to equal the escape speed is that the
temperature shall equal Te, given by

or

Te = (2mgR)/(3k)
It is now necessary to calculate m. Now,

186
where Mm is the mass of one mole and NA is Avogadro's constant. Therefore,

But,

where Mr is the relative molecular mass. Therefore, for oxygen

and for hydrogen

The gas temperature Te(0 2) that makes the root mean square speed equal to the escape speed for
oxygen at the Earth's surface is, therefore,

Te(0 2 ) = (2 X 32 X 0.001 (kg mol- 1 ) X 9.81 (m s- 2 ) X 6360 X 103 (m))/


(3 X 1.380 X 10- 23 (J K- 1) X 6.022 X 1023 (mol- 1))
= 1.60 X 105 K

For hydrogen the required temperature is Te(H 2), given by

Te(H2 ) = (2 X 2.016 X 0.001 (kg mol- 1 ) X 9.81 (m s- 2 ) X 6360 X 103 (m))/


(3 X 1.380 X 10- 23 (J K- 1 ) X 6.022 X 1023 (mol- 1 ))
= 1.01 X 104 K

For the Moon's surface the corresponding values are

and

13.4 Show that, to a first approximation,


(density of gas phase)/( density of liquid phase) = ncr
where o is the molecular diameter and n is the number of molecules in unit volume of the gas.
Given that the density of liquid oxygen at its boiling point, under a pressure of one atmosphere,
is 1.1 x 103 kg m-3, estimate the diameter of the oxygen molecule.

Solution When unit volume of the gas phase contains n molecules, each of mass m, the density of the gas
phase is equal to mn.
If the molecules in the liquid phase are assumed to be in contact with their neighbours and,
further, are assumed to be in a cubic array, the space occupied by the liquid may be treated as an
array of cubes of edge length o, each cube containing one molecule. Therefore, the number of
molecules per unit volume of liquid is (1/o) 3 . It follows that the density of the liquid phase is given
by the product of the number of molecules per unit volume and the mass of each molecule, that is,
the density of the liquid phase is equal to m/( o) 3 • Therefore,

(density of gas phase)/(density of liquid phase) = mn/(m/o 3 )


= no 3
To determine the value of o, given the density of the liquid, it is necessary to determine both the
density of the gas phase and the value of n.

187
Consider one mole of molecules of the gas at s. t. p. For oxygen molecules the relative molecular
mass is 32 so that the mass of one mole is 32 x 10- 3 kg. The volume occupied by one mole of any
ideal gas at s.t.p. is 22.4 x 10- 3 m 3 (see Example 13.1). Therefore, the density of oxygen gas at
s.t.p. is given by

density of oxygen gas = (32 X 10- 3 (kg))/(22.4 X 10- 3 (m 3 ))


1.429 kg m- 3

The number of molecules in one mole is N A, Avogadro's constant, and, at s. t. p., these molecules
occupy a volume of 22.4 x 10- 3 m 3 • Therefore, at s.t.p., the number of molecules of oxygen per
unit volume is n, given by

n = (6.022 x 1023 (mol- 1 ))/(22.4 x 10- 3 (m3 mol- 1 ))


= 0.269 x 1026 m- 3
Therefore,

o 3 = (density of gas)/[n(density of liquid)]


(1.429 (kg m- 3 ))/[(0.296 x 1026 (m- 3)) x (1.1 x 103 (kg m- 3 ))]

= 4.83 X 10- 29 m3

giving, as an approximate value for the diameter of the oxygen molecule

a = 3.64 X 10- 10 m
= 0.36 nm

13.5 A vessel is divided into two parts of equal volume, by means of a plane partition, in the middle of
which is a very small hole. Initially, both parts of the vessel contain ideal gas at a temperature of
300 Kanda low pressure p. The temperature of one half of the vessel is then raised to 600 K while
the temperature of the other half remains at 300 K. Determine the pressure difference between the
two parts of the vessel when steady conditions are achieved.

Solution In both chambers, in all equilibrium or steady-state conditions, the equation

must apply. Here pis the gas pressure, n is the number of gas molecules per unit volume, m is the
mass of each molecule and ( c 2 ) is the mean square molecular speed. Let each compartment have a
volume V and, when the temperature is the same for both compartments, contain N molecules.
Then

pV = t mnV(c 2) = t mN(c 2 )
Now, V and m are constants and (c 2 ) is proportional to the temperature of the ideal gas.
Therefore, initially, for each compartment

p = KNT (13.15)

where K is a constant. Since each compartment contains N molecules, the total number of
molecules in the system is 2N.
When the temperature of one compartment is raised, the temperature of the other being kept
constant, the number of molecules in each compartment changes until the flux of molecules from
one compartment equals the flux of molecules from the other compartment. The flux of molecules
from a compartment is proportional to the number of molecules in the compartment and also to
the mean speed (c) of the molecules in that compartment.
When conditions are steady, let the temperatures of the two compartments be T1 and T2 ,
respectively, the corresponding numbers of molecules in each compartment be N 1 and N 2 , and the
mean molecular speeds be c1 and c2 • Then, when conditions are steady

(13.16)

188
and, since the total number of molecules remains constant,

(13.17)

Now, for the Maxwell-Boltzmann distribution (c) 2 is proportional to (c 2 ) which is proportional to


T. Therefore, Equation (13.16) may be written

or,

(13.18)

From Equation (13.15)

and

Pz = KNzTz
where p 1 and p 2 are, respectively, the pressures in the chambers at temperatures T1 and T2 •
Therefore,

(13.19)

Equations (13.17) and (13.18) are now used to determine N 1 and N 2 in terms of N. From Equation
(13.17)

and from Equation (13.18)

Therefore,

and

Now, T1 = 600 K and T2 = 300 K so that

N1 = 0.828 N
and

N2 = 1.172 N
Now substitute for N 1 and N 2 in Equation (13.19) and use the value of Kfrom equation (13.15),
remembering that, initially, T1 = T2 = 300 K. Then

P1 - Pz = (p/300 (K)N)(0.828N x 600 (K) - 1.172N x 300 (K))


= 0.48 p

13.6 For nitrogen gas at s.t.p. calculate the mean free path on the fast-molecule approximation.
Assume that the radius of the nitrogen molecule is 1.5 X w-lO m.
Solution Let the system be a fixed mass of nitrogen gas at s.t.p. Using the fast-molecule approximation, the
mean free path It is given by

189
lt = 1/(nmf)
where n is the number of molecules per unit volume and a is the molecular diameter. Assuming
that nitrogen may be treated as an ideal gas, the equation of state is

pVm = RT
where Vm is the molar volume at a temperature T and a pressure p. Therefore, at s.t.p.,

V m = RT/p = (3.815 (J K- 1 mol- 1 ) x 273 (K))/101 325 (Pa)


= 0.022 403 1m3 mol- 1
One mole of gas contains NA molecules, where NA is Avogadro's constant, equal to 6.023 x
1023 mol- 1 • Therefore

n = N AIVm = (6.023 x 1023 (mol- 1 ))/(0.022 403 1 (m3 mol- 1 ))


= 268.85 x 1023 m- 3
Therefore, given that a= 3.0 X 10- 10 m,

ft = 1/[268.85 X 1023 (m- 3 ) X :rt X (3.0 X 10- 10 (m)f]


= 1.3 X 10- 7 m

13.3 Exercises
13.1 For nitrogen the molar volume at s.t.p. is 0.0224 m3 and 13.3 In current ultra-high vacuum (UHV) practice a pressure
the relative molecular mass is 28. Calculate of 10- 11 torr is reached routinely. Calculate the number of gas
molecules present in 1.0 m3 of a UHV system at a pressure of 1.0
(a) the mass of 1 mol of nitrogen; X 10- 11 torr when the temperature of the gas is 300 K.
(b) the mass of one molecule of nitrogen; [3.2 X 1011 ]
(c) the number of molecules in one cm 3 of nitrogen at s.t.p.;
(d) the root mean square molecular speed at s.t.p. 13.4 Show that, for a gas whose molecular speeds satisfy the
Maxwell-Boltzmann distribution, the mean speed (c) and the
[28 X 10- 3 kg; 4.65 X 10- 26 kg; 2.69 X 1019 ; 493 m s- 1] root mean square speed ( c2 ) 112 are related by the equation

13.2 A certain mass of an ideal monatomic gas occupies a


volume of 0.2 m3 at a pressure of 1.0 X 106 Pa and a temperature
of 300 K. Calculate 13.5 If molecules of mass m 1 and radius r 1 diffuse through a gas
whose molecules have a mass m 2 and radius r 2 , show that the
(a) the number of moles of gas in the sample; mean free path 112 of the molecules of mass m 1 is given by
(b) the number of molecules in the sample;
(c) the total internal energy of the gas. l12 = {1/[:rt(rl + r2 ) 2 N 2 ]} [mzl{m 1 + m 2 )] 112
[80.3 mol; 4.84 x 1025 ; 3 x 105 J]

190
14 Heat Jran sfer
Topic Guide

14.1 Introduction
14.2 Heat Conduction
14.3 A General Approach
to Solving Heat
Conduction Problems
14.4 Worked Examples on Heat
Conduction
14.5 Heat Convection
14.6 Worked Examples on Heat
Convection
14.7 Thermal Radiation
14.8 Worked Examples on
Thermal Radiation
14.9 Exercises

14.1 Introduction As was indicated in Chapter 4, in the subject of classical, or equilibrium, thermodyn-
amics, the term heat is used to describe an energy transfer between a system and its
surroundings arising solely from a difference in temperature between the system and its
surroundings. Therefore, heat transfer occurs only across the boundaries of systems.
Any energy transfer that takes place within a system as a result of temperature gradients
within the system is a redistribution of internal energy. Further, thermodynamics,
despite its name, has no time-scale and says nothing about the rate at which energy
transfers take place. This study is the province of heat transfer processes, in which the
rate of energy transfer is related to appropriate physical parameters.
It is conventional in this study to describe the redistribution of internal energy that
takes place as a heat transfer and, for this chapter, this convention will be observed. Heat
transfer occurs by three different mechanisms known, respectively, as conduction, con-
vection and radiation. In many situations more than one mechanism operates simul-
taneously.
The appropriate equations for the three heat transfer processes will now be considered
in turn.

14.2 Heat Heat conduction is the transfer of energy from regions of higher temperature to regions
Conduction of lower temperature within a body, without involving visible mass transport.
The conduction of heat in a medium is described by Fourier's rate equation, which
may be stated as follows. The rate of conduction of heat dQ/dt through a small plane
I
11
I
12
surface of area A drawn in the medium is given by
I I
I I

(14.1)
I I
I
I
I
I dQ!dt = -kA(iJT!iJn)
----:
I I
~
dX

where (iJT/iJn) is the temperature gradient normal to the area and the negative sign
I I
I I
I I
I
I
I
I
I
I
indicates that the direction of heat flow is down the temperature gradient. Equation
I
I
I
I (14.1) defines a quantity k known as the thermal conductivity of the medium.
Experiment shows that k is independent of (iJT/iJn) and, except at very low tempera-
I I

tures, independent of A, has the same value both for steady and changing conditions and
1\1Area A is, in general, a function of temperature though, for many materials, over wide tempera-
ture ranges, this dependence is not strong.
Consider the mass of material of area of cross-section A, lying between two planes that
are a distance dx apart and both normal to the direction of heat flow, that is, a
one-dimensional heat flow is considered, as in Fig. 14.1.
The rate of heat flow across an area A of plane 1 is (dQ/dt) 1 , given by

(dQ/dt) 1 = - kA(dT!dx) 1
Figure 14.1 One-
dimensional heat flow. Assuming that k is a constant, the heat flow across an area A of plane 2 is given by

191
(dQ/dt) 2 = -kA(dT!dx) 2 = -kA d!dx[T+ (dT/dx)dx]

The net rate of heat flow into the element of material between planes 1 and 2 is dQ/dt,
given by

dQ!dt = [(dQ/dt)l - (dQ/dt)2]


= kA(d 2 Tidx 2 ) dx

This energy flow produces a change in the temperature of the material in this element,
given by

where m is the mass of material in the element and cP is its specific heat capacity at
constant pressure. Now,

m = pA dx

where p is the density of the material. Therefore,

or,

(14.2)

where a = k/pcP is the thermal diffusivity of the material. Equation (14.2) is known as
Fourier's equation. In three dimensions it becomes

( oT!ot) = a \PT (14.3)

When the element of material contains a uniformly distributed internal energy (heat)
source that 'generates' energy Q per unit volume per unit time, Equation (14.3) becomes

peP (oT!ot) = k V 2 T +Q (14.4)

For one-dimensional steady-state heat conduction, Equation (14.4) becomes

(14.5)

and, in the absence of an internal energy source

(14.6)

14.3 A General The range of problems in heat conduction that can be tackled analytically is restricted to
Approach to Solving a small number of simple geometries but, within this limitation, a standard approach may
usually be followed.
Heat Conduction
Problems (a) First it is necessary to choose an appropriate geometrical coordinate system for the
problem and to identify the independent variable that determines the temperature.
(b) Next, the appropriate differential equation should be written down, starting either
with Fourier's rate equation (Equation 14.1) or Fourier's equation (Equations 14.3
or 14.4).
(c) A general solution of the differential equation is then obtained and ...
(d) ... the boundary conditions are used to determine the constants of the integration.

192
At this stage the solution should be checked to see that it makes sense, for
example, that it predicts the correct outcome in a simple situation.
(e) Finally, the calculation of the required quantities is carried out.

This procedure will now be illustrated with a number of simple one-dimensional


examples, the first of which will be structured explicitly to show the above approach.

14.4 Worked
Examples on Heat
Conduction 14.1 Determine the steady-state temperature distribution in a perfectly lagged cylindrical bar of length L
that has one end face maintained at a temperature T1 and the other at a temperature T2 (T1 > Tz).

Solution The condition of perfect lagging ensures that there is no heat loss from the cylindrical surfaces of
the rod so that the heat flow is one-dimensional.
(a) Choose the origin of orthogonal coordinates at the centre of the face of the bar at a
temperature T1 and let the x-axis coincide with the axis of the bar. The temperature distribution is
then a function of x only.
(b) Since conditions are steady there is no temperature variation with time and Fourier's
equation becomes

(c) Integrating this equation twice gives

where cl and c2 are constants.


(d) The values of C1 and C2 are obtained from the conditions that T equals T1 when xis zero,
and equals T2 when x equals L. Then

and

giving

and the general solution is

[Check: when x = L/2, T = (T1 + T2 )/2]


(e) If the rate of heat flow dQ/dt through an area A is wanted, this is obtained from Fourier's rate
equation

dQ/dt =- kA x (temperature gradient)

The temperature gradient is given by

dT/dx = - (T1 - T2 )1L

so that

dQ!dt = kA[(T1 - T 2 )1L]

193
14.2 A composite bar is made of three uniform cylindrical rods of the same cross-section joined
end-to-end. One rod is of copper and is 0.12 m long, one is of glass and is 0.01 m long and the
other is of iron and is 0.04 m long. The parts are bonded together in the order copper, glass and
iron and are in good thermal contact. The free copper end-face is maintained at a temperature of
373 K and that of the iron is maintained at a temperature of 273 K. If the whole bar is well lagged,
calculate the temperatures of the two faces of the glass rod when conditions have become steady.
The values of the thermal conductivities are

copper: 420 W m- 1 K- 1
glass: 0.90 W m- 1 K- 1
iron: 82 W m- 1 K- 1

Solution The arrangement is shown in Fig.14.2. Choose the origin of orthogonal coordinates to lie in the
end face of the copper rod that is maintained at a temperature of 373 K, and with the x-axis
coinciding with the common axis of the three rods.

L;

Figure 14.2 The composite or---------------------1-;-------r---x


bar. T0 =373K T, T2 T, = 273 K

Since the rods are well lagged the heat flow is one-directional in the direction of the positive
x-axis. When conditions are steady, Equation (14.2) shows that the temperature gradient in each
rod is a constant. It then follows from Equation (14.1) that the heat flow per unit area per unit time
must have the same value (dQ/dt) for all cross-sections of the composite rod.
Let the temperature of the free end of the copper rod be T0 , that of the free end of the iron rod
be Tf, that of the copper-glass interface be T1 and that of the glass-iron interface be T2 and let the
lengths of the rods be, respectively, Lc, Lg and Li, as shown in Fig. 14.2. Fourier's rate equation
(Equation 14.1) when applied to a lagged bar in a steady state takes the form

(dQ/dt)IA = -k x (temperature gradient)

as shown in Example 14.1. Applying this equation to the copper rod gives

(14.7)

for the glass rod

(14.8)

and for the iron rod

(14.9)

Adding Equations (14.7), (14.8) and (14.9) gives

which may be written

(dQ/dt)/A = K(T0 - Tt)

where

The value of 1/K is equal to

1/K = (0.12 (m)/420 (W m- 1 K- 1)) + (0.01 (m)/0.90 (W m- 1 K- 1))


+ (0.04 (m)/82 (W m- 1 K- 1 ))
118.846 X 10- 4 m2 K w- 1
194
so that

(dQ!dt)IA = (373 (K) - 273 (K)) x (11118.846 x w- 4 (m 2 K w- 1 ))


= 8414.2469 W m- 2

Substituting this value in Equation (14.7) gives

8414.2469 (W m- 2 ) = [(420 (W m- 1 K- 1))/0.12 (m)](373 (K)- T 1 )

or,

(8414.2469 X 0.12)/420 = 373 - T1

which gives

T 1 = 370.596 K
= 370.6 K
Similarly, from Equation (14.8),

Therefore,

T2 = T1 - [(dQ/dt)IA][Lglkg]
= 370.5959 (K)- (8414.2469 (W m- 2 ))(0.01 (m)/0.90 (W m- 1 K- 1))
= 277.1043 K
= 277.1 K

From Equation (14.9), as a check,

T2 - Tt = [(dQ/dt)IA][LJkd

T2 - 273 (K) = (8414.2469 (W m- 2 ))(0.04 (m)/82 (W m- 1 K- 1))

or,

T2 = 277.1 K
14.3 Determine the heat flux through a slab of material 0.10 m thick when conditions are steady. The
surfaces of the slab are maintained at temperatures of 400 K and 300 K, respectively, and the
thermal conductivity k of the material at a temperature T is given by

k = k 0 [1 + b(T - T0 )]

where

k 0 = 0.16 X 103 W m- 1 K- 1
b = 2.0 x w- 4 K- 1
T0 = 273 K

Solution Consider a parallel-sided slab of semi-infinite extent with its surfaces maintained at temperatures
T 1 and T2 , respectively. From symmetry considerations the direction of the heat flow must be
normal to the plane of the slab. Choose an orthogonal coordinate system with its origin in the face
of the slab at a temperature T1 and its x-axis normal to the plane of the slab and directed towards
the face at a temperature T2 • The temperature is then a function of x only.
When conditions are steady the rate of heat flow dQ/dt through an area A normal to the x-axis is
a constant, the value being given by the Fourier rate equation:

195
(dQ/dt)/A =- k(dT/dx)

where k is the thermal conductivity of the material of the slab. If k is given by

k = k 0 [1 + b(T - T0 )]
then

(dQ/dt)/A = -k0 [1 + b(T- T0 )](dT/dx)

Separating the variables gives

- [(dQ/dt)lk0 A] dx = [1 + b(T- T0 )] dT

which, on integrating, becomes

- [(dQ/dt)lk0 A] X = T + (bT 212) - bT0 T + C


where Cis the constant of integration. If the slab is of thickness L, T = T 1 when x = 0 and T = T2
when x = L. Therefore the equation becomes

or,

[(dQ/dt)/k0 A] L = (T1 - T2 )(1 - bT0 ) + (b/2)(T~ - TD

Now, T1 = 400 K, T2 = 300 K and L = 0.10 m. Therefore,

[(dQ/dt)/A] = (0.16 X 103 (W m- 1 K- 1 )/0.10 (m))


X [(400 (K) -300 (K))(1- 2.0 X 10- 4 (K- 1 ) X 273 (K))
+ (2.0 x w- 4 (K- 1)/2)(400 2 (K2 ) - 3002 (K2 ))]
162.5 kW m- 2

14.4 Internal energy is generated at a uniform rate (dQ/dt) in each unit volume of a slab of material of
large surface area and of thickness 2L. The material has a constant thermal conductivity k. If both
surfaces of the slab are maintained at a temperature T 0 , show that, when conditions are steady, the
temperature T at a distance x from the mid-plane of the slab is given by

T(x) = T0 + [(dQ/dt)/2k][L2 - x2 ]

Solution Take the origin of orthogonal coordinates on the mid-plane of the slab, with the x-axis normal to
the plane of the slab.
Symmetry indicates that the temperature Tin the slab is a function of x only, that is,

T = T(x)

Then, if (dQ/dt) is the rate of internal energy generation in each unit volume of the slab and Tis
the temperature in a plane at a distance x from the origin of coordinates, for steady conditions, the
Fourier Equation (14.4) becomes

d[k(dT/dx)]/dx + (dQ/dt) =0
or, since k is a constant,

Integrating this equation gives

k(dT/dx) + (dQ/dt) X = c
196
where Cis the constant of integration. Because of the symmetry of the situation (dT/d.x) is zero
when xis zero. Therefore, Cis zero and

k(dT/dx) + (dQ/dt) x = 0

Integrating this equation gives

kT + (dQ/dt) (x2/2) = B

where B is the constant of integration. When x = L the value of Tis T0 • Therefore,

kT0 + (dQ/dt) (L 2/2) = B

Substituting this value for B gives

kT + (dQ/dt) (x 212) = kT0 + (dQ/dt) (L 2/2)


or,

T = T0 + [(dQ/dt)/2k)(L2 - x2)

14.5 Heat In a fluid the major heat transfer usually occurs by a bulk movement of the fluid which
Convection transports the energy with it. This process is known as convection and the flow of fluid is
known as a convection current. When convection currents are a consequence of the
changes in the density of the fluid arising from temperature gradients within the fluid, the
process is called natural convection. When the fluid flow is produced by an external
agency, such as a pump or a fan, the process is called forced convection.
The transfer of heat from a solid to a fluid is a complex process, involving the
conduction of heat from the surface of the solid to the fluid and, in particular, to the
stagnant layer of fluid in contact with the solid surface (the boundary layer), followed by
heat flow arising from the motion of the fluid itself, which flow may be streamline or
turbulent. The net effect of these processes is incorporated in the definition of a heat
transfer coefficient (or convection coefficient) h for the particular solid-fluid interface. If
the rate of heat transfer over an area A from a solid surface at a temperature T8 to a fluid
with a bulk temperature Tt is dQ/dt, the heat transfer coefficient h is defined by the
equation

(dQ/dt) = hA(Ts - Tt) (14.10)

The value of h depends on a number of factors, including whether the solid wall is
curved or flat, horizontal or vertical, whether the contacting fluid is a liquid or a gas, the
density, viscosity, specific heat capacity and thermal conductivity of the fluid, and
whether the fluid flow is streamline or turbulent. The determination of heat transfer
coefficients is a major activity in the field of heat convection.
However, for the forced convection cooling in air of normal laboratory calorimeters, h
may usually be taken as a constant, and then

(dQ/dt) = h 0 A(Ts- Tt) (14.11)

where h0 is a constant. Equation (14.11) is known as Newton's law of cooling. When the
calorimeter is in still air, so that natural convection takes place, his proportional to (Ts-
Tt) 114 when the air flow is streamline and proportional to (Ts - TtY 14 when the air flow is
turbulent. The corresponding equations for the rate of loss of heat from a surface of area
A are, respectively,

(dQ!dt) = h~ A(Ts - Tt) 514 (14.12)

for streamline flow and

197
(dQ!dt) = h~A(Ts - Tft' 3 (14.13)

Equation (14.12) is known as Dulong and Petit's law.

14.6 Worked
Examples on
Heat Convection t4.s In a certain chemical process it is necessary to raise the temperature of a certain batch of reactants.
The reactor in which this is done consists of a cylindrical tank fitted with a coil through which
high-pressure steam at a temperature of 453 K is passed. Estimate the time needed to raise the
temperature of the reactants from 333 K to 433 K, using the following information.
The steam coil is a stainless steel tube with outside diameter of 7.0 em, wall thickness 0.5 em and
total length 50.0 m. The reactor contains 6000 kg of reactants, having a mean specific heat capacity
of 2.4 x 103 J kg- 1 K- 1 • The heat transfer coefficient for the outer surface of the coil and the
reactants is 0.92 kW m- 2 K- 1 and that for the steam-inner coil surface is 8.6 kW m- 2 K- 1 • The
thermal conductivity of stainless steel is 45 W m- 1 K- 1 •

Solution This problem may be considered in two parts:

(a) The determination of the rate of heat transfer from the condensing steam through the wall of
the coil to the reactants.
(b) The determination of the variation of the temperature of the reactants with time.

(a) Steam condenses on the inner surface of the coil, giving up its enthalpy of vaporisation, and
the heat is conducted through the wall of the coil to its outer surface, where it is transferred by
convection to the bulk of the reactants.
Let dQ/dt be the rate of heat transfer at time t, Ai be the area of the internal surface of the coil
and AT the temperature difference between the steam and the internal surface of the coil (see Fig.
14.3).
The heat transfer coefficient hh for the transfer is given by

dQ/dt = hh Ai AT

This heat must be conducted through the wall of the coil.


Let Aw be the mean surface area of the inner and outer surfaces of the coil and x be the thickness of
the wall (as in Fig. 14.3). Then, if ATw is the temperature drop across the wall and k is the thermal
conductivity of the material of the wall, the rate of heat transfer through the wall is given by

Wall of T
coil

Figure 14.3 The temperature


distribution through the wall
of the coil.

198
This energy is transferred to the reactants by convection. Let A 0 be the external surface area of the
coil and ATe the difference between the temperature of the outer surface of the coil and the bulk
temperature of the reactants, as in Fig. 14.3. A heat transfer coefficient he for the wall and
reactants is defined by

When conditions are steady, dQ!dt is uniform throughout the wall of the coil and the boundary
layers.
Define a total heat transfer coefficient U by the equation

where T 5 is the temperature of the steam and Tr is that of the reactants. A 0 is used here rather than
Aw because the temperature of the outer surface of the coil is much closer to T 5 than to Tr.
By definition

so that

This procedure eliminates AT, ATw and ATe, which are not easily measured.
(b) The variation of the temperature Tr of the reactants with time t is obtained from the
equation

where cP is the specific heat capacity at constant pressure of the reactants and M is their mass. If the
temperature of the reactants changes from Ti to Tt in a time t0 , separating the variables gives

so that

The numerical values of Ai, A 0 , Aw and U must now be calculated.

where L is the length of the steam coil and Di is its inside diameter. Therefore,

Ai 50.0 (m) X :n: X 6.0 X 10- 2 (m)

9.425 m 2

Now,

A 0 = LX Jt X D0

where D 0 is the outside diameter of the steam coil. Therefore,

A0 50.0 (m) X :n: X 7.0 X 10- 2 (m)

10.996 m 2

Next,

The value of U is given by

199
1/U = A 0 /hhAi + A 0 1[(k/x)Aw]+ llhc
= 10.996 (m2 )/(8600 (W m- 2 K- 1 ) x 9.425 (m2 ))
+ 10.996 (m 2 )/[(45 (W m- 1 K- 1 )/0.005 (m))
X 10.210 (m2 )] + 1/920 (W m- 2 K- 1 )

that is,

U = 447.98 W m- 2 K- 1

Therefore,

t0 {McP ln[(Ts - TJ!(Tt - Ti)]}IUA 0


(6000 (kg) X 2.4 X 10- 3 (J kg- 1 K- 1 ) X 1.792) + (447.98 (W m- 2 K- 1) X 10.996 (m 2 ))
= 5238.5 s
= 87 min

14.6 A window is made of glass of thickness 2.0 mm having a thermal conductivity of 0.9 W m- 1 K- 1 •
Calculate the rate of heat transfer through unit area of the window when conditions are steady and
the temperature of the room containing the window is 299 K while the temperature of the outside
air is 273 K, using the following approaches.

(a) Assume that the inside and outside surfaces of the glass are at temperatures of 299 K and 273
K, respectively.
(b) Assume that the heat transfer coefficient for a vertical plate is 1.78 x (~T) 114 J s- 1 m- 2 K-\
where ~Tis the difference in temperature between the glass surface and the bulk temperature
of the air.

Solution Assume that the heat flow is unidirectional, normal to the plane of the window, and let this
direction be the x-direction of an orthogonal coordinate system. Then the temperature T is a
function of x only.
(a) This is a rather simplistic approach that assumes that each surface of the glass is at the bulk
temperature of the air with which it is in contact and ignores the effect of the boundary layer. Then,
the rate of heat flow per unit area dQ/dt is given by the Fourier rate equation

dQ/dt = -k(dT/dx)

where k is the thermal conductivity of the glass. When conditions are steady dQ/dt is a constant
and the equation gives, on integration,

dQ/dt = k(Tr - T 0 )/L

where Tr is the temperature of the air in the room, T0 is the temperature of the outside air and L is
the thickness of the glass. Therefore, the rate of heat transfer per unit area is

dQ/dt [0.9 (W m- 1 K- 1 ) X (299 (K) - 273 (K))]/2.0 X 10- 3 (m)


11.7 x 103 W m- 2

(b) This is a more realistic approach. When conditions are steady the rate of heat transfer per
unit area by convection from the room to the inner surface of the glass must equal the rate at which
heat is conducted per unit area through the glass and must also equal the rate of heat transfer by
convection from the outer surface of the glass to the bulk of the outside air.
If Ti is the temperature of the inner surface of the glass and Tt is the temperature of the outer
surface of the glass, the steady-state condition becomes

assuming that h has the same value for both sides of the window.

200
Since T; and Tf are not known, an approximate value of dQ!dt must be obtained from which
these temperatures may be estimated.
As a first approximation, assume that the temperature drop across the glass is zero. To maintain
a constant heat flow when the heat transfer coefficient has the same values for the inner and outer
surfaces of the window requires that the temperature of the glass be (Tr + T0 )/2. Then,

dQ!dt h[Tr - (Tr + To)/2]


h(Tr - T0 )/2

The value of h is given by

h = 1.78 (~Tr 14 J s- 1 m- 2 K- 1
1. 78 X (13) 114
= 3.38 J s- 1 m- 2 K- 1

and, therefore,

dQ/dt = 3.38 (J s- 1 m- 2 K- 1 ) X 13 (K)


= 104 W m- 2

The temperature gradient across the glass cannot be zero. It must be sufficient to provide a heat
flux of 104 W. Using the Fourier equation

dQ!dt = (k/L) (T; - Tt)

gives

T; - Tt = (104 (W m- 2 ) X 2.0 X 10- 3 (m))/0.9 (W m- 1 K- 1 )


= 0.23 K

It is, therefore, sufficiently accurate to say that the inner surface of the glass is at a temperature of
286 + 0.17 = 286.2 K and that the outer glass surface is at a temperature of286- 0.17 = 285.8 K
and that the rate of heat flow through the window is 104 W m- 2 • This result should be compared
with the result of the calculation in (a).

14.7 A closed metal vessel containing a liquid has a surface area of 0.014 m 2 and a heat capacity of 490 J
K- 1 • When the vessel is placed in a current of air at a temperature of 293 K its temperature T varies
with timet as shown in the table. Draw a suitable graph to show that Newton's law of cooling is
satisfied in this situation and determine the value of the heat transfer coefficient.
If a small resistor is placed in the vessel and the power dissipated is 25 W, determine the steady
temperature of the vessel and its contents.

t!min 0 2 4 6 8 10 12
TIK 308.1 304.7 301.1 298.7 297.2 295.9 295.2

Solution For a body of constant heat capacity CP at a temperature T, losing heat through forced convection
in surroundings that are at a temperature T,, Newton's law of cooling may be written

dQ/dt = Cp(dT!dt) -hA(T- T.)


-K(T- T.)

where h is the (constant) heat transfer coefficient, A is the surface area of the body and K is a
constant, equal to hA. The minus sign is included because the body can here be clearly identified as
the system of interest and the direction of heat flow must be down the temperature gradient.
Integrating the above equation gives

ln(T - T,) = - (K!Cp) t + constant

201
Therefore, if Newton's law is valid in the situation considered, a graph of ln(T - T.) against t
should give a straight line plot. This graph is shown in Fig. 14.4, from which it can be seen that
Newton's law holds.

2.8

2.4

2.0

;: 1.6
I
1-
1.2
E
0.8

0.4

Figure 14.4 Graph of 0 2 4 6 8 10 12


ln(T - T 8 ) against t. t/min

The slope of the line is - K/CP, so that h is given by

h = -(Cp/A) x slope of graph


= - (490 (J K- 1 )/0.014 (m2 )) X (- 2.82 X 10- 3 (s- 1))

= 98.7 W m- 2 K- 1
With the small resistor dissipating 25 W and conditions steady

dQ/dt = - hA(T- T8 )

or,

-25 (W) = -98.7 (W m- 2 K- 1) X 0.014 (m2 ) X (T- 293 (K))

or

T = 293 + [25/(98.7 X 0.014)]


= 311 K

14.7 Thermal Thermal radiation is the electromagnetic radiation emitted by a liquid or a solid by virtue
Radiation of its temperature. This radiation is characterised by a continuous spectrum extending
over all wavelengths, though the energy deosity in the spectrum depends on the nature
and temperature of the emitting surface. Thermal radiation is emitted independently of
the nature and temperature of the surroundings. Consequently, when two or more
bodies, exchanging energy by radiation only, reach equilibrium, it is only the net effect
that is observed. Each body radiates appropriately to its surface and temperature but, in
equilibrium, each body absorbs thermal radiation from all the other bodies at the same
rate at which it emits radiation. This is Prevost's theory of exchanges.
The total thermal radiation emitted by unit area of the surface of a body in unit time is
the radiant emittance R of the surface. For a given surface, R is a function R(T) of the
temperature of the surface. The total thermal energy emitted per unit time from the
surface of a body is called the radiant flux <j>.
A surface that emits thermal radiation lying in the wavelength range A to A + dA of
amount eA dA per unit area per unit time is said to have a spectral emissive power eA.

202
The fraction of the isotropic thermal radiation falling on a surface that is absorbed by
that surface is the absorptance a. The absorptance is a function of the nature of the
surface and its temperature.
A black body is an ideal surface that is able to absorb all the thermal radiation that falls
on it at all temperatures. For a black body the absorptance aB is equal to unity. The
radiant emittance RB of a black body is a function of temperature only and is given by

(14.14)

where Tis the temperature of the black surface and o is a constant. Equation (14.14) is
known as the Stefan-Boltzmann law and o as the Stefan-Boltzmann constant.
The irradiance H within an evacuated cavity with opaque walls is the radiant energy
falling in unit time on a surface of unit area placed anywhere within the cavity. It is
readily shown that the thermal radiation within a cavity is isotropic and that H is a
function of temperature only and does not depend on the nature of the walls of the
cavity. Therefore, if a black surface at a temperature Tis placed within a cavity whose
walls are also at a temperature T, it may be shown that

H(T) = RB(T) (14.15)

that is, the irradiance within an evacuated cavity with opaque walls maintained at a
temperature T is equal to the radiant emittance of a black body whose surface is at a
temperature T.
Similarly, for a given non-black body

R(T) = a RB(T) (14.16)

that is, the radiant emittance of any surface at a temperature T divided by the radiant
emittance of a black body at the same temperature is equal to the absorptance of the
non-black surface. This result is known as Kirchhoff's law. The ratio R(T)IRB(T) is often
known as the surface emissivity E.
The spectral emissive power e"-s of a black body varies with wavelength, as shown in
Fig. 14.5. As the temperature of the surface is varied the wavelength A.., at which the
spectral emissive power has its maximum value varies in accordance with the law

A.., T = constant (14.17)

a result known as Wien's displacement law.

Figure 14.5 The spectral


emissive power of a black
body eA.B as a function of
wavelength A for two different
temperatures. The dashed T = 1000 K
curve gives the locus of the
maxima. 0

203
14.8 Worked
Examples on
Thermal
Radiation 14.8 A perfectly black long cylinder of radius r is maintained at a constant temperature Tin evacuated
surroundings that are at a constant temperature T0 • Calculate the net rate of energy exchange
between the body and its surroundings per unit length of the body.
Determine the effect of placing a thin cylindrical radiation shield of radius R, blackened on both
surfaces, coaxially with the body.

Solution The solution to this problem involves the emission of thermal radiation by a black body and the
behaviour of a radiating body in a cavity.
Any small region of the surface of the black cylinder, the radiation shield or the surroundings
will radiate in all directions, but the cylindrical symmetry of the arrangement considered means
that the behaviour is independent of the distance along the common axis.
Consider the situation without the radiation shield. If the black body were at a temperature T0 in
the same surroundings it would be in thermal equilibrium and would, therefore, absorb radiant
energy from the surroundings at the same rate as it emitted such radiation. This rate of emission
would be

2nroT6

per unit length per unit time. This is, therefore, the rate at which radiant energy is absorbed from
the surroundings. When the black body is at a temperature Tit emits thermal radiation per unit
length per unit time of amount

Therefore, with the black body at a temperature T and the surroundings at a temperature T0 , the
net rate of exchange of radiant energy with the surroundings per unit length of the black body is
dQ/dt, given by

dQ/dt = 2n r o (T 4 - 76)
where o is the Stefan-Boltzmann constant. Since this net energy exchange is a consequence of the
temperature difference between the body and its surroundings, it is a heat flow and is, therefore,
denoted by dQ/dt.
When the radiation shield is in place let its equilibrium temperature be T,. The net exchange of
radiant energy per unit length per unit time between the shield and the surroundings dQ' /dt is
given by

dQ'/dt = 2n R o (Ti- Tri)

using arguments similar to those used above.


If the black body were at the temperature of the shield it would emit radiation per unit length
per unit time of amount

2nroT'i

and would absorb thermal radiation at the same rate. This rate of absorption is independent of the
temperature of the black body which is, in fact, radiating energy

per unit length per unit time. Therefore, the net rate of energy exchange per unit length per unit
time between the black body and the shield is dQ"/dt, given by

dQ"/dt = 2n r o (T 4 - Ti)
But, when the shield is in thermal equilibrium,

dQ' /dt = dQ"/dt

204
so that

or,

T{(R + r) = rT 4 + RTri
so that

dQ"!dt 2:rc r a {T 4 - [(rT 4 + RTri)I(R + r)]}


2:rc r a {[R(T 4 - Tri)]/(R + r)}

Therefore, the effect of the radiation shield is to change the net rate of loss of radiant energy per
unit length of the black body from dQ/dt to dQ"Idt, where

(dQ"/dt)/( dQ!dt) {R(T 4 - Tri)I[(R + r)(T 4 - Tri)]}


RI(R + r)

14.9 Repeat the calculation of Example 14.8 for the situation where the surface of the radiation shield is
not perfectly black, but has a surface emissivity E.

Solution The net rate of exchange of energy per unit length of the body when the radiation shield is not in
place is, of course, given by

dQ/dt = 2:rc r a (T 4 - Tri)

When the radiation shield is in position let its equilibrium temperature be T2 • With regard to the
surroundings, the shield appears to be in a large evacuated cavity with its walls at a temperature
T0 • If this effective cavity is so large that the cavity radiation field within it is not appreciably
disturbed by the introduction of the shield, the radiation falling on unit length of the shield per unit
time is

2:rcRaTri

of which the shield now absorbs

E2:rcRaTri

The radiant energy emitted by the shield towards the surroundings per unit time per unit length is

2:rcReaTi

Therefore, the net radiation exchange per unit length per unit time between the shield and the
surroundings is dQ' ldt, given by

dQ'!dt = 2:rc REa (Ti- Tri)

For the black body, however, the shield appears as an enclosure with walls at a temperature T2 so
that the net rate of exchange of radiant energy per unit length is given by

dQ"!dt = 2:rc r a (T 4 - T;)

When the shield is in thermal equilibrium

dQ' !dt = dQ"/dt

so that

205
r(T 4 - 7i) = E R(Ti - 16)
or,

7i = (rT 4 + eR7ci)/( ER + r)

Therefore, in this situation,

(dQ"/dt)/( dQ/dt) = eR/( ER + r)

14.10 The solar constant is the total radiant energy falling on unit area in unit time at the Earth from the
Sun, measured over all wavelengths, and has the value 1.367 kW m- 2 •
Calculate the total rate of radiant energy emission by the Sun and determine a value for the
effective temperature of the surface of the Sun, assuming that the Sun radiates as a black body.
The distance from the Earth to the Sun is 1.469 x 1011 m and the radius of the Sun is 6. 96 x 108 m.

Solution If R is the distance from the Earth to the Sun, the surface area of a sphere of radius R, centred on
the Sun, is 4:rtR 2 • The radiant flux from the Sun is uniformly distributed over this surface, the flux
per square metre being the solar constantS. Therefore, the total radiant energy flux from the Sun
is E, given by

E 4:rtR2S
4:rt X (1.496 X 1011 ) 2 (m2 ) X 1.367 X 103 (W m- 2 )
3.84 X 1026 W

Assuming that the Sun behaves as a black body with an effective surface temperature T, the
value of E is also given by

where r is the radius of the Sun and o is the Stefan-Boltzmann constant, equal to 5.67 x 10- 8 W
m- 2 K- 4 • Therefore,

3.84 X 1026 (W)

which gives

T- 5800 K

14.11 Determine a value for the surface temperature of the Earth, assuming that it radiates as a black
body in thermal equilibrium and that its temperature is constant over the day-night cycle. Take the
effective surface temperature of the Sun to be 5800 K, the radius of the Sun to be 6. 96 x 108 m and
the distance from the Earth to the Sun to be 1.496 X 1011 m.

Solution Assume that the Sun is a black body with a surface temperature T5 • Then, if the radius of the Sun is
r5 , the total radiant flux from the Sun is given by

where o is the Stefan-Boltzmann constant. At a distance R from the Sun this flux is distributed
uniformly over a sphere of surface area 4:rtR 2 , giving a radiant flux per unit area at this distance
from the Sun of

If R is, in fact, the distance from the Earth to the Sun, the radiation falling on the surface of the
Earth per unit time is given by

206
where rE is the radius of the Earth. The radiant energy emitted by the Earth, assuming that it
radiates as a black body at a temperature TE, is given by

If the Earth is in equilibrium, the rates of emission and absorption of radiant energy must be equal
so that

or,

TE (r5 /2R) 112 X T5


[(6.96 X 108 (m)/2 X 1.496 X 1011 (m))] 112 X 5800 (K)
280 K

14.12 Two infinite parallel conducting planes are made of a material with an absorptivity a (equal to the
surface emissivity E) and are placed in an evacuated region. If the temperature of one plane is
maintained at a value T1 and the other at T2 , with T1 greater than T2 , show that the net rate of heat
transfer between them by radiation, per unit area, is given by

[ao(Ti - Ji)]/(2 - a)

where o is the Stefan-Boltzmann constant.

Solution By symmetry, the effective direction of energy exchange between the two parallel planes is in a
direction normal to the two planes. Consider the energy emitted from each plane in a small interval
of time and follow the progress of each 'packet' of radiant energy as it is successively partially
absorbed and partially reflected as it passes back and forth between the two planes.
Let the plane at a temperature T1 be designated piane 1 and that at a temperature T2 be
designated plane 2. The net heat transfer between the two planes, per unit area per unit time, when
conditions are steady, is equal to the energy emitted by plane 1 per unit area per unit time, less the
amount of that energy that is eventually absorbed by plane 1, and also less the amount of the
energy emitted per unit area per unit time by plane 2 that is absorbed by plane 1.
Let the energy emitted per unit area of plane 1 in unit time be E 1 • The fate of this packet of
radiant energy is as follows:

plane 1 emits energy E 1


plane 2 absorbs energy aE1
plane 2 reflects energy E 1 (1 - a)
plane 1 then absorbs energy E 1(1 - a)a and reflects energy E 1(1 - a)(1 - a)
plane 2 then absorbs energy E 1 (1 - a)(1 - a)a and reflects energy E 1 (1 - a)(1 - a)(1 - a)

and so on.
The energy E 2 emitted from unit area of plane 2 in unit time experiences a similar fate:

plane 2 emits energy E 2


plane 1 absorbs energy aE2 and reflects energy E 2 (1 - a)
plane 2 absorbs energy Ez(1 - a)a and reflects energy E 2 (1 - a)(1 - a)

and so on.
Now add up all the energy that is finally absorbed by plane 1 and all the energy that is finally
absorbed by plane 2. If the fraction (1 - a) 2 is denoted by y, that part of the energy E 1 that plane 1
absorbs is Q1 , given by

Q1 E1(1 - a)a + E 1(1 - a) 2 (1 - a)a + ....


E 1(1 - a)a (1 + y + y 2 + ... )

207
Since

11(1 - y) = (1 - yt 1 = 1 + y + y 2 + ...

the expression for Q1 may be written

Ql = [E1 (1 - a)a]/(1 - y)

Similarly, that part of the energy E 2 that is absorbed by plane 1 is Q2 , given by

Therefore, the net energy transfer between unit area of planes 1 and 2 in unit time is Q, given by

Q El- Ql- Q2
E1 - {[E1 (1 - a)a]/[1 - y]} - [E2a/(1 - y)]
[E1 (1 - y) - E 1a + E 1a 2 - E 2a]/(1 - y)
Putting y = (1 - a) 2 gives

E 1 {1 - 1 + a + a- a 2 - a + a 2 - (E2 /E 1 )a}
Q
{1 - 1 + a + a - a 2 }
[(E1 - E 2)a]/[2a - a] = (E1 - E 2)/(2 - a)

Now

and

so that

Q = [ao(T{- Ji)]/(2- a)

14.9 Exercises
14.1 A method for determining the thermal conductivity k of a and also the radial distance from the axis of the rod at which the
metal at liquid helium temperatures involves forming a sample temperature is (T1 + T2)/2. [[2nLk(T1 - T2)]/[ln(r2/r1 )]; (r1 r2) 112]
of the metal into a ring of radius r and length L. This wire is
made a thermal link between a copper block and a bath of liquid 14.4 Internal energy is generated at a uniform rate Q in each
maintained at a temperature T0 • A resistor and a thermometer unit volume of a slab of material of large surface area and
are attached to the copper block, which then has its temperature thickness L. If the surfaces are maintained at temperatures T1
raised to the value T1 • If the thermal conductivity of the metal at and T2 , respectively, with T1 greater than T2 , show that, when
these temperatures is given by k = aT, where a is a constant, conditions are steady, the maximum temperature TM within the
derive an expression for the power P that must be dissipated in slab is given by
the resistor to maintain the temperature of the copper block at
the value T1 • [P = (nr 2 a/2L)(T~- T5)]

14.2 Show that, when conditions are steady, the rate of heat if k is a constant.
flow dQ/dt through a spherical shell of material, having an exter-
nal radius r2 and an inner radius r 1 , is given by 14.5 A tube of outside diameter 1.0 X 10- 2 m is in an environ-
ment where the heat transfer coefficient is 5.0 W m- 2 K- 1 • The
tube is then insulated with a layer of material of thickness 0.5 x
10- 2 m, having a thermal conductivity of 0.1 W m- 1 K- 1 • If the
temperature of the tube and the heat transfer coefficient are
where T1 is the temperature of the inner surface, T2 is that of the unaltered by this change, will the rate of heat transfer to the
outer surface and k is assumed to be a constant. environment be reduced?
Calculate the thickness of insulation that produces the lowest
14.3 A hollow cylindrical rod of length L has an inner surface rate of heat transfer from the tube to the environment.
of radius r 1 , maintained at a constant temperature T1 , and an [no; thickness = 1.0 x 10- 2 m]
outer surface of radius r 2 , maintained at a constant temperature
T2 , with T1 greater than T2 • Determine the rate of heat flow from 14.6 If it takes 3.0 minutes for the temperature of a certain
the inner surface to the outer surface when conditions are steady body to fall from 373 K to 333 K, what will its temperature be

208
after a further 10 minutes, assuming that the temperature of the which are maintained at a temperature of 273 K. Sphere A is of
surroundings is 283 K and that Newton's law of cooling is copper and its temperature falls from 276 K to 274 Kin 14.2 min.
obeyed? (290.1 K] Sphere B is made of aluminium and takes 10.04 min to undergo
the same fall in temperature. If the specific heat capacity of
14.7 A solid copper sphere of radius 150 mm has its tempera- aluminium is 903 J kg- 1 K-\ determine the value for copper.
ture raised to 470 K before being placed in a stream of air, the The density of aluminium is 2.70 x 103 kg m- 3 while that of
temperature of which is maintained at 300 K. Assuming that copper is 8.96 x 103 kg m- 3 • (903 J kg- 1 K- 1]
copper is a perfect heat conductor, calculate the time it will take
for the temperature of the sphere to fall from 470 K to 330 K. 14.12 In a tungsten filament lamp it may be assumed that,
The heat transfer coefficient may be taken as a constant equal to when conditions are steady, all the energy supplied to the fila-
20.0 W m- 2 K- 1 , the density of copper is 8.95 X liP kg m- 3 and ment is lost by radiation and that the radiation received from the
the specific heat capacity of copper may be taken as a constant surroundings is negligible. If the diameter of the filament is 0.32
equal to 0.383 kJ kg- 1 K- 1 • (248 min) x 10- 3 m, the absorptance of its surface is 0.3 and its operating
temperature is 2450 K, estimate the length of wire needed in a
14.8 A long straight cable carrying an electric current has a 100 W lamp. [16.2 em]
solid core of radius r 1 , covered by a concentric insulating layer
with an outer radius r 2 • When a current is flowing, the energy 14.13 The maximum value of the solar spectral emissive power
dissipated per unit length per unit time ('resistance heating') is occurs at a wavelength of 0.48 x 10-6 m and the effective tem-
Q. The insulating material has a constant thermal conductivity k perature of the Sun's surface is 6000 K. If another star has a
and the heat transfer coefficient for the surface of the insulating maximum in its spectral emissive power at a wavelength of 0.36
layer and the surrounding air has the constant value h. Show X 10-6 m, calculate its effective surface temperature. (8000 K)
that, when conditions are steady, the temperature difference
between the surface of the core and the surroundings is AT, 14.14 A blackened silver sphere of radius 1.0 em is suspended
given by in an evacuated enclosure with walls at a temperature of 273 K.
Embedded in the sphere is a small resistor carrying a current
which maintains the sphere at a uniform temperature of 360 K.
Calculate the power dissipated in the resistor to maintain the
14.9 A body of mass m, surface area A and specific heat steady temperature and also the initial rate of fall of the tem-
capacity at constant pressure cP, independent of temperature, is perature of the sphere when the current is switched off.
placed in surroundings at a constant temperature T5 • If the heat For silver, density = 10.50 X 103 kg m- 3 and specific heat
transfer coefficient his a constant and Newton's law of cooling is capacity at constant pressure= 236 J kg- 1 K- 1 • Both these values
obeyed, show that the temperature T of the body varies with may be taken as constants. (0.80 W; 4.6 K min- 1 ]
time t according to the equation
14.15 A certain design of Dewar vessel may be treated as two
concentric spherical surfaces with the space between them
evacuated. In a given vessel the radius of the inner surface is
where Ti is the temperature of the body at time zero. 0.50 m and that of the outer surface is 0.51 m. If the surfaces
enclosing the evacuated space are blackened, calculate the rate
14.10 A small black body at a temperature Tis placed inside a at which liquid nitrogen evaporates from the Dewar vessel when
large evacuated enclosure, the walls of which are maintained at a it is in surroundings at a temperature of 293 K and conditions are
constant temperature Tw· Show that, when (T- Tw) is small, the steady. The boiling point of liquid nitrogen under a pressure of
net rate of exchange of radiant energy between the body and the one atmosphere is 77 K and the specific latent heat of vaporisa-
walls of the enclosure is proportional to (T- Tw)· tion of nitrogen at this temperature and pressure is 200 x
103 J kg- 1 • (1.2 x w- 3 kg s- 1 ]
14.11 Two small blackened spheres A and B, having the same
radius, are placed in a large evacuated enclosure, the walls of

209
Recommended Reading

A textbook at the appropriate standard that covers all the topics treated in this book is Thermal
Physics (1991) by M. T. Sprackling (published by Macmillan).
Other books of a similar standard having a good treatment of the material, but a slightly less
wide coverage, are Equilibrium Thermodynamics (1983) by C. J. Adkins (published by Cambridge
University Press) and Thermal Physics (1986) by C. B. P. Finn (published by Routledge and Kegan
Paul). Readers who want an elegant and mathematically more advanced treatment of classical
thermodynamics should consult The Elements of Classical Thermodynamics (1957) by A. B.
Pippard (published by Cambridge University Press).

210
Index

absolute zero of temperature 50 conditions of equilibrium 2


unattainability of 163 conduction, heat 191-7
absorptance 203 conductivity, thermal 191
adiabatic boundary 2 conservation of energy 27
adiabatic equations 102 convection 197-202
for ideal gas 112 forced 197
adiabatic process 2, 26 natural 197
allotropic transformation 165-7 convection current 197
availability 90--100 cooling, Newton's law of 33, 37, 197
and useful work 90 coordinates, thermodynamic 1
extensive 1
Beattie-Bridgeman equation of independent 1
state 140 intensive 1
Berthelot's equation of state 140 primitive 1
black body 203 critical point 139
boundary 1 critical temperature 139
adiabatic 2 cubic expansivity 85, 102
diathermic 2 cyclic processes 43
Boyle point 15
Boyle's law 15-20, 176 Daniell cell 127
Boyle temperature 15, 178 degrees of freedom 1
diathermic boundary 2
calorimeter 32 displacement work 8
Carnot cycle 48 Dulong and Petit law of cooling 198
Carnot's theorem 48
corollary to 49 efficiency
cavity radiation 203 of Carnot engines 50
cell, voltaic 12~ of heat engines 43
Celsius temperature scale 53 elastic solid 3, 119-26
change of phase 147-59 electrical work 6, 32, 40
chemical equilibrium 2 e.m.f. of voltaic cell 126
chemical interaction 2 emissive power, spectral 203
Clapeyron-Clausius equation 153-7, 163 emissivity of a surface 203
Clausius' equation 157-9 energy, internal 26
Clausius' equation of state 140, 142 enthalpy 131
Clausius' inequality 61 and heat capacity 131
Clausius' statement of second law 43 of a phase change 147-53
Clausius' theorem 61 and throttling process 174
closed hydrostatic systems 8, 101-11, entropy 62, 76-81
161 form of first law 68-71
closed systems 1 and heat capacity 62
coefficient of performance 44 in irreversible processes 71
compressibility factor 144 law of increase of 71-5
compressibility, isothermal 85, 102 and work 66-8

211
equations of state 3, 139-45 heat transfer coefficient (convection
Beattie-Bridgeman 140 coefficient) 197
Berthelot 140 Helmholtz function (Helmholtz free
ideal gas 20--4, 183 energy) 82-6
van der Waals 140, 142, 143, 177 as a potential function 83
equilibrium
chemical 2 ideal gas 16, 111-19
conditions of 2 adiabatic equations of 112
mechanical 2 equation of state 20--4, 183
thermal 2 heat capacity relations 80, 133
thermodynamic 2 internal energy 31, 176, 183
equilibrium state 2 scale of temperature 20
exact differential 30 increase of entropy, law of 71
exchanges, Prevost's theory of 202 independent coordinates 1
expansivity inequality of Clausius 61
cubic 85, 102 inexact differential 30
linear 120 intensive coordinates 1
extensive coordinates 1 interactions 2, 5
chemical 2
figure of merit thermal 2
of heat pumps 44 work 2, 5
of refrigerators 44 internal energy 26
first law of thermodynamics 27-9 inversion curve 175
entropy form 68--71 inversion temperature 175
infinitesimal form 29 irradiance within a cavity 203
first-order phase change 147 irreversible processes 30, 168--82
Fourier's equation of heat isenthalp 175
conduction 192 isolated systems 1
Fourier's rate equation 191 isothermal compressibility 85, 102
free expansion 30, 168 isotherms 3, 15
of ideal gases 31
theory of 168--9 Joule coefficient 168
functions of state 2 Joule process 168--73
Joule's law 31, 176, 183
gas constant (molar) 20, 185 Joule-Thomson coefficient 174
gases, kinetic theory of 183-90 Joule-Thomson (throttling)
Gibbs function (Gibbs free energy) process 173-82
86-90
as a potential function 87 Kelvin's statement of the second law 48
Kelvin, unit of temperature 20
heat 27 kinetic theory of gases 183-90
dependence on process 29 Kirchhoff's laws
latent 147 for radiation 203
sign convention for 27 for electrical circuits 7
heat capacity 31-42, 130--9
of a closed hydrostatic system 31 latent heat 147
general relationships for law of the increase of entropy 71
principal 131-2 laws of thermodynamics
molar 32 first 27-9
principal 32 second 43-60
specific 31 third 161-7
heat conduction 191-7 zeroth 3
heat convection 197-202 linear expansivity 120
heat engines 43-8, 50--3
heat pumps 44, 56 Maxwell-Boltzmann distribution of
heat reservoirs 30, 64 speeds 184
heat transfer 191-208 Maxwell's relations 75

212
mean free path of a gas molecule 184, surface emissivity 203
189 surroundings 1
mean square speed 183, 185 systems 1
molar gas constant 120, 185 closed 1
molar heat capacity 32 isolated 1
open 1
Nernst's heat theorem (third law of
thermodynamics) 161 temperature 2
Newton's law of cooling 33, 37, 197 Boyle 15
Celsius 53
open systems 1 critical 139
empirical 3
path 2 inversion 175
perfectly elastic solids 3, 119-26 and kinetic theory of gases 184
phase changes 147-59 thermodynamic 49
phase equilibrium 153-4 universal functions of 49
Planck's statement of the third law 164 theorem of Clausius 61
porous plug experiment 173 thermal conductivity 191
pressure-volume (displacement) thermal diffusivity 192
work 8--12 thermal efficiency 43
Prevost's theory of exchanges 202 thermal equilibrium 2
primitive coordinates 1 thermal interaction 2
processes 2 thermal radiation 202-8
dissipative 30 thermodynamic coordinates 1
irreversible 30, 168--82 thermodynamic equilibrium 2
quasistatic 2, 30 thermodynamic potential functions 82-100
reversible 29 thermodynamic properties 1
thermodynamic temperature 49
quasistatic processes 2, 30 thermodynamic variables 1
thermometer 3
radiant emittance 202 thermodynamics, laws of
radiant flux 202 Clausius statement 43
radiation, thermal 202-8 first 27-9
black body 203 Kelvin statement 48
laws of 203 second 43-60
reciprocal theorem 75 third 161-7
reciprocity theorem 75 zeroth 3
refrigerators 44, 77 third law of thermodynamics 161-7
reservoirs, heat 30, 64 Nernst statement 161
reversible processes 29 Planck statement 164
root mean square speed 183 Simon statement 165
throttling process 173
second law of thermodynamics 43-60 total heat transfer coefficient 199
Clausius statement 43 triple point 139
Kelvin statement 48 triple temperature 139
sign conventions
for heat 27 unattainability of absolute zero 163
for work 5 universal temperature functions 49
specific heat capacity 31 useful work 8
spectral emissive power 202 and availability 90
state 1 useless work 8
equations of 3, 139-45
equilibrium 2 van der Waals equation of state 140,
state functions 2 142, 143, 177
state variables 2 voltaic cells 126-8
Stefan-Boltzmann constant 203
Stefan-Boltzmann law 203 Wien's displacement law 203

213
work 5, 26 work interaction 2, 5
done in charging a reversible electric work, pressure-volume
cell 126 (displacement) 8-12
done on a closed hydrostatic system 8
doneinextendinganelasticrod 12-14 Young's modulus 120
sign convention for 5
work, electrical 6, 32, 40
work and entropy 66 zeroth law of thermodynamics 3

214

You might also like